Tag Archives: DeThi

Đề và đáp án thi vào lớp 10 Chuyên Toán TPHCM năm 2022

Bài 1. (1,0 diểm)
Cho $x, y$ là hai số thực thỏa mãn $x y+\sqrt{\left(1+x^2\right)\left(1+y^2\right)}=1$.
Tính giá trị của biểu thức $M=\left(x+\sqrt{1+y^2}\right)\left(y+\sqrt{1+x^2}\right)$.
Bài 2. (2,5 diểm)
a) Giải phương trình $\sqrt{x+4}+|x|=x^2-x-4$.
Bài 3. (1,5 diểm)
Cho hình vuông $A B C D$ Trên các cạnh $B C$ và $C D$ lần lượt lấy các điểm $M$ và $N$ sao cho $\angle M A N=45^{\circ}$.
a) Chứng minh $M N$ tiếp xúc với dường tròn tâm $A$ bán kính $A B$.
b) Kẻ $M P$ song song với $A N$ ( $P$ thuộc đoạn $A B)$ và kẻ $N Q$ song song với $A M(Q$ thuộc đoạn $A D)$. Chứng minh $A P=A Q$.
Bài 4. (2,0 diểm)
Cho ba số thực dương $a, b, c$ thỏa $a+b+c=3$.
a) Chứng minh rằng $a b+b c+c a \leq 3$.
b) Tìm giá trị nhỏ nhất của biểu thức $P=\frac{a}{b^2+1}+\frac{b}{c^2+1}+\frac{c}{a^2+1}$.
Bài 5. (2,0 diểm)
Cho tam giác $A B C$ nhọn $(A B<A C)$ có các đường cao $A D, B E, C F$ cắt nhau tại $H$. Đường thẳng $E F$ cắt đường thẳng $B C$ tại $I$. Đường thẳng qua $A$ vuông góc với $I H$ tại $K$ và cắt $B C$ tại $M$.
a) Chứng minh tứ giác $I F K C$ nội tiếp và $\frac{B I}{B D}=\frac{C I}{C D}$.
b) Chứng minh $M$ là trung diểm của $B C$.

Bài 6. (1,0 diểm)
Số nguyên dương $n$ được gọi là “số tốt” nếu $n+1$ và $8 n+1$ dều là các số chính phương.
a) Hãy chỉ ra ví dụ ba “số tốt” lần lượt có 1, 2, 3 chữ số.
b) Tìm các số nguyên $k$ thỏa mãn $|k| \leq 10$ và $4 n+k$ là hợp số với mọi $n$ là “số tốt”.

Đáp án được thực hiện vởi Star Education

Bài 1.

Điều kiện: $x y \leq 1$. Biến đổi giả thiết
$$
\sqrt{\left(1+x^2\right)\left(1+y^2\right)}=1-x y \Leftrightarrow\left(1+x^2\right)\left(1+y^2\right)=(1-x y)^2 \Leftrightarrow(x+y)^2=0 \Leftrightarrow y=-x .
$$
Thay vào biểu thức $M$ ta được
$$
\begin{aligned}
M & =\left(x+\sqrt{1+y^2}\right)\left(y+\sqrt{1+x^2}\right) \
& =\left(x+\sqrt{1+x^2}\right)\left(-x+\sqrt{1+x^2}\right) \
& =\left(\sqrt{1+x^2}\right)^2-x^2=1
\end{aligned}
$$

Bài 2.

a)

Lời giải:
a) Điều kiện: $\left\{\begin{array}{l}x+4 \geq 0 \\\\ x^2-x-4 \geq 0\end{array} \right.$

$\Leftrightarrow\left[\begin{array}{l}-4 \leq x \leq \frac{1-\sqrt{17}}{2} \\\\ x \geq \frac{1+\sqrt{17}}{2}\end{array}\right.$
Phương trình đã cho tương đương
$$
x^2-\sqrt{x+4}-|x|-(x+4)=0 \Leftrightarrow(|x|+\sqrt{x+4})(|x|-\sqrt{x+4}-1)=0 \Leftrightarrow|x|-1=\sqrt{x+4}
$$

  • Nếu $x \geq 0,(1) \Rightarrow x-1=\sqrt{x+4}$
    $$
    \Rightarrow x^2-2 x+1=x+4 \Leftrightarrow x^2-3 x-3=0 \Leftrightarrow\left[\begin{array}{l}
    x=\frac{3+\sqrt{21}}{2} \text { (Nhận) } \\\\
    x=\frac{3-\sqrt{21}}{2} \text { (Loại) }
    \end{array}\right.
    $$
  • Nếu $x<0,(1) \Rightarrow-x-1=\sqrt{x+4}$
    $$
    \Rightarrow x^2+2 x+1=x+4 \Leftrightarrow x^2+x-3=0 \Leftrightarrow\left[\begin{array}{l}
    x=\frac{-1+\sqrt{13}}{2} \text { (Loại) } \\\\
    x=\frac{-1-\sqrt{13}}{2} \text { (Nhận) }
    \end{array} .\right.
    $$
    Thử lại, ta được $x=\frac{3+\sqrt{21}}{2}$ và $x=\frac{-1-\sqrt{13}}{2}$ là các nghiệm của phương trình đã cho.

b) Điều kiện: $(x+y)(y+z)(z+x) \neq 0$. Hệ dã cho tương dương
$$
\left\{\begin{array} { l }
{ \frac { x } { y + z } + 1 = 2 x } \\\\
{ \frac { y } { z + x } + 1 = 3 y } \\\\
{ \frac { z } { x + y } + 1 = 5 z }
\end{array} \Leftrightarrow \left\{\begin{array} { l }
{ \frac { x + y + z } { y + z } = 2 x } \\\\
{ \frac { x + y + z } { z + x } = 3 y } \\\\
{ \frac { x + y + z } { x + y } = 5 z }
\end{array} \Leftrightarrow \left\{\begin{array}{l}
x+y+z=2 x(y+z) \\\\
x+y+z=3 y(z+x) \\\\
x+y+z=5 z(x+y)
\end{array}\right.\right.\right.
$$
Dễ thấy $x y z \neq 0$. Từ trên suy ra
$$
2 x(y+z)=3 y(z+x)=5 z(x+y) \Leftrightarrow 2\left(\frac{1}{y}+\frac{1}{z}\right)=3\left(\frac{1}{z}+\frac{1}{x}\right)=5\left(\frac{1}{x}+\frac{1}{y}\right) .
$$
Ta tính được $\frac{1}{z}=\frac{19}{x}, \frac{1}{y}=\frac{11}{x} \Rightarrow x=11 y=19 z$. Thay lại vào phương trình $(*)$ ta dược
$$
x+\frac{x}{11}+\frac{x}{19}=2 x\left(\frac{x}{11}+\frac{x}{19}\right) \Leftrightarrow 1+\frac{1}{11}+\frac{1}{19}=2\left(\frac{x}{11}+\frac{x}{19}\right) \Leftrightarrow x=\frac{239}{60} .
$$
Suy ra $y=\frac{239}{660}, z=\frac{239}{1140}$.
Vậy nghiệm duy nhất của hệ là $(x, y, z)=\left(\frac{239}{60}, \frac{239}{660}, \frac{239}{1140}\right)$.

Bài 3.

a) Trên tia đối của tia $D C$ lấy $F$ sao cho $D F=B M$.
Xét $\triangle A D F$ và $\triangle A B M$ có $A D=A B, \angle A D F=\angle A B M=90^{\circ}$ và $D F=B M$.
Do đó $\triangle A D F=\triangle A B M(\mathrm{c}-\mathrm{g}-\mathrm{c})$
$\Rightarrow \angle D A F=\angle B A M$ và $A F=A M$.
Suy ra $\angle D A F+\angle D A N=\angle B A M+\angle D A N=90^{\circ}-45^{\circ}=45^{\circ}$.
$\Rightarrow \angle N A F=45^{\circ}=\angle N A M$, mà $A F=A M$ nên $\triangle N A F=\triangle N A M$. (c-g-c)
Kẻ $A E \perp M N(E \in M N) \Rightarrow A E=A D=A B \Rightarrow M N$ tiếp xúc với $(A, A B)$.
b) Ta có: $\triangle N A F=\triangle N A M \Rightarrow \angle A N F=\angle A N M$, mà $\angle A N F=\angle N A P($ do $D C | A B)$, dẫn đến $\angle A N M=\angle N A P$.

Từ $A N | M P \Rightarrow A P M N$ là hình thang, kết hợp với $\angle A N M=\angle N A P$, ta được $A P M N$ là hình thang cân.
Do đó $A P=M N$, tương tự ta cũng có $A Q=M N$, dẫn dến $A P=A Q$.

Bài 4.

a)

a) Ta có $a^2+b^2 \geq 2 a b, b^2+c^2 \geq 2 b c, c^2+a^2 \geq 2 c a$ nên
$$
2\left(a^2+b^2+c^2\right) \geq 2(a b+b c+c a) \Leftrightarrow a^2+b^2+c^2 \geq a b+b c+c a .
$$
Khi đó
$$
\begin{aligned}
9=(a+b+c)^2 & =a^2+b^2+c^2+2 a b+2 b c+2 c a \
& \geq a b+b c+c a+2(a b+b c+c a)=3(a b+b c+c a)
\end{aligned}
$$
Do đó $a b+b c+c a \leq 3$.
Dấu “=” xảy ra khi và chỉ khi $a=b=c=1$.

b)

b) Ta có
$$
\begin{aligned}
& \frac{a}{b^2+1}-a=\frac{-a b^2}{b^2+1} \geq-\frac{a b^2}{2 b}=-\frac{a b}{2} \
& \frac{b}{c^2+1}-b=\frac{-b c^2}{c^2+1} \geq-\frac{b c^2}{2 c}=-\frac{b c}{2} \
& \frac{c}{a^2+1}-c=\frac{-c a^2}{a^2+1} \geq-\frac{c a^2}{2 a}=-\frac{c a}{2}
\end{aligned}
$$
Do đó
$$
\begin{aligned}
& \frac{a}{b^2+1}+\frac{b}{c^2+1}+\frac{c^2}{a^2+1}-(a+b+c) \geq-\frac{a b+b c+c a}{2} \geq-\frac{3}{2} \
\Rightarrow & \frac{a}{b^2+1}+\frac{b}{c^2+1}+\frac{c}{a^2+1} \geq-\frac{3}{2}+a+b+c=\frac{3}{2}
\end{aligned}
$$
Vậy giá trị nhỏ nhất của $P$ là $\frac{3}{2}$, dấu “=” xảy ra khi và chỉ khi $a=b=c=1$.

Bài 5.

Vẽ dường tròn $(O)$ ngoại tiếp $\triangle A B C$
a) Ta có: Các tứ giác $A F D C, A K D I, B F E C, A F H E$ nội tiếp.
$\Rightarrow H F \cdot H C=H D \cdot H A=H K . H I \Rightarrow I F K C$ nội tiếp.
Mặt khác: $\widehat{I F B}=\widehat{A C B}=\widehat{B F D}$ (do các tứ giác $B F E C, A F D C$ nội tiếp)
$\Rightarrow F B$ là phân giác $\widehat{I F D}$.
Mà $F B \perp F C$ nên $F B$ là phân giác trong, $F C$ là phân giác ngoài $\triangle I F D$
$$
\Rightarrow \frac{B I}{B D}=\frac{C I}{C D}
$$
b) Gọi $S$ là giao điểm thứ hai của $I A$ và đường tròn ngoại tiếp $O$.
Ta chứng minh được $I F . I E=I B . I C=I S . I A$
$\Rightarrow A S F E$ nội tiếp hay 5 điểm $A, S, F, H, E$ cùng thuộc đường tròn đường kính $A H$
$\Rightarrow \widehat{A S H}=\widehat{A F H}=90^{\circ}$
Mặt khác do: $I K \perp A M, A D \perp I M$ nên $H$ là trực tâm $\triangle A I M \Rightarrow M H \perp A I$.
Từ đó, ta có: $S, H, M$ thẳng hàng.
Vẽ đường kính $A Q$ của đường tròn ngoại tiếp $\triangle A B C$.
Ta có $\widehat{A S Q}=90^{\circ}$ nên $S, H, M, Q$ thẳng hàng
Xét tứ giác $B H C Q$ có: $B H / / C Q$ (cùng $\perp A C)$ và $C H / / B Q($ cùng $\perp A B)$
Nên $B H C Q$ là hình bình hành nghĩa là có $M$ là trung điểm $B C$.

Bài 6.

Lời giải:
a) Ví dụ: $3\left(3+1=2^2\right.$ và $\left.8 \cdot 3+1=5^2\right), 15\left(15+1=4^2\right.$ và $\left.8 \cdot 15+1=11^2\right)$ và 120 $\left(120+1=11^2\right.$ và $\left.8 \cdot 120+1=31^2\right)$.
b) Nhận xét $a^2 \equiv 0,1(\bmod 3)$ với mọi $a \in \mathbb{N}$.
Đặt $n+1=x^2$ và $8 n+1=y^2(x, y \in \mathbb{N})$.

  • Nếu $n \equiv 1(\bmod 3)$ thì $x^2=n+1 \equiv 2(\bmod 3)$, vô lí.
  • Nếu $n \equiv 2(\bmod 3)$ thì $y^2=8 n+1 \equiv 17 \equiv 2(\bmod 3)$, vô lí.
    Vậy $n \equiv 0(\bmod 3)$ hay $n$ chia hết cho 3 .
    Nếu $k=1,5,7,-5,-7$ thì với $n=3$ (là số tốt), $4 n+k$ nhận các giá trị $13,17,19,7,5$ là các số nguyên tố. (Loại)
    Nếu $k=-1$, với $n=15$ (là số tốt) thì $4 n+k=59$ là số nguyên tố. (Loại)
    Nếu $k=-10$, với $n=3$ thì $4 n+k=2$ là số nguyên tố. (Loại)
    Nếu $k=-9$, với $n=3$ thì $4 n+k=3$ là số nguyên tố. (Loại)
    Nếu $k \geq-8, k$ chẵn hoặc $k$ chia hết cho 3 thì $4 n+k \geq 4 \cdot 3-8=4$ và $4 n+k$ có ước là 2 hoặc 3 , do đó $4 n+k$ là hợp số.
    Vậy các giá trị cần tìm của $k$ là
    $$
    k \in{-8,-6,-4,-3,-2,0,2,3,4,6,8,9,10} .
    $$

ĐỀ THI CHỌN ĐỘI TUYỂN QUỐC GIA CỦA CÁC TỈNH, THÀNH

ĐỀ THI CHỌN ĐỘI TUYỂN TRƯỜNG PHỔ THÔNG NĂNG KHIẾU

ĐỀ THI CHỌN ĐỘI TUYỂN HSG QUỐC GIA CỦA TRƯỜNG PTNK NĂM 2008 – 2009 – Toán Việt (toanviet.net)

ĐỀ THI CHỌN ĐỘI TUYỂN HSG QUỐC GIA CỦA TRƯỜNG PTNK NĂM 2009 – 2010 – Toán Việt (toanviet.net)

ĐỀ THI CHỌN ĐỘI TUYỂN HSG QUỐC GIA CỦA TRƯỜNG PTNK 2010 – 2011 – Toán Việt (toanviet.net)

ĐỀ THI CHỌN ĐỘI TUYỂN HSG QUỐC GIA CỦA TRƯỜNG PTNK NĂM 2011 – 2012 – Toán Việt (toanviet.net)

ĐỀ THI CHỌN ĐỘI TUYỂN QUỐC GIA CỦA TRƯỜNG PTNK NĂM 2013 – 2014 – Toán Việt (toanviet.net)

ĐỀ THI CHỌN ĐỘI TUYỂN HSG QUỐC GIA CỦA TRƯỜNG PTNK NĂM 2014 – 2015 – Toán Việt (toanviet.net)

Đáp án thi chọn đội tuyển Toán trường PTNK năm 2015 – Toán Việt (toanviet.net)

Đáp án đề thi chọn đội tuyển trường Phổ thông Năng khiếu thi HSG QG năm 2016 – Toán Việt (toanviet.net)

ĐỀ THI CHỌN ĐỘI TUYỂN HSG QUỐC GIA CỦA TRƯỜNG PTNK NĂM 2017 – 2018 – Toán Việt (toanviet.net)

ĐỀ THI CHỌN ĐỘI TUYỂN HSG QUỐC GIA CỦA TRƯỜNG PTNK NĂM 2018 – 2019 – Toán Việt (toanviet.net)

ĐỀ THI CHỌN ĐỘI TUYỂN HSG QUỐC GIA CỦA TRƯỜNG PTNK NĂM 2019 – 2020 – Toán Việt (toanviet.net)

Đáp án đề thi chọn đội tuyển trường PTNK năm 2020 – Toán Việt (toanviet.net)

Đề thi và đáp án chọn đội tuyển toán trường PTNK năm 2021 – Toán Việt (toanviet.net)

ĐỀ THI CHỌN ĐỘI TUYỂN CÁC TỈNH THÀNH KHÁC

Đề thi học sinh giỏi khối 10

Kì thi chọn đội dự tuyển trường Phổ thông Năng khiếu

Đề thi và đáp án chọn đội dự tuyển 10 trường PTNK năm 2023

Đề thi và đáp án chọn đội dự tuyển trường PTNK năm 2022

Đề thi và đáp án chọn đội dự tuyển PTNK năm 2021 – Toán Việt (toanviet.net)

Đề thi và đáp án chọn đội dự tuyển trường PTNK năm 2020 – Toán Việt (toanviet.net)

Đề thi và đáp án chọn đội dự tuyển PTNK năm học 2019 – 2020 – Toán Việt (toanviet.net)

Đề và đáp thi chọn đội dự tuyển PTNK năm học 2017 – 2018 – Toán Việt (toanviet.net)

Đáp án đề thi chọn đội dự tuyển lớp 10 năm 2016 – 2017 – Toán Việt (toanviet.net)

Đề và đáp án thi chọn đội dự tuyển lớp 10 năm 2012 – 2013 – Toán Việt (toanviet.net)

Kì thi Olympic truyền thống 30/4 (SGD TPHCM)

ĐỀ THI OLYMPIC 30 THÁNG 4 – TOÁN LỚP 10 NĂM 2011 – Toán Việt (toanviet.net)

ĐỀ THI OLYMPIC 30 THÁNG 4 – TOÁN LỚP 10 NĂM 2010 – Toán Việt (toanviet.net)

ĐỀ THI OLYMPIC 30 THÁNG 4 – TOÁN LỚP 10 NĂM 2009 – Toán Việt (toanviet.net)

ĐỀ THI OLYMPIC 30 THÁNG 4 – TOÁN LỚP 10 NĂM 2008 – Toán Việt (toanviet.net)

ĐỀ THI OLYMPIC 30 THÁNG 4 – TOÁN LỚP 10 NĂM 2007 – Toán Việt (toanviet.net)

ĐỀ THI OLYMPIC 30 THÁNG 4 – TOÁN LỚP 10 NĂM 2005 – Toán Việt (toanviet.net)

ĐỀ THI OLYMPIC 30 THÁNG 4 – TOÁN LỚP 10 NĂM 2004 – Toán Việt (toanviet.net)

ĐỀ THI OLYMPIC 30 THÁNG 4 – TOÁN LỚP 10 NĂM 2003 – Toán Việt (toanviet.net)

ĐỀ THI OLYMPIC 30 THÁNG 4 – TOÁN LỚP 10 NĂM 2002 – Toán Việt (toanviet.net)

ĐỀ THI OLYMPIC 30 THÁNG 4 – TOÁN LỚP 10 NĂM 2000 – Toán Việt (toanviet.net)

ĐỀ THI OLYMPIC 30 THÁNG 4 – TOÁN LỚP 10 NĂM 1999 – Toán Việt (toanviet.net)

ĐỀ THI OLYMPIC 30 THÁNG 4 – TOÁN LỚP 10 NĂM 1998 – Toán Việt (toanviet.net)

Kì thi duyên hải Bắc bộ

Kì thi HSG lớp 10 của các tỉnh, thành phố

ĐỀ THI OLYMPIC 30 THÁNG 4 – TOÁN LỚP 10 NĂM 2002

ĐỀ THI

Câu 1

Giải hệ phương trình: $\left\{\begin{array}{l}x_1=\frac{1}{2}\left(x_2+\frac{1}{x_2}\right) \\ x_2=\frac{1}{2}\left(x_3+\frac{1}{x_3}\right) \\ \cdots \\ x_{2002}=\frac{1}{2}\left(x_1+\frac{1}{x_1}\right)\end{array}\right.$

Câu 2

Chứng minh rằng: Phần nguyên của $(\sqrt{11}+3)^{3 \mathrm{n}+1}$ thì chia hết cho $2^{\mathrm{n}+1}$ và không chia hết cho $2^{\mathrm{n}+2}$ với mọi $\mathrm{n}$ là số tự nhiên.

Câu 3

Cho tam giác $\mathrm{ABC}$ thỏa:

$\quad\quad\quad\quad\quad\frac{\sin ^2 A+\sin ^2 B+\sin ^2 C}{\cot g A+\cot g B+\cot g C}=\sqrt{\frac{\sin ^2 A \cdot \sin ^2 B \cdot \sin ^2 C}{\operatorname{tg} \frac{A}{2} \cdot \operatorname{tg} \frac{B}{2} \cdot \operatorname{tg} \frac{C}{2}}}$

Chứng minh rằng: Tam giác $\mathrm{ABC}$ đều.

Câu 4

Giả sử điểm $\mathrm{M}$ nằm trong tam giác $\mathrm{ABC}$ sao cho: $\widehat{\mathrm{AMC}}=90^{\circ}$; $\widehat{\mathrm{AMB}}=150^{\circ} ; \widehat{\mathrm{BMC}}=120^{\circ}$. Gọi các điểm $\mathrm{P}, \mathrm{Q}, \mathrm{R}$ lần lượt là tâm các đường tròn ngoại tiếp của tam giác $\mathrm{AMC}, \mathrm{AMB}, \mathrm{BMC}$.

Chứng minh rằng:

 

LỜI GIẢI

Câu 1

Giải hệ phương trình: $\left\{\begin{array}{l}x_1=\frac{1}{2}\left(x_2+\frac{1}{x_2}\right) \\ x_2=\frac{1}{2}\left(x_3+\frac{1}{x_3}\right) \\ \cdots \\ x_{2002}=\frac{1}{2}\left(x_1+\frac{1}{x_1}\right)\end{array}\right.$

Lời Giải

  • Nhận xét: Nếu $\left(x_1, x_2, \ldots, x_{2002}\right)$ là nghiệm thì $x_1, x_2, \ldots, x_{2002}$ phải cùng dấu và khác 0

Đồng thời $\left(-x_1,-x_2, \ldots,-x_{2002}\right)$ cũng là nghiệm, nên ta chỉ cần xét với $x_1, x_2, \ldots, x_{2002}$ dương.

  • Theo bất đẳng thức Côsi: $x_i+\frac{1}{x_i} \geq 2(I=1,2, \ldots, 2002)\quad\quad\quad (1)$

Từ các phương trình trong hệ và (1) ta được: $2 x_i \geq 2$ hay $x_i \geq 1\quad\quad\quad (2)$

  • Mặt khác cộng các phương trình trong hệ thì:

$x_1+x_2+\ldots+x_{2002}=\frac{1}{x_1}+\frac{1}{x_2}+\ldots+\frac{1}{x_{2002}}\quad\quad\quad(3)$

Từ (2) và (3) được: $x_1=x_2=\ldots=x_{2002}=1$

  • Vậy hệ có 2 nghiệm: $\left[\begin{array}{l}x_1=x_2=\ldots=x_{2002}=1 \\ x_1=x_2=\ldots=x_{2002}=-1\end{array}\right.$

Câu 2

Chứng minh rằng: Phần nguyên của $(\sqrt{11}+3)^{3 \mathrm{n}+1}$ thì chia hết cho $2^{\mathrm{n}+1}$ và không chia hết cho $2^{\mathrm{n}+2}$ với mọi $\mathrm{n}$ là số tự nhiên.

Lời Giải

Trước hết, nhận xét rằng: $(\sqrt{11}+3)^{2 \mathrm{n}+1}-(\sqrt{11}-3)^{2 \mathrm{n}+1}$ là một số tự nhiên. Thật vậy, ta có:

$(\sqrt{11}+3)^{2 n+1}=C_{2 n+1}^0(\sqrt{11})^{2 n+1}+C_{2 n+1}^1(\sqrt{11})^{2 n} \cdot 3+C_{2 n+1}^2(\sqrt{11})^{2 n-1} 3^2$

$\quad\quad\quad\quad\quad\quad\quad\quad\quad\quad\quad\quad\quad\quad\quad +\ldots+C_{2 n+1}^{2 n}(\sqrt{11})^1 3^{2 n}+C_{2 n+1}^{2 n+1} 3^{2 n+1}$

$(\sqrt{11}-3)^{2 n+1}=C_{2 n+1}^0(\sqrt{11})^{2 n+1}-C_{2 n+1}^1(\sqrt{11})^{2 n} \cdot 3+C_{2 n+1}^2(\sqrt{11})^{2 n-1} 3^2$

$\quad\quad\quad\quad\quad\quad\quad\quad\quad\quad\quad\quad\quad\quad\quad -\ldots+C_{2 n+1}^{2 n}(\sqrt{11})^1 3^{2 n}-C_{2 n+1}^{2 n+1} 3^{2 n+1}$

$\Rightarrow(\sqrt{11}+3)^{2 \mathrm{n}+1}-(\sqrt{11}-3)^{2 \mathrm{n}+1}=$

$\quad\quad\quad\quad\quad\quad\quad\quad\quad\quad =2\left[C_{2 n+1}^1(\sqrt{11})^{2 n} \cdot 3+C_{n+1}^3(\sqrt{11})^{2 n-2} \cdot 3^2+\ldots+C_{2 n+1}^{2 n+1} \cdot 3^{2 n+1}\right]$

Suy ra $(\sqrt{11}+3)^{2 \mathrm{n}+1}-(\sqrt{11}-3)^{2 \mathrm{n}+1}$ là số tự nhiên.

$\operatorname{Mà}(\sqrt{11}-3)^{2 \mathrm{n}+1} \in(0 ; 1)$ nên

$\left[(\sqrt{11}+3)^{2 \mathrm{n}+1}\right]=(\sqrt{11}+3)^{2 \mathrm{n}+1}-(\sqrt{11}-3)^{2 \mathrm{n}+1}$

(Vì: $\mathrm{a}-\mathrm{b}=\mathrm{k} \in \mathrm{N} \Rightarrow \mathrm{a}=\mathrm{k}+\mathrm{b}$ với $\mathrm{b} \in(0 ; 1)$ nên $[\mathrm{a}]=\mathrm{k}^{\prime}=\mathrm{a}-\mathrm{b}$, kí hiệu $[$.$] là$ phần nguyên của số thực)

  • Với n $=0:(\sqrt{11}+3)^1-(\sqrt{11}-3)^1=6$ chia hết cho $2^{0+1}=2$ nhưng không chia hết cho $2^2=4$

  • Lại có: $(\sqrt{11}+3)^2-(\sqrt{11}-3)^2=40 \Rightarrow$ với $\mathrm{n}=1$ thì

$(\sqrt{11}+3)^3-(\sqrt{11}-3)^3=\left(\frac{(\sqrt{11}+3)-(\sqrt{11}-3)}{6}\right)$

$\quad\quad\quad\quad\quad\quad\quad\quad\quad\quad\quad\quad\left[\frac{(\sqrt{11}+3)^2+(\sqrt{11}-3)^2}{40}+\frac{(\sqrt{11}+3)(\sqrt{11}-3)}{2}\right]$

$=6.42=2^2 \cdot 3^2 \cdot 7$

chia hết cho $2^2$ nhưng không chia hết cho $2^3$.

Giả sử tính chất này đúng với mọi số tự nhiên $\mathrm{k}<\mathrm{n}$. Ta chứng minh tính chất này đúng với $\mathrm{k}=\mathrm{n}$.

Trước hết nhận xét rằng:

$(\sqrt{11}+3)(\sqrt{11}-3)=2 \Rightarrow \sqrt{11}-3=\frac{2}{\sqrt{11}+3}$

$\sqrt{11}+3=\frac{2}{\sqrt{11}-3}$

Thật vậy:

$\quad\quad (\sqrt{11}+3)^{2 \mathrm{n}+1}-(\sqrt{11}-3)^{2 \mathrm{x}+1}$

$=[(\sqrt{11}\left.+3)^2+(\sqrt{11}-3)^2\right]\left[(\sqrt{11}+3)^{2 n+1}-(\sqrt{11}-3)^{2 n-1}\right] $

$\quad\quad\quad\left.-\left[(\sqrt{11}-3)^2(\sqrt{11}+3)^{2 n-1}\right]-(\sqrt{11}+3)^2(\sqrt{11}-3)^{2 n-1}\right]$

$=40\left[(\sqrt{11}+3)^{2 \mathrm{n}-1}-(\sqrt{11}-3)^{2 \mathrm{n}-1}\right]-4\left[(\sqrt{11}+3)^{2 \mathrm{n}-3}-(\sqrt{11}-3)^{2 \mathrm{n}-3}\right]$

$=\underbrace{2^3 5 \cdot\left[(\sqrt{11}+3)^{2 \mathrm{n}-1}-(\sqrt{11}-3)^{2 \mathrm{n}-1}\right]}_{\text {chia hết cho } 2^{\mathrm{n}}}-$

$\quad\quad\quad\quad\quad\quad\quad\quad\quad\quad\quad\quad\quad\quad\quad -\underbrace{2^2 \cdot\left[(\sqrt{11}+3)^{2 \mathrm{n}-3}-(\sqrt{11}-3)^{2 \mathrm{n}-3}\right]}_{\text {chia hết cho } 2^{\mathrm{u}-1} \text { nhưng không chia hết cho } 2^{\mathrm{n}}}$

Vậy $\left[(\sqrt{11}+3)^{2 n+1}\right]$ chia hết cho $2^{n+1}$ nhưng không chia hết cho $2^{n+2}$.

Câu 3

Cho tam giác $\mathrm{ABC}$ thỏa:

$\quad\quad\quad\quad\quad\frac{\sin ^2 A+\sin ^2 B+\sin ^2 C}{\cot g A+\cot g B+\cot g C}=\sqrt{\frac{\sin ^2 A \cdot \sin ^2 B \cdot \sin ^2 C}{\operatorname{tg} \frac{A}{2} \cdot \operatorname{tg} \frac{B}{2} \cdot \operatorname{tg} \frac{C}{2}}}$

Chứng minh rằng: Tam giác $\mathrm{ABC}$ đều.

Lời Giải

Ta có: $\mathrm{a}^2=\mathrm{b}^2+\mathrm{c}^2-2 \mathrm{bc} \cdot \cos \mathrm{A}$

$\quad\quad\quad\quad=\mathrm{b}^2+\mathrm{c}^2-(2 \mathrm{bcsin} \mathrm{A}) \cdot \operatorname{cotg} \mathrm{A}=\mathrm{b}^2+\mathrm{c}^2-4 \mathrm{~S} \cdot \operatorname{cotg} \mathrm{A} \text {. }$

(S là diện tích $\triangle \mathrm{ABC}$ )

Tương tự: $\mathrm{b}^2=\mathrm{a}^2+\mathrm{c}^2-4 \operatorname{ScotgB} ; \mathrm{c}^2=\mathrm{a}^2+\mathrm{b}^2-4 \mathrm{~S} \operatorname{cotg} \mathrm{C}$

Suy ra: $a^2+b^2+c^2=4 S(\operatorname{cotg} A+\operatorname{cotg} B+\operatorname{cotg} C)$

$\quad\quad\quad\Leftrightarrow \sin ^2 \mathrm{~A}+\sin ^2 \mathrm{~B}+\sin ^2 \mathrm{C}=\frac{\mathrm{S}}{\mathrm{R}^2}(\operatorname{cotg} \mathrm{A}+\operatorname{cotg} \mathrm{B}+\operatorname{cotg} \mathrm{C})$

$\quad\quad\quad\Leftrightarrow \frac{\sin ^2 \mathrm{~A}+\sin ^2 \mathrm{~B}+\sin ^2 \mathrm{C}}{\cot g \mathrm{~A}+\cot g \mathrm{~B}+\cot g \mathrm{C}}=\frac{\mathrm{S}}{\mathrm{R}^2}\quad(1)$

  • Mặt khác:

$\quad\quad\quad\quad a^2=b^2+c^2-2 b c \cdot \cos A \geq 2 b c-2 b c \cdot \cos A=4 b c \cdot \sin ^2 \frac{A}{2}=4 S \cdot \operatorname{tg} \frac{A}{2} $

$\quad\quad\quad\Rightarrow \frac{\sin ^2 A}{\operatorname{tg} \frac{A}{2}} \geq \frac{S}{R^2}$

Tương tự: $\frac{\sin ^2 \mathrm{~B}}{\operatorname{tg} \frac{\mathrm{B}}{2}} \geq \frac{\mathrm{S}}{\mathrm{R}^2} ; \frac{\sin ^2 \mathrm{C}}{\operatorname{tg} \frac{\mathrm{C}}{2}} \geq \frac{\mathrm{S}}{\mathrm{R}^2}$

Từ đó: $\sqrt[3]{\frac{\sin ^2 \mathrm{~A} \cdot \sin ^2 \mathrm{~B} \cdot \sin ^2 \mathrm{C}}{\operatorname{tg} \frac{\mathrm{A}}{2} \cdot \operatorname{tg} \frac{\mathrm{B}}{\mathrm{C}} \cdot \operatorname{tg} \frac{\mathrm{C}}{2}}} \geq \frac{\mathrm{S}}{\mathrm{R}^2}\quad(2)$

Tữ (1) và (2) suy ra: $\sqrt[3]{\frac{\sin ^2 A \cdot \sin ^2 B \cdot \sin ^2 C}{\operatorname{tg} \frac{A}{2} \cdot \operatorname{tg} \frac{B}{C} \cdot \operatorname{tg} \frac{C}{2}}} \geq \frac{\sin ^2 A+\sin ^2 B+\sin ^2 C}{\cot g A+\cot g B+\cot g C}$

Dấu “=” xảy ra khi $\mathrm{a}=\mathrm{b}=\mathrm{c} \Leftrightarrow \triangle \mathrm{ABC}$ đều. Từ đó suy ra điều phải chứng minh.

Câu 4

Giả sử điểm $\mathrm{M}$ nằm trong tam giác $\mathrm{ABC}$ sao cho: $\widehat{\mathrm{AMC}}=90^{\circ}$; $\widehat{\mathrm{AMB}}=150^{\circ} ; \widehat{\mathrm{BMC}}=120^{\circ}$. Gọi các điểm $\mathrm{P}, \mathrm{Q}, \mathrm{R}$ lần lượt là tâm các đường tròn ngoại tiếp của tam giác $\mathrm{AMC}, \mathrm{AMB}, \mathrm{BMC}$.

Chứng minh rằng:

Lời Giải

$A, M$ đối xứng nhau qua $P Q$.

$\mathrm{B}, \mathrm{M}$ đối xứng nhau qua $\mathrm{QR}$.

$\mathrm{C}, \mathrm{M}$ đối xứng nhau qua $\mathrm{RP}$.

$\quad\quad\quad\quad\quad\quad\quad\Rightarrow\left\{\begin{array}{l}\mathrm{S}(\mathrm{MPQ})=\mathrm{S}(\mathrm{APQ}) \\ \mathrm{S}(\mathrm{MQR})=\mathrm{S}(\mathrm{BQR}) \\ \mathrm{S}(\mathrm{MPR})=\mathrm{S}(\mathrm{CPR})\end{array}\right.$

$\quad\quad\quad\quad\quad\quad\quad\Rightarrow 2 \mathrm{~S}(\mathrm{PQR})=\mathrm{S}(\mathrm{AQBRC})$

$\quad\quad\quad\quad\quad\quad\quad\Rightarrow \mathrm{S}(\mathrm{PQR})=\frac{1}{2} \mathrm{~S}(\mathrm{AQBRC})$

Do sự đối xứng trên ta có

$\quad\quad\quad\quad\quad\widehat{\mathrm{AQB}}=2 \widehat{\mathrm{PQR}}=2\left(180^{\circ}-\widehat{\mathrm{AMB}}\right)=60^{\circ}$

$\quad\quad\quad\quad\quad\widehat{\mathrm{BRC}}=2 \widehat{\mathrm{QRP}}=120^{\circ}$

$\mathrm{S}(\mathrm{PQR})=\frac{1}{2} \mathrm{~S}(\mathrm{AQBRC})=\frac{1}{2}[\mathrm{~S}(\mathrm{ABC})+\mathrm{S}(\mathrm{AQB})+\mathrm{S}(\mathrm{BRC})]$

$\quad\quad\quad\quad =\frac{1}{2}\left[\mathrm{~S}(\mathrm{ABC})+\frac{\sqrt{3}}{4} \mathrm{AB}^2+\frac{\mathrm{CB}^2}{4 \sqrt{3}}\right]$

$\quad\quad\quad\quad\geq \frac{1}{2} S(\mathrm{ABC})+\frac{1}{4} \mathrm{AB} \cdot \mathrm{CB} \geq \frac{1}{2} \mathrm{~S}(\mathrm{ABC})$

$\quad\quad\quad\quad\quad\quad\quad +\frac{1}{2} \cdot \frac{1}{2} \mathrm{AB} \cdot \mathrm{CB} \cdot \sin \widehat{\mathrm{ABC}}$

(Do $\mathrm{M}$ nằm trong $\triangle \mathrm{ABC}$ và $\left.\widehat{\mathrm{AMC}}=90^{\circ} \Rightarrow \widehat{\mathrm{ABC}}<90^{\circ}\right)$

$\quad\quad\quad\quad S_{\triangle P Q R}>\frac{1}{2} S(A B C)+\frac{1}{2} S(A B C)=S(A B C)$

 

 

 

 

 

 

 

 

 

 

 

 

 

 

 

 

 

 

 

Đề thi và đáp án kì thi chọn đội tuyển thi Quốc gia trường Phổ thông Năng khiếu năm học 2018 – 2019

ĐỀ THI

Ngày thi thứ nhất

Bài 1. Cho số nguyên $a>1$. Tìm giá trị lớn nhất của số thực $d$ sao cho tồn tại một cấp số cộng có công sai $d$, số hạng đầu tiên là $a$ và có đúng hai trong các số $a^2, a^3, a^4, a^5$ là những số hạng của cấp số cộng đó.

Bài 2. Cho $n$ số thực $x_1, x_2, \ldots, x_n$. Với mỗi $i \in{1,2, \ldots, n}$, gọi $a_i$ là số các chỉ số $j$ mà $\left|x_i-x_j\right| \leq 1$ và $b_i$ là số các chỉ số $j$ mà $\left|x_i-x_j\right| \leq 2$ ( $i$ và $j$ có thể bằng).

(a) Chứng minh rằng tồn tại $i$ để $b_i \leq 3 a_i$.

(b) Gọi $A$ là số cặp $(i, j)$ có thứ tự mà $\left|x_i-x_j\right| \leq 1$ và $B$ là số cặp $(i, j)$ có thứ tự mà $\left|x_i-x_j\right| \leq 2$ ( $i$ và $j$ có thể bằng nhau). Chứng minh rằng $B \leq 3 A$.

Bài 3. Cho $p$ là số tự nhiên. Xét phương trình nghiệm nguyên $x^3+x+p=y^2$.

(a) Tìm số nguyên tố $p$ nhỏ nhất dạng $4 k+1$ sao cho phương trình có nghiệm.

(b) Chứng minh rằng nếu $p$ là số chính phương thì phương trình trên có nghiệm nguyên dương.

Bài 4. Cho tam giác $A B C$ nhọn nội tiếp đường tròn $(O)$ với $B, C$ cố định và $A$ di động trên $(O)$. $D$ là trung điểm $B C$. Trên $A B$ lấy các điểm $M, P$ và trên $A C$ lấy các điểm $N, Q$ sao cho $D A=D P=D Q$, dồng thời $D M \perp A C, D N \perp A B$.

(a) Chứng minh rằng các điểm $M, N, P, Q$ cùng thuộc một đường tròn $(\mathcal{C})$ và $(\mathcal{C})$ luôn đi qua một điểm cố định.

(b) Chứng minh rằng tâm của $(\mathcal{C})$ luôn thuộc một đường tròn cố định.

 

Ngày thi thứ hai

Bài 5. Cho số thực $a \neq 0$. Tìm giới hạn của dãy số $\left(u_n\right)$ thoả mãn:

$\quad\quad\quad\quad\quad\quad\quad\quad u_1=0, u_{n+1}\left(u_n+a\right)=a+1, \forall n \in \mathbb{N}^*$

Bài 6. Tìm tất cả các hàm số $f: \mathbb{R}^{+} \rightarrow \mathbb{R}^{+}$thoả mãn điều kiện:

$\quad\quad\quad\quad f\left(x f\left(y^2\right)-y f\left(x^2\right)\right)=(y-x) f(x y) \forall x, y \in \mathbb{R}^{+}, x>y$

Bài 7. Cho $n=2018.2019$. Gọi $A$ là tập hợp các bộ $\left(a_1, a_2, \ldots, a_n\right)$ có thứ tự thoả mãn điều kiện $a_i \in{0,1} \forall i \in{1,2, \ldots, n}$ và $\sum_{i=1}^n a_i=2018^2$. Có bao nhiêu bộ $\left(a_1, a_2, \ldots, a_n\right)$ từ $A$ để $\sum_{i=1}^k a_i \geq \frac{a}{2}$ và $\sum_{i=n-k+1}^n a_i \geq \frac{k}{2} \forall k \in{1,2, \ldots, n}$ ?

Bài 8. Đường tròn $(\mathcal{C})$ tâm $I$ nội tiếp tam giác $A B C$ và tiếp xúc với các cạnh $A B, A C$ tại $E, F$. $A M, A N$ là các đường phân giác trong, phân giác ngoài của góc $\angle B A C(M, N$ nằm trên $B C)$. Gọi $d_M, d_N$ lần lượt là các tiếp tuyến của $(\mathcal{C})$ qua $M, N$ và khác $B C$.

(a) Chứng minh rằng $d_M, d_N, E F$ đồng quy tại điểm $D$.

(b) Lấy trên $A B, A C$ các điểm $P, Q$ thoả mãn $D P|A C, D Q| A B$. Gọi $R, S$ là trung điểm của $D E, D F$. Chứng minh rằng $I$ thuộc đường thẳng qua các trực tâm của hai tam giác $D P S, D Q R$.

 

LỜI GIẢI

Ngày thi thứ nhất

Bài 1. Cho số nguyên $a>1$. Tìm giá trị lớn nhất của số thực $d$ sao cho tồn tại một cấp số cộng có công sai $d$, số hạng đầu tiên là $a$ và có đúng hai trong các số $a^2, a^3, a^4, a^5$ là những số hạng của cấp số cônng đó.

Lời giải: Trước hết, ta chứng minh rằng $d=a^3-a$ thoả mãn điều kiện. Thật vậy, xét cấp số cộng có số hạng đầu là $a$ và công sai là $d=a^3-a$ thì

$\quad\quad\quad\quad\quad\quad\quad\quad\quad\quad \left\{\begin{array}{l}a^3=a+\left(a^3-a\right) \\ a^5=a+\left(a^3-a\right)\left(a^2+1\right)\end{array} .\right.$

Do đó $a^3, a^5$ cùng thuộc cấp số cộng có công sai $d=a^3-a$.

Giả sử rằng tồn tại giá trị $d>a^3-a$ thoả mãn điều kiện bài toán. Khi đó:

$\quad\quad\quad\quad\quad\quad\quad\quad\quad\quad a+d>a+a^3-a=a^3$

Dẫn đến hai số hạng thuộc cấp số cộng phải là $a^4$ và $a^5$. Lại để ý rằng $a>1$ nên có $a<a^4<a^5$, kết hợp lại thì phải tồn tại hai số nguyên dương $k<l$ sao cho:

$\quad\quad\quad\quad\quad\quad\quad\quad\quad\quad \left\{\begin{array}{l}a^4=a+k d \\ a^5=a+l d\end{array}\right.$

Từ đó $a(a+k d)=a+l d$ hay $d(l-a k)=a^2-a>0$.

Chú ý rằng ta có $d>0$ nên $l-a k>0$, hơn nữa $l-a k \in \mathbb{Z}$ nên $l-a k \geq 1$. Điều này dẫn đến $a^2-a \geq d>a^3-a$, vô lý do $a>1$.

Vậy giá trị lớn nhất của $d$ là $\max d=a^3-a$.

Bài 2. Cho $n$ số thực $x_1, x_2, \ldots, x_n$. Với mỗi $i \in{1,2, \ldots, n}$, gọi $a_i$ là số các chỉ số $j$ mà $\left|x_i-x_j\right| \leq 1$ và $b_i$ là số các chỉ số $j$ mà $\left|x_i-x_j\right| \leq 2(i$ và $j$ có thể bằng nhau).

(a) Chứng minh rằng tồn tại $i$ dể $b_i \leq 3 a_i$.

(b) Gọi $A$ là số cặp $(i, j)$ có thứ tự mà $\left|x_i-x_j\right| \leq 1$ và $B$ là số cặp $(i, j)$ có thứ tự mà $\left|x_i-x_j\right| \leq 2$ ( $i$ và $j$ có thể bằng nhau). Chứng minh rằng $B \leq 3 A$.

Lời giải . (a) Không mất tính tổng quát, giả sử $x_1 \leq x_2 \leq \ldots \leq x_n$.

Xét $k=\max [a_1, a_2, \ldots, a_n]$ và $a_i=k$, khi đó tồn tại $k$ số trong dãy là:

$\quad\quad\quad\quad x_u \leq x_{u+1} \leq \ldots \leq x_i \leq \ldots \leq x_v \text { với }\left|x_u-x_i\right|,\left|x_v-x_i\right| \leq 1 .$

Ngoài ra vì tính lớn nhất của $k$ nên $\left|x_{u-1}-x_i\right|>1,\left|x_{v+1}-x_i\right|>1$.

Trong $\left[x_u, x_v\right]$, có đúng $k$ chỉ số $j$ để $\left|x_j-x_i\right| \leq 1<2$. Còn trước $x_u$, xét hai số $x_r, x_s$ sao cho $x_r \leq x_s$ và $\left|x_r-x_i\right| \leq 2,\left|x_s-x_i\right| \leq 2$ thì:

$\quad\quad\quad\quad \left|x_r-x_s\right|=x_s-x_r=\left(x_i-x_r\right)-\left(x_i-x_s\right)<2-1=1$

nên sẽ có không quá $k$ số $j$ để $\left|x_j-x_i\right| \leq 2$ vì nếu ngược lại, sẽ có nhiều hơn $k$ số liên tiếp trong dãy cách nhau không quá 1 đơn vị, mâu thuẫn với tính lớn nhất của $k$. Tương tự với các số sau $x_v$, vì thế nên $b_i \leq 3 k$ kéo theo $b_i \leq 3 a_i$.

(b) Ta sẽ chứng minh bằng quy nạp theo $n$.

Với $n=1$ rõ ràng $A=B=1$ nên khẳng định hiển nhiên đúng. Giả sử kết quả đúng với $n \geq 1$, ta sẽ chứng minh nó cũng đúng với $n+1$.

Xét dãy số thực $T=\left(x_1, x_2, \ldots, x_{n+1}\right)$ bất kỳ và giả sử $x_1 \leq x_2 \leq \ldots \leq x_{n+1}$. Ký hiệu $A_T, B_T$ là số cặp có thứ tự các chỉ số $(i, j)$ tương ứng với định nghĩa của đề bài. Giả sử $k \geq 1$ là số lượng lớn nhất các số của $T$ được chứa trong một đoạn độ dài bằng 2 nào đó.

Gọi $x_i$ là số cuối cùng của dãy mà trong đoạn $\left[x_i-1, x_i+1\right]$ có chứa đúng $k$ số (kể cả $x_i$ ). Gọi $T^{\prime}$ là dãy mới sau khi bỏ $x_i$ đi. Khi đó, số lượng các số thuộc $T^{\prime}$ có trong $\left[x_i-1, x_i+1\right]$ là $k-1$, ngoài ra $x_i$ đã bị bỏ đi thuộc về đúng $2 k-1$ cặp của $A_T$.

Do đó: $A_T=A_{T^{\prime}}+2 k-1$.

Ta viết lại như sau

$\quad\quad\quad\quad \left[x_i-2 ; x_i+2\right]=\left[x_i-2 ; x_i-1\right] \cup\left[x_i-1 ; x_i+1\right] \cup\left[x_i+1 ; x_i+2\right]$

Trừ đoạn ở giữa thì hai đoạn đầu và cuối chứa tối đa $k$ phần tử của $T$. Hơn nữa, do định nghĩa số $x_i$ nên trong đoạn $\left[x_i+1 ; x_i+2\right]$ có tối đa $k-1$ phần tử của $T$. Từ đó có tối đa:

$\quad\quad\quad\quad\quad\quad\quad\quad\quad\quad\quad\quad 2(k-1)+k=3 k-2$

phần tử của $T$ (không tính $x_i$ ) thuộc $\left[x_i-2 ; x_i+2\right]$. Dẫn đến:

$\quad\quad\quad\quad\quad\quad B_T \leq 2(3 k-2)+1+B_{T^{\prime}}=3(2 k-1)+B_{T^{\prime}}$

Áp dụng giả thiết quy nạp, ta có $B_{T^{\prime}}<3 A_{T^{\prime}}$ nên từ các điều trên thì:

$\quad\quad\quad\quad B_T \leq 3(2 k-1)+B_{T^{\prime}}<3(2 k-1)+3 A_{T^{\prime}}=3\left(A_{T^{\prime}}+2 k-1\right)=3 A_T .$

Theo nguyên lý quy nạp, bài toán cũng đúng với $n+1$.

Vậy bài toán được chứng minh hoàn toàn.

Nhận xét. Bài toán này thật ra liên quan đến phương pháp xác suất trong tổ hợp, có thể xem tại quyển “The Probabilistic Method” của GS. Noga Alon. Ta xét một lời giải khác như sau:

(a) Chọn $i$ sao cho số các chỉ số $j$ để $\left|x_i-x_j\right| \leq 1$ là lớn nhất. Khi đó, số lượng chỉ số $j$ sao cho $x_j \in\left(x_i+1, x_i+2\right]$ tối đa là $a_i$, vì nếu không thì tồn tại $j$ để $a_j>a_i$. Tương tự, số lượng chỉ số $j$ sao cho $x_j \in\left[x_i-2, x_i-1\right)$ tối đa là $a_i$.

Chú ý rằng với các chỉ số $j$ để $\left|x_i-x_j\right| \leq 2$ thì ta có điều sau:

$\quad\quad\quad\quad x_j \in\left[x_i-2, x_i-1\right) \cup\left(x_i-1, x_i+1\right) \cup\left(x_i+1, x_i+2\right]$

Số lượng các chỉ số đó chính là $b_i$, dẫn đến $b_i \leq a_i+a_i+a_i=3 a_i$. Hơn nữa, nếu đẳng thức xảy ra, ta phải có mỗi đoạn (hay nửa khoảng) ở phân hoạch trên chứa chính xác $a_i$ chỉ số $j$ của $x_j$.

(b) Bài toán hiển nhiên đúng với $n=1$. Giả sử rằng tồn tại $n>1$ để kết luận không đúng, ta chọn $n$ nhỏ nhất. Ta cũng chọn $i$ sao cho $a_i$ lớn nhất.

Gọi $A^{\prime}, B^{\prime}$ tương ứng là số cặp chỉ số $(k, l)$ mà $\left|x_k-x_l\right| \leq 1$ và $\left|x_k-x_l\right| \leq 2$, trong đó $1 \leq k, l \leq n$ và $k, l \neq i$. Vì $n$ là phản ví dụ nhỏ nhất nên $B^{\prime} \leq 3 A^{\prime}$.

Các cặp chỉ số $(k, l)$ mà $k=i$ hoặc $l=i$ và $\left|x_k-x_l\right| \leq 1$ đều phải có dạng $(k, i)$ hoặc $(i, k)$ trong đó $k \neq i$ và $(i, i)$. Có tổng cộng $2\left(a_i-1\right)+1$ cặp như thế nên $A=A^{\prime}+2\left(a_i-1\right)+1$.

Tương tự thì $B=B^{\prime}+2\left(b_i-1\right)+1$. Do đó nếu $b_i \leq 3 a_i-1$ thì:

$\quad\quad\quad\quad B=B^{\prime}+2 b_i-1 \leq 3 A^{\prime}+2\left(3 a_i-1\right)-1=3\left(A^{\prime}+2 a_i-1\right)=3 A$

Điều này trái với việc $n$ là phản ví dụ nhỏ nhất. Do đó $b_i \geq 3 a_i$. Theo ý (a) thì $b_i \leq 3 a_i$, từ đây phải có $b_i=3 a_i$. Hơn nữa, số lượng chỉ số $j$ để thỏa mãn $x_j \in\left[x_i-2, x_i-1\right)$ hoặc $x_j \in\left(x_i+1, x_i+2\right]$ dều phải bằng $a_i$.

Với mỗi $j, j^{\prime}$ sao cho $x_j, x_{j^{\prime}} \in\left[x_i-2, x_i-1\right)$, ta có $\left|x_j-x_{j^{\prime}}\right|<1$, dẫn đến $a_j \geq a_i$. Mặt khác $a_i$ là lớn nhất có thể nên $a_j=a_i$. Tương tự, với mỗi $j$ sao cho $x_j \in\left(x_i+1, x_i+2\right]$ thì $a_j=a_i$. Như vậy với mọi $j$ sao cho $1<\left|x_i-x_j\right| \leq 2$ thì $a_j=a_i$. Cũng với cách chọn chỉ số $j$ đó, lập luận tương tự như những ý trên, ta cũng phải có $b_j=3 a_j$.

Xây dựng đồ thị $\mathcal{G}$ với các đỉnh được đánh số là $1,2, \ldots, n$ sao cho cặp đỉnh $(k, l)$ kề nhau khi và chỉ khi $1<\left|x_k-x_l\right| \leq 2$. Những lập luận trên cho thấy mọi đỉnh $j$ mà tồn tại một đường đi từ $i$ đến $j$ đều phải thỏa mãn $a_j=a_i$ và $b_j=3 a_j$. Gọi $\mathcal{X}$ là tập hợp tất cả các đỉnh $j$ sao cho tồn tại một đường đi từ $i$ dến $j$ trong $\mathcal{G}$. Đặt $\mathcal{Y}={1,2, \ldots, n} \backslash \mathcal{X}(\mathcal{Y}$ có thể rỗng $)$.

Bây giờ, gọi $A_y, B_y$ tương ứng là số cặp chỉ số $(k, l)$ có tính thứ tự, có thể bằng nhau mà $\left|x_k-x_l\right| \leq 1$ và $\left|x_k-x_l\right| \leq 2$, trong đó $k, l \in \mathcal{Y}$. Chú ý rằng $A_{\mathcal{Y}}=B_{\mathcal{Y}}=0$ nếu $\mathcal{Y}=\emptyset$. Bởi $n$ là phản ví dụ nhỏ nhất, ta phải có $B_{\mathcal{Y}} \leq 3 A_{\mathcal{Y}}$. Ta gọi $a_{y, k}$ và $b_{y, k}$ tương ứng là số chỉ số $j \in \mathcal{Y}$ mà $\left|x_j-x_k\right| \leq 1$ và $\left|x_j-x_k\right| \leq 2$. Định nghĩa tương tự $a_{\mathcal{X}, k}$ và $b_{\mathcal{X}, k}$.

Với mọi $k \in \mathcal{Y}$, dễ thấy $k$ không kề bất cứ đỉnh nào trong $\mathcal{X}$, vì vậy ta có được $b_{\mathcal{X}, k}=0$ và $b_k=b_{\mathcal{Y}, k}+a_{\mathcal{X}, k}$. Từ đây dẫn đến đẳng thức sau:

$\quad\quad\quad\quad\quad\quad B=\sum_{k \in \mathcal{X}} b_k+\sum_{k \in \mathcal{Y}} b_k=3 \sum_{k \in \mathcal{X}} a_k+\sum_{k \in \mathcal{Y}}\left(b_{y, k}+a_{\mathcal{X}, k}\right)$

Ta đồng thời có $\sum_{k \in \mathcal{Y}} b_{y, k}=B_{\mathcal{Y}} \leq 3 A_{\mathcal{Y}}$. Hơn nữa, ta cũng có được:

$\quad\quad A=\sum_{k \in \mathcal{X}} a_k+\sum_{k \in \mathcal{Y}} a_k=\sum_{k \in \mathcal{X}} a_k+\sum_{k \in \mathcal{Y}}\left(a_{\mathcal{Y}, k}+a_{\mathcal{X}, k}\right)=\sum_{k \in \mathbb{X}} a_k+A_{\mathcal{Y}}+\sum_{k \in \mathcal{Y}} a_{\mathcal{X}, k}$

Do đó:

$\quad\quad\quad\quad B \leq 3 A_{\mathcal{Y}}+\sum_{k \in \mathcal{Y}} a_{\mathcal{X}, k}+3 \sum_{k \in \mathcal{X}} a_k \leq 3\left(A_{\mathcal{Y}}+\sum_{k \in \mathcal{Y}} a_{\mathcal{X}, k}+\sum_{k \in \mathcal{X}} a_k\right)=3 A$

Điều này dẫn đến giả sử phản chứng là sai.

Vì vậy, với mọi số nguyên dương $n$, ta phải có $B \leq 3 A$. Bài toán kết thúc.

Bài 3. Cho $p$ là số tự nhiên. Xét phương trình nghiệm nguyên

$\quad\quad\quad\quad\quad\quad\quad\quad\quad\quad\quad\quad x^3+x+p=y^2 .$

(a) Tìm số nguyên tố $p$ nhỏ nhất dạng $4 k+1$ sao cho phương trình có nghiệm.

(b) Chứng minh rằng nếu $p$ là số chính phương thì phương trình trên có nghiệm nguyên dương.

Lời giải. (a) Các số nguyên tố có dạng $4 k+1$ là $5,13,17, \ldots$

Trước hết, ta thấy với $p=13$ thì $x^3+x+13=y^2$ có nghiệm là $(x ; y)=(4 ; 9)$. Ta sẽ chứng minh rằng phương trình $x^3+x+5=y^2$ không có nghiệm nguyên. Xét modulo 4. Có các khả năng sau xảy ra:

  • Khi $x$ chia 4 dư $0,1,2,3$, vế trái chia 4 lần lượt dư $1,3,3,3$.
  • Khi $y$ chia 4 dư $0,1,2,3$, vế phải chia 4 lần lượt dư $0,1,0,1$.

Do đó $y$ phải lẻ và $4 \mid x$. Viết biểu thức đã cho thành:

$\quad\quad\quad\quad\quad\quad\quad\quad\quad\quad (x+3)\left(x^2-3 x+10\right)=y^2+5^2$

Do $x+3 \equiv 3(\bmod 4)$ nên $x+3$ có ước nguyên tố $q \equiv 3(\bmod 4)$. Ta biết rằng với $a, b \in \mathbb{Z}$ thì $a^2+b^2$ chia hết cho số nguyên tố $q \equiv 3(\bmod 4)$ khi và chỉ khi $q \mid a$ và $q \mid b$. Từ đó thì $q \mid 5$ hay $q=5$, mâu thuẫn.

Vậy $p=13$ là số nguyên tố nhỏ nhất cần tìm.

(b) Trước hết, ta giới thiệu kết quả sau (còn gọi là định lý 4 số):

Bổ Đề. Với các số nguyên dương $a, b, c, d$ thoả mãn $a b=c d$ thì tồn tại các số nguyên dương $x, y, z, t$ sao cho $a=x y, b=z t, c=x z, d=y t$.

Chứng minh bổ đề. Đặt $k=\operatorname{gcd}(a, c)$ và viết $a=k a_1, c=k c_1$ thì rõ ràng $\operatorname{gcd}\left(a_1, c_1\right)=1$. Thay vào đề bài, ta có

$\quad\quad\quad\quad\quad\quad\quad\quad\quad\quad k a_1 b=k c_1 d \text { hay } a_1 b=c_1 d .$

Từ đây chú ý $a_1 \mid c_1 d$, nên $a_1 \mid d$, đặt $d=a_1 \ell$. Thay vào thì có $b=\ell c_1$. Từ đó, ta chọn $x=k, y=a_1, z=c_1, t=\ell$ thì có ngay điều phải chứng minh.

Quay lại bài toán, do $p$ là số chính phương nên đặt $p=a^2, a \in \mathbb{Z}$. Ta viết lại phương trình thành dạng:

$\quad\quad\quad\quad\quad\quad\quad\quad x^3+x+a^2=y^2 \text { hay } x\left(x^2+1\right)=(y-a)(y+a) .$

Áp dụng kết quả trên vào bài toán, ta thấy tồn tại các số nguyên dương $m, n, p, q$ để $x=m n, x^2+1=p q, y+a=m p, y-a=n q$. Từ đó:

$\quad\quad\quad\quad\quad\quad\quad\quad\quad\quad (m n)^2+1=p q \text { và } m p-n q=2 a \text {. }$

Xét dãy số $\left(u_n\right)$ xác định bởi $u_0=0, u_1=1, u_{n+2}=\alpha u_{n+1}+u_n$, trong đó $\alpha$ là hằng số mà ta sẽ chọn sau. Rõ ràng với mọi $n$ thì

$\quad\quad\quad\quad\quad\quad\quad\quad u_n^2-u_{n+1} u_{n-1}=(-1)^{n-1}\left(u_1^2-u_2 u_0\right)=(-1)^{n-1} .$

Khi đó, với $n$ chẵn thì $u_n^2-u_{n+1} u_{n-1}=-1$. Chọn $m n=u_{2 k}$. Ta có:

$\quad\quad\quad\quad\quad\quad\quad u_2=\alpha, u_3=\alpha^2+1, u_4=\alpha\left(\alpha^2+2\right), u_5=\alpha^4+3 \alpha^2+1$

Chọn $p=u_3, q=u_5, m n=u_4$ thì rõ ràng $(m n)^2+1=p q$. Bây giờ ta chỉ cần có được

$\quad\quad\quad\quad\quad\quad m u_3-n u_5=2 a \text { hay } m\left(\alpha^2+1\right)-n\left(\alpha^4+3 \alpha^2+1\right)=2 a .$

Từ đây chọn $\alpha=4 a^2$ và viết $m=2 a\left(\alpha^2+2\right), n=2 a\left(\alpha^4+3 \alpha^2+1\right)$ thì đẳng thức trên sẽ thoả mãn, vì

$\quad\quad\quad\quad\quad\quad\quad\quad \left(\alpha^2+1\right)\left(\alpha^2+2\right)-\left(\alpha^4+3 \alpha^2+1\right)=1 .$

Vậy phương trình có một cặp nghiệm cụ thể là

$\quad\quad (x, y)=\left(4 a^2\left(16 a^4+2\right), 2 a\left(16 a^4+2\right)\left(16 a^4+1\right)-a\right) \text { với } a=\sqrt{p} \in \mathbb{Z}^{+} .$

Bài 4. Cho tam giác $A B C$ nhọn nội tiếp đường tròn $(O)$ với $B, C$ cố định và $A$ di động trên $(O)$. $D$ là trung điểm $B C$. Trên $A B$ lấy các điểm $M, P$ và trên $A C$ lấy các điểm $N, Q$ sao cho $D A=D P=D Q$, dồng thời $D M \perp A C, D N \perp A B$.

(a) Chứng minh rằng các điểm $M, N, P, Q$ cùng thuộc một đường tròn $(\mathcal{C})$ và (C) luôn đi qua một điểm cố định.

(b) Chứng minh rằng tâm của $(\mathcal{C})$ luôn thuộc một đường tròn cố định.

Lời giải . (a) Dễ thấy tam giác $A M Q$ cân tại $M$ nên

$\quad\quad \angle D M Q=\angle D M A=90^{\circ}-\angle A=\frac{180^{\circ}-2 \angle A}{2}=\frac{180^{\circ}-\angle P D Q}{2}=\angle D P Q$

Do đó tứ giác $M P D Q$ nội tiếp. Chứng minh tương tự, ta có tứ giác $Q N D P$ nội tiếp nên $M, N, P, Q$ cùng thuộc một đường tròn $(\mathcal{C})$, và $(\mathcal{C})$ luôn đi qua điểm $D$ cố định.

(b) Gọi $K B, K C$ là hai tiếp tuyến của $(O)$. Ta có $D, K, O$ thẳng hàng, lại có:

$\quad\quad\quad\quad\quad \angle B K O=90^{\circ}-\angle B O K=90^{\circ}-\angle B A C=\angle B M D$

Từ đó tứ giác $B D K M$ nội tiếp. Để ý rằng $K D \perp B C$ nên $K M \perp A B$, hơn nữa $D N \perp A B$ nên $K M | D N$. Tương tự thì $K N | D M$. Do đó $D M K N$ là hình bình hành hay $D K, M N$ có $J$ là trung điểm chung.

Gọi $I$ là tâm của $(\mathcal{C})$ thì $I J \perp M N$ và $J L | A D$. Chú ý rằng $D$ là tâm $(A P Q)$ và cũng là trực tâm tam giác $A M N$ nên $P Q, M N$ là hai đường đối song. Đồng thời nếu $L$ là trung điểm $A D$ thì $J L$ vuông góc với đường nối hai chân đường cao từ $M, N$ của tam giác $A M N$ nên $J L \perp P Q$. Lại có $D P=D Q$ và $I P=I Q$ nên $I D \perp P Q$, do đó $J L | D I$.

Từ đây $I D L J$ là hình bình hành và $I L, D J$ có $T$ là trung điểm chung cố định. Xét phép vị tự tâm $D$ tỉ số $\frac{1}{2}$ hợp với phép đối xứng tâm $T$ thì $A \mapsto I$. Do $A$ thuộc đường tròn $(O)$ cố định nên $I$ cũng thuộc đường tròn cố định là ảnh của $(O)$ qua hợp các phép biến hình trên. Bài toán kết thúc.

Nhận xét. Bài toán này còn một hướng tiếp cận bản chất hơn như sau. Nếu gọi $A^{\prime}$ là điểm đối xứng của $A$ qua $D$ thì $K, A^{\prime}$ là hai điểm liên hợp đẳng giác trong tam giác $A B C$, từ đó đường tròn $(\mathcal{C})$ chính là đường tròn đi qua các hình chiếu của $K, A^{\prime}$ trên các cạnh tam giác $A B C$, dồng thời $I$ là trung diểm $K A^{\prime}$.

Dưới đây là một bài toán tương tự: Cho tam giác nhọn $A B C$ nội tiếp đường tròn $(O)$ có $B C$ cố định và $A$ di dộng trên $(O)$. Gọi $H$ là trực tâm tam giác và lấy điểm $E, F$ thuộc $A B, A C$ theo thứ tự đó sao cho $H$ là trung điểm $E F$.

  1. Chứng minh rằng tâm của đường tròn $(A E F)$ luôn thuộc một đường tròn cố định. Đặt là $\omega$.
  2. Giả sử $\omega$ cắt lại $(O)$ tại các điểm $X, Y$. Chứng minh rằng $X, Y, O$ thẳng hàng.

 

Ngày thi thứ hai

Bài 5. Cho số thực $a \neq 0$. Dãy số $\left(u_n\right)$ thoả mãn:

$\quad\quad\quad\quad\quad\quad\quad\quad\quad\quad u_1=0, u_{n+1}\left(u_n+a\right)=a+1 \forall n \in \mathbb{N}^*$

Tìm giới hạn của dãy số $\left(u_n\right)$.

Lời giải: Đặt $x_{n+1}=(a+1) y_n$ và $y_{n+1}=x_n+a y_n$. Ta có:

$\quad\quad\quad\quad\quad\quad\quad y_{n+2}=x_{n+1}+a y_{n+1}=a y_{n+1}+(a+1) y_n$

Đồng thời $u_n=\frac{x_n}{y_n}$. Để ý rằng $u_1=0, u_2=\frac{a+1}{a}$. Chọn $y_1=1, y_2=a$. Từ đó:

$\quad\quad\quad\quad\quad\quad\quad\quad\quad\quad y_n=\frac{(a+1)^n-(-1)^n}{a+2} \forall n \geq 1$

Công thức trên chỉ xác định với $a \neq-2$ nên xét trường hợp $a=-2$, ta có dãy

$\quad\quad\quad\quad\quad\quad\quad\quad\quad\quad \left\{\begin{array}{l}u_1=0, \\ u_{n+1}=\frac{1}{2-u_n}, n \geq 1\end{array} .\right.$

Bằng quy nạp, ta chứng minh được $u_n \in[0 ; 1)$ nên:

$\quad\quad\quad\quad\quad\quad\quad u_{n+1}-u_n=\frac{1}{2-u_n}-u_n=\frac{\left(u_n-1\right)^2}{2-u_n}>0$

Dãy $\left(u_n\right)$ tăng và bị chặn trên bởi 1 nên có giới hạn hữu hạn là $L \in(0,1)$. Giải phương trình giới hạn, ta có được $L=\frac{1}{2-L}$. Khi đó thì $L=1$.

Tiếp theo, xét $a \neq-2$, ta có:

$\quad\quad\quad\quad u_n=\frac{x_n}{y_n}=\frac{(a+1) y_{n-1}}{y_n}=\frac{(a+1)^n+(a+1)(-1)^n}{(a+1)^n-(-1)^n} \forall n \in \mathbb{N}^*$

Đặt $-(a+1)=b \in{-1 ; 1}$, ta viết lại thành:

$\quad\quad\quad\quad\quad\quad\quad\quad\quad\quad u_n=\frac{b^n-b}{b^n-1} \forall n \geq 1$

Có các khả năng sau xảy ra:

  • Nếu $b>1$ hoặc $b<-1$, tương ứng là $a<-2$ hoặc $a>0$, thì $\lim u_n=1$.
  • Nếu $-1<b<1$, tương ứng là $-2<a<0$, thì $\lim u_n=b=-(a+1)$.

Vậy ta có kết luận sau trong các trường hợp của $a$ :

  • Nếu $a \in(-2 ; 0)$ thì $\lim u_n=-(a+1)$.
  • Nếu $a \notin(-2 ; 0)$ thì $\lim u_n=-1$.

Bài 6. Tìm tất cả các hàm số $f: \mathbb{R}^{+} \rightarrow \mathbb{R}^{+}$thoả mãn diều kiện:

$\quad\quad\quad\quad\quad\quad f\left(x f\left(y^2\right)-y f\left(x^2\right)\right)=(y-x) f(x y) \forall x, y \in \mathbb{R}^{+}, x<y .$

Lời giải . Theo giả thiết thì với mọi $y>x>0$, ta đều có

$\quad\quad\quad\quad\quad\quad\quad\quad x f\left(y^2\right)-y f\left(x^2\right)>0 \Rightarrow \frac{f\left(y^2\right)}{f\left(x^2\right)}>\frac{y}{x}>1 .$

Do đó,

$\quad\quad\quad\quad\quad\quad\quad\quad y^2>x^2 \Leftrightarrow y>x \Leftrightarrow f\left(y^2\right)>f\left(x^2\right)$

nên hàm $f$ dã cho đồng biến trên $\mathbb{R}^{+}$. Trong đề bài, thay $y=x+1$, ta có

$\quad\quad\quad\quad\quad\quad\quad f\left(x f\left((x+1)^2\right)-(x+1) f\left(x^2\right)\right)=f(x(x+1))$

hay

$\quad\quad\quad\quad\quad\quad\quad\quad x f\left((x+1)^2\right)-(x+1) f\left(x^2\right)=x(x+1) $

$\quad\quad\quad\quad\quad\quad\quad\quad \Leftrightarrow \frac{f\left((x+1)^2\right)}{x+1}=\frac{f\left(x^2\right)}{x}+1, \forall x>0$

Thực hiện thao tác này nhiều lần, ta có

$\quad\quad\quad\quad\quad\quad\quad\quad \frac{f\left((x+n)^2\right)}{x+n}=\frac{f\left(x^2\right)}{x}+n, \forall x>0, n \in \mathbb{Z}^{+}$

hay

$\quad\quad\quad\quad\quad\quad\quad\quad x f\left((x+n)^2\right)-(x+n) f\left(x^2\right)=n x(x+n) .$

Trong dề bài, thay $y=x+n$, ta có

$\quad\quad\quad\quad\quad\quad f\left(x f\left((x+n)^2\right)-(x+n) f\left(x^2\right)\right)=n f(x(x+n)) $

$\quad\quad\quad\quad\quad\quad \Leftrightarrow f(n x(x+n))=n f(x(x+n)) .$

Với mọi $n \in \mathbb{Z}^{+}, y>0$, ta luôn chọn được $x>0$ để $x(x+n)=y$ nên ta có

$\quad\quad\quad\quad\quad\quad\quad\quad\quad\quad f(n y)=n f(y), \forall n \in \mathbb{Z}^{+}, y \in \mathbb{R}^{+} .$

Đặt $f(1)=a>0$, với mọi $n \in \mathbb{Z}^{+}$, cho $y=\frac{1}{n}$, suy ra

$\quad\quad\quad\quad\quad\quad\quad\quad\quad\quad f(1)=n f\left(\frac{1}{n}\right) \Rightarrow f\left(\frac{1}{n}\right)=\frac{a}{n} .$

Do đó,

$\quad\quad\quad\quad\quad\quad\quad\quad f\left(\frac{n}{m}\right)=n f\left(\frac{1}{m}\right)=\frac{n}{m} a, \forall m, n \in \mathbb{Z}^{+}$

hay $f(x)=a x, \forall x \in \mathbb{Q}^{+}$. Với mọi số thực $x_0>0$, chọn hai dãy số hữu tỷ $\left(a_n\right),\left(b_n\right)$ sao cho $a_n<x_0<b_n$ và $\lim a_n=\lim b_n=x_0$. Rõ ràng

$\quad\quad\quad\quad\quad\quad f\left(a_n\right)<f\left(x_0\right)<f\left(b_n\right) \Rightarrow a \cdot a_n<f\left(x_0\right)<a \cdot b_n,$

nên cho $n \rightarrow+\infty$, ta có $f\left(x_0\right)=a x_0$. Do đó, với mọi số thực $x>0$ thì $f(x)=a x$. Thay vào biểu thức đã cho, ta có

$\quad\quad\quad\quad\quad \left\{\begin{array}{l}f\left(x f\left(y^2\right)-y f\left(x^2\right)\right)=a^2\left(x y^2-x^2 y\right)=a^2(y-x) x y \\ (y-x) f(x y)=a(y-x) x y\end{array}\right.$

nên $a=1$. Vậy tất cả các hàm số cần tìm là $f(x)=x, \forall x>0$.

Nhận xét. Có một điều đáng chú ý ở bài toán này là việc từ giả thiết, ta phải ngầm hiểu rằng $x f\left(y^2\right)-y f\left(x^2\right)>0$ với mọi cặp số dương $x<y$. Ta có thể thêm tường minh điều kiện đó vào đề bài cho rõ. Tuy nhiên, nếu thêm theo kiểu như sau thì sẽ có một chút vấn đề phát sinh:

Tìm tất cả các hàm số $f: \mathbb{R}^{+} \rightarrow \mathbb{R}^{+}$thoả mãn với mọi cặp số dương $x<y$, nếu $x f\left(y^2\right)-y f\left(x^2\right)>0$ thì

$\quad\quad\quad\quad\quad\quad\quad\quad\quad\quad f()=(y-x) f(x y) \forall x, y \in \mathbb{R}^{+}, x<y .$

Khi đó, ta có thể nhận thêm một hàm số thỏa mãn nữa là $f(x)=\sqrt{x}$. Lý do là vì với mọi cặp số $y>x>0$, ta đều có $x f\left(y^2\right)-y f\left(x^2\right)=0$, mà vì thế, điều kiện “nếu” ở trên là sai nên mệnh đề kéo theo là đúng.

Bài 7. Cho $n=2018.2019$. Gọi $A$ là tập hợp các bộ $\left(a_1, a_2, \ldots, a_n\right)$ có thứ tự thoả mãn điều kiện $a_i \in{0,1} \forall i \in{1,2, \ldots, n}$ và $\sum_{i=1}^n a_i=2018^2$.

Có bao nhiêu bộ $\left(a_1, a_2, \ldots, a_n\right)$ từ $A$ dể:

$\quad\quad\quad\quad\quad \sum_{i=1}^k a_i \geq \frac{a}{2} \text { và } \sum_{i=n-k+1}^n a_i \geq \frac{k}{2} \forall k \in{1,2, \ldots, n}$

Lời giải. Ta giải bài toán tổng quát khi thay 2018 bởi $m \in \mathbb{Z}^{+}$. Bài toán đã cho tương đương với bài toán sau:

Trong hệ trục tọa độ Oxy, xét lưới điểm nguyên trong hình chũ nhật có đỉnh dưới bên trái là $O(0 ; 0)$ và dỉnh trên bên phải là $A\left(m^2 ; m\right)$. Dặt $B(m ; m)$ và $C\left(m^2-m ; 0\right)$, hỏi có bao nhiêu đương đi tù̀ $O \rightarrow A$ sao cho mỗi bước, ta đi sang phải hoặc lên trên 1 đơn vị, gọi là đương đi đơn, và không vượt lên trên $O B$ cũng nhu không xuống dưới $A C$ ?

Ở đây, các số $0 ; 1$ tương ứng với các bước đi lên trên, các bước đi sang phải; còn điều kiện tổng $k$ số đầu và tổng $k$ số cuối không nhỏ hơn $\frac{k}{2}$ tương ứng với số lượng bước đi lên không vượt quá số lượng bước đi sang phải. Để thuận tiện, ta gọi đường đi cắt $d$ nếu nó có các phần nằm về cả hai phía của $d$. Trước hết, ta sẽ chứng minh bổ đề sau:

Bổ Đề. Số đường đi đơn từ $O \rightarrow A(m ; n)$, có cắt đường thẳng $y=x$, là $C_{m+n}^{m+1}$.

Thật vậy, Xét đường thẳng $(d): y=x+1$, rõ ràng các đường đi đơn cắt $y=x$ dều sẽ có điểm chung với đường thẳng $(d)$ này. Tại các điểm chung đó, ta thực hiện đối xứng trục để được một đường đi mới xuất phát từ $O \rightarrow A^{\prime}(n-1, m+1)$.

Trong hình trên, đường cũ là đứt nét, còn đường mới là liền nét. Rõ ràng phép đối xứng trục trên là song ánh, biến các đường cần tìm (cắt $y=x$ ), thành các đường từ $O \rightarrow A^{\prime}$; do đó, số lượng đường cần tìm là $C_{m+n}^{n-1}$.

Trở lại bài toán,

Số đường đi đơn từ $O \rightarrow A\left(m^2 ; m\right)$ là $C_{m^2+m}^m$ vì nó bằng số cách chọn $m$ lần đi lên trong tổng số $m^2+m$ lần di chuyển, trong đó số đường đi cắt $O B$ bằng số đường đi cắt $A C$ và bằng $C_{m^2+m}^{m-1}$ (theo bổ đề).

Do đó, ta chỉ cần tìm số đường đi cắt cả $O B, A C$ với ý tưởng đối xứng hai lần đã dùng để chứng minh bổ đề.

Đầu tiên, ta thực hiện đối xứng qua đường thẳng $y=x+1$; khi đó, các đường đi đơn sẽ xuất phát từ $O \rightarrow A^{\prime}\left(m-1 ; m^2+1\right)$. Do các đường ban đầu còn vượt qua $A C$ nên các đường mới phải cắt thêm $y=x+m^2-m+3$. Tiếp tục đối xứng qua đường thẳng này, ta đưa về đếm số đường đi đơn từ $O \rightarrow A^{\prime \prime}\left(m-2, m^2+2\right)$. Suy ra số đường đi trong trường hợp này là $C_{m^2+m}^{m-2}$. Vậy theo nguyên lý bù trừ, kết quả cần tìm sẽ là

$\quad\quad\quad\quad\quad\quad\quad\quad\quad\quad C_{m^2+m}^m-2 C_{m^2+m}^{m-1}+C_{m^2+m}^{m-2} .$

Thay $m=2018$, ta có số lượng đường đi, cũng chính là số bộ thỏa mãn đề bài.

Nhận xét. Dưới đây là một số kết quả tương tự về đường đi đơn trong đề bài

$1$. Số đường đi đơn từ $(0 ; 0) \rightarrow(m ; n)$ mà không có điểm chung với $y=x$ là

$\quad\quad\quad\quad\quad\quad\quad\quad\quad\quad\quad\quad \frac{m-n}{m+n} C_{m+n}^m .$

$2$. Số đường đi đơn từ $(0 ; 0) \rightarrow(m ; n)$ mà không vượt qua $y=x$ là

$\quad\quad\quad\quad\quad\quad\quad\quad\quad\quad\quad C_{m+n}^n-C_{m+n}^{n-1} .$

$3$. Số đường đi gồm $n$ bước mà không vượt $y=x$ là

$\quad\quad\quad\quad\quad\quad\quad\quad\quad \sum_{i=n / 2}^n \frac{n !(2 i+1-n)}{(i+1) !(n-i) !}=C_n^{[n / 2]} .$

$4$. Số đường đi đơn từ $(0 ; 0) \rightarrow(m ; n)$ mà không có điểm chung với $y=x+t$ là

$\quad\quad\quad\quad\quad\quad\quad\quad\quad\quad\quad\quad C_{m+n}^n-C_{m+n}^{m-t}$.

Bạn đọc có thể dùng phương pháp tương tự trên để giải quyết các bài toán này.

Bài 8. Đường tròn $(\mathcal{C})$ tâm $I$ nội tiếp tam giác $A B C$ và tiếp xúc với các cạnh $A B, A C$ tại $E, F$. $A M, A N$ là các đường phân giác trong, phân giác ngoài của góc $\angle B A C(M, N$ nằm trên $B C)$. Gọi $d_M, d_N$ lần lượt là các tiếp tuyến của $(\mathcal{C})$ qua $M, N$ và khác $B C$.

(a) Chứng minh rằng $d_M, d_N, E F$ dồng quy tại điểm $D$.

(b) Lấy trên $A B, A C$ các điểm $P, Q$ thoả mãn $D P|A C, D Q| A B$. Gọi $R, S$ là trung điểm của $D E, D F$. Chứng minh rằng $I$ thuộc đường thẳng qua các trực tâm của hai tam giác $D P S, D Q R$.

Lời giải. (a) Gọi $X, Y$ lần lượt là tiếp điểm của tiếp tuyến thứ hai kẻ từ $M$ dến $(I)$ và $D^{\prime}$ là tiếp điểm của $(I)$ trên $B C$. Gọi $K$ là trung điểm $E F$.

Xét trong đường tròn $(I)$ thì $E F$ là đường đối cực của $A$ và $K \in E F$ nên đối cực của $K$ sẽ đi qua $A$, mà $N A \perp I A$ nên $N A$ chính là đường đối cực của $K$.

Đường đối cực của $K$ đi qua $N$ nên đối cực của $N$, là $D^{\prime} Y$, sẽ đi qua $K$. Dễ thấy rằng $A M$ là trục đối xứng của tứ giác $D^{\prime} X E F$ nên suy ra $D^{\prime} X | E F$. Xét $D^{\prime}(E F, X Y)$, ta có có $D^{\prime} Y$ đi qua trung điểm của $E F$ và $D^{\prime} X | E F$ nên

$\quad\quad\quad\quad\quad\quad\quad\quad\quad\quad D^{\prime}(E F, X Y)=-1$

hay tứ giác $E X F Y$ điều hòa. Suy ra $M X, N Y, E F$ đồng quy. Ngoài ra ta cũng có $X, Y, A$ thẳng hàng.

(b) Dễ thấy các tam giác $P E D$ và $D Q F$ là các tam giác cân. Gọi $H_1, H_2$ lần lượt là trực tâm của tam giác $\triangle D P S, \triangle D Q R$. Ta có

$\quad\quad\quad\quad\quad\quad\quad\quad \angle P H_1 S=\angle P D F=\angle A F E=\angle P E S$

nên $E P S H_1$ là tứ giác nội tiếp. Suy ra $R H_1 \cdot R P=R S \cdot R E$. Ngoài ra,

$\quad\quad\quad\quad\quad\quad\quad\quad\quad\quad K A \cdot K I=K E \cdot K F$

nên

$\quad\quad\quad\quad\quad\quad\quad\quad\quad \frac{R P}{K A} \cdot \frac{R H_1}{K I}=\frac{R E}{K E} \cdot \frac{R S}{K F} .$

Theo định lý Thales thì $\frac{R P}{K A}=\frac{R E}{K E}$ nên $\frac{R H_1}{K I}=\frac{R S}{K F}$, mà

$\quad\quad\quad\quad\quad\quad\quad R S=R D-S D=\frac{D E-D F}{2}=\frac{E F}{2}=K F$

Suy ra $R H_1=K I$, mà $R H_1 | K I$ (do cùng vuông góc với $E F$ ) nên $I K R H_1$ là hình chữ nhật, kéo theo $I H_1 | E F$. Một cách tương tự, ta có $I H_2 | E F$ vậy nên đường thẳng $H_1 H_2$ đi qua $I$.

Nhận xét. Trong câu a, tính chất $A, X, Y$ thẳng hàng của bài toán cũng đúng khi thay $M, N$ là chân các đường phân giác bởi cặp điểm liên hợp điều hòa bất kỳ với $B, C$. Điều này có được nhờ tính chất của các đường đối cực (hoặc có thể chứng minh nhờ việc sử dụng phép chiếu trực giao các chùm điều hòa).

 

 

 

 

 

 

 

 

 

 

 

 

 

 

 

 

 

 

 

 

 

 

 

 

 

 

 

 

 

 

 

 

 

 

 

 

 

 

 

 

 

 

 

 

 

 

 

 

 

 

 

 

 

 

 

 

 

 

 

 

 

 

 

 

 

 

 

 

 

 

 

 

 

 

 

 

 

 

 

 

 

 

 

 

 

 

 

 

 

 

 

 

 

 

 

 

 

 

 

 

 

 

 

 

 

 

 

 

 

 

 

 

 

 

 

 

 

 

 

 

 

 

 

 

Đề thi và đáp án kì thi chọn đội tuyển thi Quốc gia trường Phổ thông Năng khiếu năm học 2019 – 2020

ĐỀ THI

Ngày thi thứ nhất

Bài 1. Số thực $\alpha$ được gọi là điểm tụ của dãy số $\left(u_n\right)$ nếu tồn tại ít nhất một dãy con của $\left(u_n\right)$ có hội tụ đến $\alpha$.

(a) Hãy chỉ ra một dãy số có vô hạn điểm tụ.

(b) Chứng minh rằng nếu dãy số có mọi dãy con hội tụ thì nó cũng hội tụ.

(c) Gọi $S$ là tập hợp tất cả các số chính phương dương. Dãy số $\left(a_n\right)$ xác định bởi $a_n=\frac{1}{n}$ nếu $n \in S$ và $a_n=\frac{1}{n^2}$ nếu $n \notin S$.

Đặt $b_n=\sum_{k=1}^n a_k$. Xét tính hội tụ của các dãy số $\left(a_n\right)$ và $\left(b_n\right)$.

Bài 2. Tìm tất cả các hợp số dương $n$ sao cho $\sigma(n) \equiv 2(\bmod \varphi(n))$, trong đó ký hiệu $\sigma(n), \varphi(n)$ là hàm tổng các ước của $n$ và hàm Euler.

Bài 3. Tìm tất cả các hàm số $f: \mathbb{R} \rightarrow \mathbb{R}$ thỏa mãn

$\quad\quad\quad\quad\quad f(f(x)+y)+f(x) f(f(y))=x f(y)+x+y, \forall x, y \in \mathbb{R} .$

Bài 4. Cho tam giác $A B C$ không cân nội tiếp trong đường tròn $(O)$ với $B C$ cố định và $A$ thay đổi trên cung lớn $B C$. Các đường tròn bàng tiếp góc $A, B, C$ lần lượt tiếp xúc với $B C, C A, A B$ tại $D, E, F$. Gọi $L, M, N$ lần lượt là giao điểm khác $A, B, C$ của $(A B E),(A C F) ;(B C F),(B A D) ;(C A D),(C B E)$.

(a) Chứng minh rằng $A L$ luôn đi qua điểm cố định khi $A$ thay đổi.

(b) Gọi $K, I, J$ lần lượt là trung điểm của $A D, B E, C F$. Chứng minh rằng $K L, I M, J N$ dồng quy.

Ngày thi thứ hai

Bài 5. Cho $a, b, c$ là các số thực dương thỏa mãn $8\left(a^2+b^2+c^2\right)=9(a b+b c+c a)$.

Tìm giá trị lớn nhất và giá trị nhỏ nhất của biểu thức

$\quad\quad\quad\quad\quad\quad\quad\quad\quad\quad T=\frac{a+b}{c}+\frac{b+c}{a}+\frac{c+a}{b} .$

Bài 6. Tìm tất cả các hàm số $f: \mathbb{Z}^{+} \rightarrow \mathbb{Z}^{+}$thỏa mãn đồng thời các điều kiện sau

$\quad\quad$ i) $m f(m)+n f(n)+2 m f(n)$ là số chính phương với mọi $m, n$;

$\quad\quad$ ii) $f(m n)=f(m) f(n)$ với mọi $m, n$ nguyên dương;

$\quad\quad$  iii) Với mọi số nguyên tố $p, f(p)$ không chia hết cho $p^2$.

Bài 7. Một trường phổ thông có $n$ học sinh. Các học sinh tham gia vào tổng cộng $m$ câu lạc bộ là $A_1, A_2, \ldots, A_m$.

(a) Chứng minh rằng nếu mỗi câu lạc bộ có 4 học sinh và hai học sinh bất kỳ tham gia chung nhất một câu lạc bộ thì $m \leq \frac{n(n-1)}{12}$.

(b) Giả sử tồn tại $k>0$ sao cho hai câu lạc bộ bất kỳ có chung nhau $k$ thành viên và tồn tại một câu lạc bộ $A_t$ có $k$ thành viên. Chứng minh rằng $m \leq n$

Bài 8. Cho tam giác $A B C$ nội tiếp đường tròn $(O)$. Đường tròn nội tiếp $(I)$ tiếp xúc với các cạnh $B C, C A, A B$ lần lượt tại $D, E, F$. Gọi $J$ là tâm bàng tiếp góc $A$ của tam giác $A B C$ và $H$ là hình chiếu của $D$ lên $E F$.

(a) Chứng minh rằng giao điểm của $A H, J D$ thì thuộc đường thẳng $O I$.

(b) Giả sử $D H$ cắt lại $(I)$ ở $K$ và $I K$ cắt lại đường tròn ngoại tiếp $(I E F)$ ở $L$. Chứng minh rằng $A D, L H$ cắt nhau tại một điểm nằm trên $(I E F)$.

 

LỜI GIẢI

Ngày thi thứ nhất

Bài 1. Số thực $\alpha$ được gọi là điểm tụ của dãy số $\left(u_n\right)$ nếu tồn tại ít nhất một dãy con của $\left(u_n\right)$ có hội tụ đến $\alpha$.

(a) Hãy chỉ ra một dãy số có vô hạn điểm tụ.

(b) Chứng minh rằng nếu một dãy số có mọi dãy con hội tụ thì nó cũng hội tụ.

(c) Gọi $S$ là tập hợp tất cả các số chính phương dương. Dãy số $\left(a_n\right)$ xác định bởi $a_n=\frac{1}{n}$ nếu $n \in S$ và $a_n=\frac{1}{n^2}$ nếu $n \notin S$.

Đặt $b_n=\sum_{k=1}^n a_k$. Xét tính hội tụ của các dãy số $\left(a_n\right)$ và $\left(b_n\right)$.

Lời giải. (a) Ta sẽ chỉ ra dãy số mà mỗi số nguyên dương xuất hiện vô hạn lần trong đó. Chẳng hạn $\left(u_n\right)$ : là

$\quad\quad\quad\quad\quad\quad\quad\quad\quad\quad 1,2,1,2,3,1,2,3,4,1,2,3,4,5, \ldots$

với $u_n=1$ nếu $n \in S$ và $u_{n+1}=u_n+1$ nếu $n \notin S$, trong đó $S$ là tập hợp các số có dạng $\frac{m(m+1)}{2}$ như $1,3,6,10,15, \ldots$ Khi đó, với mỗi số nguyên dương $m \in \mathbb{Z}^{+}$thì ta luôn có thể trích ra một dãy con vô hạn của $\left(u_n\right)$ có tất cả các phần tử đều bằng $m$, tức là hội tụ về $m$.

(b) Do mỗi dãy số là dãy con của chính nó nên rõ ràng khẳng định của bài toán là đúng.

(c) Ta có $0 \leq a_n \leq \frac{1}{n}$ với mọi $n$ nên theo nguyên lí kẹp, ta suy ra $\lim a_n=0$. Nhận xét rằng $b_n$ là dãy tăng. Ta có

$\quad\quad\quad\quad\quad b_{n^2}=\sum_{i=1}^{n^2} a_i =\sum_{i \in S, i=1}^{n^2} a_i+\sum_{i \notin S, i=1}^{n^2} a_i=\sum_{i \in S, i=1}^{n^2} \frac{1}{i}+\sum_{i \notin S, i=1}^{n^2} \frac{1}{i^2} $

$\quad\quad\quad\quad\quad\quad\quad\quad\quad\quad\quad =\left(1+\frac{1}{2^2}+\cdots+\frac{1}{n^2}\right)+\sum_{i \notin S, i=1}^{n^2} \frac{1}{i^2} $

$\quad\quad\quad\quad\quad\quad\quad\quad\quad\quad\quad <\left(1+\frac{1}{2^2}+\cdots+\frac{1}{n^2}\right)+\left(\sum_{i=1}^{n^2} \frac{1}{i^2}\right) $

$\quad\quad\quad\quad\quad\quad\quad\quad\quad\quad\quad =\left(1+\frac{1}{2^2}+\cdots+\frac{1}{n^2}\right)+\left(1+\frac{1}{2^2}+\cdots+\frac{1}{n^4}\right)$

Vì dãy $u_n=1+\frac{1}{2^2}+\cdots+\frac{1}{n^2}<1+\frac{1}{1 \cdot 2}+\frac{1}{2 \cdot 3}+\cdots+\frac{1}{(n-1) n}=2-\frac{1}{n}<2$ là bị chặn trên nên từ đánh giá đã xây dựng được, ta có $b_{n^2}$ cũng bị chặn trên. Kết hợp với $b_n$ là dãy tăng, ta suy ra bản thân dãy $b_n$ cũng bị chặn trên nên nó hội tụ.

Bài 2. Tìm tất cả các hợp số dương $n$ sao cho

$\quad\quad\quad\quad\quad\quad\quad\quad\quad\quad \sigma(n) \equiv 2 \quad(\bmod \varphi(n)),$

trong đó ký hiệu $\sigma(n), \varphi(n)$ là hàm tổng các ước của $n$ và hàm Euler.

Lời giải . Giả sử $p$ là một ước nguyên tố lẻ của $n$. Nếu $v_p(n)>1$ thì theo công thức của hàm Euler, ta có $p \mid \varphi(n)$, mà $n \cdot \sigma(n)-2$ chia hết cho $\varphi(n)$, tức là cũng chia hết cho $p$ nên kéo theo $p \mid 2$, vô lý. Suy ra $v_p(n)=1$ với mọi $p \mid n$.

Đặt $n=2^k \cdot p_1 p_2 \ldots p_t$ với $k \geq 0$ và $p_1<p_2<\ldots<p_t$ là các số nguyên tố phân biệt. Theo công thức tính các hàm, ta có

$\quad\quad\quad\quad\quad\quad\quad \varphi(n)=2^{k-1}\left(p_1-1\right)\left(p_2-1\right) \ldots\left(p_t-1\right)$

$\quad\quad\quad\quad\quad\quad\sigma(n)=\left(2^{k+1}-1\right)\left(p_1+1\right)\left(p_2+1\right) \ldots\left(p_t+1\right) .$

Đánh giá lũy thừa 2 trong các số trên, ta có

$\quad\quad\quad\quad\quad\quad v_2(\varphi(n)) \geq k-1+t \text { và } v_2(n \cdot \sigma(n)) \geq k+t .$

Do đó từ $\varphi(n) \mid n \cdot \sigma(n)-2$, ta suy ra $1 \geq k-1+t$ nên $k+t \leq 2$. Ta xét các trường hợp sau

  • Nếu $t=0$ thì $n=2^k$ là hợp số nên $k=2, n=4$, thử trực tiếp ta thấy thỏa.
  • Nếu $t=1$ thì $n=2 p$ nên $\varphi(n)=p-1, \sigma(n)=3(p+1)$ và đưa về

$\quad\quad\quad\quad\quad\quad\quad\quad\quad\quad p-1 \mid 6 p(p+1)-2$

Chú ý rằng

$\quad\quad\quad\quad\quad\quad 6 p(p+1)-2=6 p^2+6 p-2=(p-1)(6 p+12)-10$

nên $p-1 \mid 10$. Từ đó ta tìm được $p=3, p=11$ tương ứng với $n=6, n=22$.

  • Nếu $t=2$ thì $k=0$, ta có $n=p_1 p_2$ nên

$\quad\quad\quad\quad \varphi(n)=\left(p_1-1\right)\left(p_2-1\right) \text { và } \sigma(n)=\left(p_1+1\right)\left(p_2+1\right)$

đưa về

$\quad\quad\quad\quad \left(p_1-1\right)\left(p_2-1\right) \mid\left(p_1+1\right)\left(p_2+1\right)-2 .$

Điều này không thể xảy ra vì $\left(p_1-1\right)\left(p_2-1\right)$ chia hết cho 4 trong khi biểu thức còn lại chia 4 dư 2 . Do đó, trường hợp này không có số $n$ thỏa mãn.

Vậy tất cả các số cần tìm là $4,6,22$.

Nhận xét. Chú ý rằng mọi số nguyên tố đều thỏa mãn yêu cầu của đề bài.

Bài 3. Tìm tất cả các hàm số $f: \mathbb{R} \rightarrow \mathbb{R}$ thỏa mãn

$\quad\quad\quad\quad\quad\quad f(f(x)+y)+f(x) f(f(y))=x f(y)+x+y$

với mọi số thực $x, y$.

Lời giải. Thay $x=y=0$ vào phương trình đề cho, ta có

$\quad\quad\quad\quad\quad\quad\quad\quad\quad\quad f(f(0))+f(0) f(f(0))=0 .$

suy ra $f(f(0))=0$ hoặc $f(0)=-1$. Ta xét các trường hợp sau:

  1. Nếu $f(f(0))=0$. Thay $y=0$, vào phương trình dề cho, ta có $f(f(x))=$ $x f(0)+x, \forall x \in \mathbb{R}$ Thay $x=f(0)$ và sử dụng $f(f(0))=0$, ta được $f(0)=$ $[f(0)]^2+f(0)$, hay $f(0)=0$. Do đó $f(f(x))=x$ với mọi $x \in \mathbb{R}$. Thay vào phương trình đề bài, ta có

$\quad\quad\quad\quad\quad\quad f(f(x)+y)+y f(x)=x f(y)+x+y, \forall x, y \in \mathbb{R} .$

Thay $y$ bởi $f(y)$ và sử dụng tính đối xứng của vế trái, ta được

$\quad\quad\quad\quad\quad\quad f(f(x)+f(y))+f(x) f(y)=x y+x+f(y)=x y+y+f(x) .$

Do đó $f(x)-x=f(y)-y$ với mọi $x, y \in \mathbb{R}$, hay $f(x)=x+c$. Thử lại, ta có $c=0$.

  1. Nếu $f(0)=-1$. Thay $y=0$ vào phương trình đề cho, ta có $f(f(x))+$ $f(x) f(-1)=0, \forall x \in \mathbb{R}$. Từ đây suy ra $f(f(-1))=-[f(-1)]^2$. Thay $x=0$ vào phương trình đề cho, ta có $f(y-1)-f(f(y))=y, \forall y \in \mathbb{R}$. Kết hợp các đẳng thức trên lại, ta có

$\quad\quad\quad\quad\quad\quad\quad\quad f(x-1)+f(x) f(-1)=x, \forall x \in \mathbb{R} .$

Thay $y=-1$ vào phương trình đề cho và sử dụng $f(f(-1))=-[f(-1)]^2$, ta lại có

$\quad\quad\quad\quad\quad\quad f(f(x)-1)-f(x)[f(-1)]^2=x f(-1)+x-1, \forall x \in \mathbb{R} .$

Mặt khác, ta cũng có

$\quad\quad\quad\quad\quad\quad\quad\quad f(-1) f(f(x))+f(x)[f(-1)]^2=0, \forall x \in \mathbb{R} .$

Cộng vế theo vế hai biểu thức trên lại, ta có

$\quad\quad\quad\quad\quad\quad\quad\quad\quad\quad f(x)=[1+f(-1)] x+1, \forall x \in \mathbb{R} .$

Thử lại, ta thấy không thỏa mãn.

Vậy phương trình có nghiệm hàm duy nhất là $f(x)=x$.

Bài 4. Cho tam giác $A B C$ không cân nội tiếp trong đường tròn $(O)$ với $B C$ cố định và $A$ thay đổi trên cung lớn $B C$. Các đường tròn bàng tiếp góc $A, B, C$ lần lượt tiếp xúc với các cạnh $B C, C A, A B$ tại $D, E, F$. Gọi $L, M, N$ lần lượt là giao điểm khác $A, B, C$ của các cặp đường tròn

$\quad\quad\quad\quad (A B E),(A C F) ;(B C F),(B A D) ;(C A D),(C B E) .$

(a) Chứng minh rằng $A L$ luôn đi qua điểm cố định khi $A$ thay đổi.

(b) Gọi $K, I, J$ lần lượt là trung điểm của $A D, B E, C F$. Chứng minh rằng $K L, I M, J N$ đồng quy.

Lời giải . (a) Đặt $B C=a, C A=b, A B=c$ và $p$ là nửa chu vi thì theo tính chất tiếp điểm bàng tiếp, ta có $B F=C E=p-a$.

Bằng biến đổi góc, ta có được $\triangle L B F \sim \triangle L E C(g . g)$, mà $B F=C E$ nên hai tam giác này bằng nhau. Suy ra $L B=L E, L C=L F$ nên $L$ là trung điểm cung $B E$ của đường tròn $(A B E)$ và cũng là trung diểm cung $C F$ của $(A C F)$.

Từ đó ta có $A L$ là phân giác góc $B A C$ hay $A L$ luôn đi qua trung điểm cung nhỏ $B C$ của $(O)$, là điểm cố định.

(b) Để ý rằng vai trò của $M, N, L$ là bình đẳng trong tam giác $A B C$. Do đó, từ câu a, một cách tương tự, ta có $M, N$ thuộc phân giác góc $B, C$ nên cũng lần lượt là trung điểm các cung nhỏ của các đường tròn tương ứng. Suy ra $M, K, N$ thẳng hàng (cùng thuộc trung trực của đoạn $A D$ ); tương tự với các bộ ba $N, I, L$ và $L, J, M$. Cuối cùng, ta thấy rằng

$\quad\quad\quad\quad\quad\quad\quad \frac{K M}{K N}=\frac{A K \cdot \tan \angle M A K}{A K \cdot \tan \angle N A K}=\frac{\tan (B / 2)}{\tan (C / 2)} .$

Tương tự với các tỷ số khác. Đến đây, áp dụng định lý Ceva cho tam giác $M N L$, ta có các đoạn thẳng $L K, I M, J N$ dồng quy.

Nhận xét. Một cách khác cho câu a như sau: Xét phép nghịch đảo đối xứng với phương tích $k=A B \cdot A C$ và trục đối xứng là phân giác góc $A$. Ta có $E \rightarrow E^{\prime} \in$ $A C, F \rightarrow F^{\prime} \in A B$ sao cho $A E \cdot A E^{\prime}=A F \cdot A F^{\prime}=k$. Ta tính được

$\quad\quad\quad\quad A E^{\prime}=\frac{b c}{p-c} \rightarrow B E^{\prime}=\frac{c(p-a)}{p-c} \rightarrow \frac{E^{\prime} B}{E^{\prime} A}=\frac{p-a}{b} .$

Tương tự thì $\frac{F^{\prime} C}{F^{\prime} A}=\frac{p-a}{c}$. Áp dụng định lý Ceva cho tam giác $A B C$ thì $C E^{\prime}, B F^{\prime}$ và phân giác góc $A$ đồng quy.

Lại có qua phép nghịch đối xứng trên thì phân giác giữ nguyên,

$\quad\quad\quad\quad\quad\quad\quad\quad\quad\quad (A B E) \rightarrow C F^{\prime},(A C F) \rightarrow B E^{\prime}$

nên ta có $L$ thuộc phân giác góc $A$.

 

Ngày thi thứ hai

Bài 5. Cho $a, b, c$ là các số thực dương thỏa mãn $8\left(a^2+b^2+c^2\right)=9(a b+b c+c a)$. Tìm giá trị lớn nhất và giá trị nhỏ nhất của biểu thức

$\quad\quad\quad\quad\quad\quad\quad\quad\quad\quad T=\frac{a+b}{c}+\frac{b+c}{a}+\frac{c+a}{b} .$

Lời giải . Do tính thuần nhất đối xứng của các biến nên chuẩn hóa

$\quad\quad\quad\quad\quad a b+b c+c a=8 \rightarrow a^2+b^2+c^2=9 \rightarrow a+b+c=5 .$

Ta có $P+3=(a+b+c)\left(\frac{1}{a}+\frac{1}{b}+\frac{1}{c}\right)=\frac{40}{a b c}$ nên ta đưa về tìm min, max của $T=a b c$ trong điều kiện

$\quad\quad\quad\quad\quad\quad\quad\quad\quad\quad\quad \left\{\begin{array}{l}a+b+c=5 \\ a b+b c+c a=8\end{array}\right.$

Chú ý rằng $b+c=5-a, b c=8-a(b+c)=8-a(5-a)$ nên từ đánh giá quen thuộc $(b+c)^2 \geq 4 b c$, ta có

$\quad\quad\quad\quad\quad\quad (5-a)^2 \geq 4\left(8-5 a+a^2\right) \Leftrightarrow 1 \leq a \leq \frac{7}{3} .$

Suy ra $T=a b c=a\left(8+a^2-5 a\right)=f(a)$. Đến đây khảo sát hàm số này trên miền $\left[1 ; \frac{7}{3}\right]$, ta được $\min T=4, \max T=\frac{112}{27}$ nên $\min P=\frac{93}{14}$, $\max P=7$. Từ đó, ta thu được kết luận như sau

  • Giá trị lớn nhất của $P$ là 7 , đạt được chẳng hạn khi $(a, b, c)=(2,2,1)$.
  • Giá trị nhỏ nhất của $P$ là $\frac{93}{14}$, đạt được chẳng hạn khi $(a, b, c)=\left(\frac{7}{3}, \frac{4}{3}, \frac{4}{3}\right)$.

Bài 6. Tìm tất cả các hàm số $f: \mathbb{Z}^{+} \rightarrow \mathbb{Z}^{+}$thỏa mãn đồng thời các điều kiện sau đây

$\quad\quad$ i) $m f(m)+n f(n)+2 m f(n)$ là số chính phương với mọi $m, n$;

$\quad\quad$  ii) $f(m n)=f(m) f(n)$ với mọi $m, n$ nguyên dương;

$\quad\quad$  iii) Với mọi số nguyên tố $p, f(p)$ không chia hết cho $p^2$.

Lời giải . Thay $m=n=1$ vào ii), ta suy ra $f(1)=f(1)^2$ nên $f(1)=1$. Thay $m=n$ vào i), ta suy ra $4 m f(m)$ là số chính phương với mọi $m \in \mathbb{Z}^{+}$nên $m f(m)$ cũng là số chính phương với mọi $m \in \mathbb{Z}^{+}$.

Với $p$ là số nguyên tố, vì $p f(p)$ là số chính phương nên $p \mid f(p)$ và ta đặt $f(p)=k^2 p$, với $k$ là số nguyên dương nào đó. Thay $m=p, n=1$ vào i), ta suy ra $p f(p)+1+2 p$ là số chính phương, hay $k^2 p^2+2 p+1$ là số chính phương.

Vì $k^2 p^2+2 p+1>(k p)^2$ nên

$\quad\quad\quad\quad\quad\quad k^2 p^2+2 p+1 \geq(k p+1)^2=k^2 p^2+2 k p+1 .$

Do đó $2 p \geq 2 k p$ nên ta phải có $k=1$.

Vì thế nên $f(p)=p$ với mọi số nguyên tố $p$. Sử dụng điều kiện ii), hàm $f$ nhân tính, và cũng vì mọi số nguyên dương bất kỳ đều có thể viết dưới dạng tích của các số nguyên tố nên ta có được $f(n)=n$ với mọi $n \in \mathbb{Z}^{+}$.

Thử lại ta thấy hàm số này thỏa mãn các ràng buộc của đề bài.

Nhận xét. Trên thực tế, ta có thể bỏ bớt diều kiện ii), iii) đi mà bài toán gốc vẫn có thể giải quyết được. Cụ thể như sau:

Chứng minh rằng nếu hàm số $f: \mathbb{Z}^{+} \rightarrow \mathbb{Z}^{+}$thỏa mãn $f(1)=1$ và với mọi $m, n \in \mathbb{Z}^{+}$, ta có $m f(m)+n f(n)+2 m f(n)$ là số chính phương thì $f(n)=n, \forall n \in \mathbb{Z}^{+}$.

Bài 7. Một trường phổ thông có $n$ học sinh. Các học sinh tham gia vào tổng cộng $m$ câu lạc bộ là $A_1, A_2, \ldots, A_m$.

(a) Chứng minh rằng nếu mỗi câu lạc bộ có 4 học sinh và hai học sinh bất kỳ tham gia chung nhất một câu lạc bộ thì $m \leq \frac{n(n-1)}{12}$.

(b) Giả sử tồn tại $k>0$ sao cho hai câu lạc bộ bất kỳ có chung nhau $k$ thành viên và tồn tại một câu lạc bộ $A_t$ có $k$ thành viên. Chứng minh rằng $m \leq n$

Lời giải . (a) Gọi $S$ là số bộ $({A, B}, C)$ mà trong đó học sinh $A, B$ cùng tham gia vào câu lạc bộ $C$. Ta thực đếm $S$ bằng hai cách

  1. Chọn câu lạc bộ trước, có $m$ cách, chọn cặp học sinh cùng tham gia vào đó có $C_4^2=6$ cách nên $S=6 \mathrm{~m}$.
  2. Chọn cặp học sinh trước, có $C_n^2$ cách, chọn câu lạc bộ mà hai học sinh đó cùng tham gia, có không quá 1 cách nên $S \leq C_n^2$.

Từ đó suy ra

$\quad\quad\quad\quad\quad\quad\quad\quad\quad 6 m \leq C_n^2 \Leftrightarrow m \leq \frac{n(n-1)}{12} .$

(b) Xét câu lạc bộ $X$ nào đó có $k$ thành viên. Xét $m-1$ câu lạc bộ còn lại thì theo giả thiết, rõ ràng các câu lạc bộ này đều có chứa $k$ thành viên trên của câu lạc bộ $X$. Từ đó suy ra $m-1$ câu lạc bộ còn lại đôi một không có thành viên chung.

Xét $n-k$ học sinh còn lại của trường thì rõ ràng một học sinh thuộc tối đa một câu lạc bộ (trong số các câu lạc bộ còn lại), suy ra số câu lạc bộ còn lại không vượt quá $n-k$ nên suy ra $m \leq n-k+1 \leq n$. Ta có điều phải chứng minh.

Nhận xét. Ý b của bài toán khá hiển nhiên, nhưng thực ra nó là một “phiên bản dễ” của bất đẳng thức Fisher sau đây:

Cho $A_1, A_2, \ldots, A_m$ là các tập con của tập ${1,2, \ldots, n}$ sao cho hai tập con bất kỳ có chung nhau đúng $k$ (với $k$ là số nguyên cố định nào đó không vượt quá n). Khi đó $m \leq n$.

Tuy nhiên, chứng minh sơ cấp cho kết quả này quả thực rất khó. Cách phổ biến nhất là dùng đại số tuyến tính. Cụ thể là:

Ta đặt tương ứng mỗi tập $A_i$ với một vector $v_i$ trong $\mathbb{F}_2^n$ như sau

$\quad\quad\quad\quad\quad\quad\quad\quad v_{i j}=\left\{\begin{array}{l}1 \text { nếu } j \in A_i \\ 0 \text { nếu } j \notin A_i\end{array} .\right.$

Chú ý rằng $\left|A_i \cap A_j\right|=k$ với mọi $i \neq j$. Bởi vậy, các vector $v_1, \ldots, v_m$ là các vector trong $\mathbb{R}^n$. Mặt khác, ta có số chiều của $\mathbb{R}^n$ là $n$. Do đó, trong bước tiếp theo chúng ta chỉ cần chứng minh $v_1, \ldots, v_m$ độc lập tuyến tính trong không gian $\mathbb{R}^n$.

Giả sử phản chứng rằng tồn tại các hệ số $\alpha_1, \ldots \alpha_m$ không đồng nhất bằng không sao cho $\sum_{i=1}^m \alpha_i v_i=0$. Do đó, ta có

$\quad\quad\quad\quad\quad\quad 0 =\left|\sum_{i=1}^m \alpha_i v_i\right|^2=\left\langle\sum_{i=1}^m \alpha_i v_i, \sum_{i=1}^m \alpha_i v_i\right\rangle $

$\quad\quad\quad\quad\quad\quad\quad =\sum_{i=1}^m \alpha_i^2\left|v_i\right|^2+\sum_{1 \leq i \neq j \leq m} \alpha_i \alpha_j\left\langle v_i, v_j\right\rangle$

Mặt khác, $\left|v_i\right|^2=\left|A_i\right|$, và $\left\langle v_i, v_j\right\rangle=\left|A_i \cap A_j\right|$. Bởi vậy,

$\quad\quad\quad\quad 0=\sum_{i=1}^m \alpha_i^2\left|v_i\right|^2+\sum_{i \neq j} k \alpha_i \alpha_j=\sum_{i=1}^m \alpha_i^2\left(\left|A_i\right|-k\right)+k \sum_{1 \leq i, j \leq m} \alpha_i \alpha_j$

Ta thấy rằng $\sum_{1 \leq i, j \leq m} \alpha_i \alpha_j=\left(\sum_{1 \leq i \leq m} \alpha_i\right)^2$, nên $0=\sum_{i=1}^m \alpha_i^2\left(\left|A_i\right|-k\right)+k\left(\sum_{1 \leq i, j \leq m} \alpha_i\right)^2$. Vì $\left|A_i\right| \geq k$ và có nhiều nhất một tập với chính xác $k$ phần tử, nên $\alpha_1=\cdots=\alpha_m=0$.

Điều này mâu thuẫn với giả thiết, hay các vector $v_1, \ldots, v_m$ là độc lập tuyến tính. Như vậy ta sẽ có $m \leq n$.

Bài 8. Cho tam giác $A B C$ nội tiếp đường tròn $(O)$. Đường tròn nội tiếp $(I)$ tiếp xúc với các cạnh $B C, C A, A B$ lần lượt tại $D, E, F$. Gọi $J$ là tâm bàng tiếp góc $A$ của tam giác $A B C$ và $H$ là hình chiếu của $D$ lên $E F$.

(a) Chứng minh rằng giao điểm của $A H, J D$ thì thuộc đường thẳng $O I$.

(b) Giả sử $D H$ cắt lại $(I)$ ở $K$ và $I K$ cắt lại đường tròn ngoại tiếp $(I E F)$ ơ $L$. Chứng minh rằng $A D, L H$ cắt nhau tại một diểm nằm trên $(I E F)$.

Lời giải. (a) Ta có bổ đề sau:

Bổ ĐỀ. $O I$ là đường thẳng Euler của tam giác $D E F$.

Bổ đề này quen thuộc và có thể chứng minh bằng cách hướng như sau (chi tiết xin dành cho bạn đọc).

  1. Sử dụng phép nghịch đảo tâm $I$, phương tích $r^2$ biến $(O)$ thành đường tròn Euler của $D E F$ nên có các tâm thẳng hàng.
  2. Sử dụng phép vị tự bằng cách gọi thêm trung điểm các cung nhỏ $B C, C A, A B$ của $(O)$.

Khi đó, gọi $T$ là giao điểm của $I O$ và $H D$ thì rõ ràng $T$ là trực tâm của tam giác $D E F$. Gọi $M$ là trung điểm cung nhỏ $B C$ của $(O)$ thì dễ thấy $M$ là trung điểm $I J$.

Bằng biến đổi góc, ta có $\triangle T E F \sim \triangle I B C$, mà $T H, I D$ là hai đường cao tương ứng nên $\frac{T H}{I D}=\frac{E F}{B C}$. Mặt khác, $\triangle I E F \sim \triangle M B C$ nên

$\quad\quad\quad\quad\quad\quad\quad\quad \frac{E F}{B C}=\frac{I E}{M C}=\frac{2 I E}{2 M I}=\frac{2 I E}{I J}$

suy ra $\frac{T H}{I D}=\frac{2 I E}{I J}$. Do đó

$\quad\quad\quad\quad\quad\quad T H \cdot I J=2 I D^2=2 I N \cdot I A=T D \cdot I A$

(vì $I, T$ lần lượt là tâm ngoại tiếp và trực tâm tam giác $D E F$ ) nên $\frac{T H}{T D}=\frac{I A}{I J}$. Cuối cùng, vì $H D | A J$ (cùng vuông góc với $E F$ ) nên theo định lý Talet thì $A H, J D, T I$ đồng quy hay nói cách khác, $A H, J D$ cắt nhau trên $O I$.

(b) Giả sử $A D$ cắt lại $(I)$ tại $G$. Ta cần chứng minh rằng $G, H, L$ thẳng hàng.

Do $D K | A I$ nên

$\quad\quad\quad\quad\quad\quad\quad\quad\quad\quad\angle A G L=\angle A I L=\angle A I K=\angle D K I$

suy ra $\angle D G L=\angle D K L$. Vì thế nên $D G K L$ là tứ giác nội tiếp. Do đó, $L G$ là trục đẳng phương của $(L K D),(I E F)$. Lại có

nên suy ra $H$ thuộc trục đẳng phương của hai đường tròn này, tức là $H \in L G$. Từ đó ta có điều phải chứng minh.

Nhận xét. Liên quan đến ý b, có một bài toán khá thú vị với nội dung như sau:

Trung tuyến đỉnh $D$ của tam giác $D E F$ cắt $(I)$ ở $L$. Chứng minh rằng trục đẳng phương của $(L B F),(L C E)$ đi qua giao điểm của $J D$ và đường thẳng qua $A$, vuông góc với $A I$.

 

 

 

 

 

 

 

 

 

 

 

 

 

 

 

 

 

 

 

 

 

 

 

 

 

 

 

 

 

 

 

 

 

 

 

 

 

 

 

 

 

 

 

 

 

 

 

 

 

 

 

 

 

 

 

 

 

 

 

 

 

 

 

 

 

 

 

 

 

 

 

 

 

 

 

 

 

 

 

 

 

 

 

 

 

 

 

 

 

 

 

 

 

 

 

 

 

 

 

 

Đề thi và đáp án kì thi chọn đội tuyển thi Quốc gia trường Phổ thông Năng khiếu năm học 2014 – 2015

ĐỀ THI

 

Ngày thi thứ nhất

Bài 1. Cho $a, b, c>0$ thỏa mãn điều kiện $(a+1)(b+1)(c+1)=1+4 a b c$. Chứng mình rằng ta có bất đẳng thức $a+b+c \leq 1+a b c$.

Bài 2. Cho tập hợp $A=[n^3-4 n+15 \mid n \in \mathbb{N}]$. Tìm tất cả các phần tử $a \in A$ thỏa mãn đồng thời hai điều kiện sau đây:

$\quad (i)\quad a$ là số chẵn.

$\quad (ii)$ Nếu $a_1, a_2$ là các ước số của $\frac{a}{2}$ với $a_1, a_2>1$ thì $\operatorname{gcd}\left(a_1, a_2\right)>1$.

Bài 3. Tìm tất cả các hàm số $f: \mathbb{N}^* \rightarrow \mathbb{N}^*$ thỏa mãn:

$\quad\quad\quad\quad\quad\quad\quad\quad\quad\quad f\left(\frac{f(n)}{n}\right)=n^2 \forall n \in \mathbb{N}^*$

Bài 4. Cho tam giác $A B C$ nội tiếp $(O)$, có $B, C$ cố định và $A$ thay đổi trên $(O)$. Ký hiệu $(I)$ là đường tròn nội tiếp tam giác $A B C$. Gọi $\left(O_1\right)$ là đường tròn qua $A, B$ và tiếp xúc với đường tròn $(I)$ tại $E$. Gọi $\left(O_2\right)$ là đường tròn qua $A, C$ và tiếp xúc với đường tròn $(I)$ tại $F$. Đường phân giác trong của góc $\angle A E B$ cắt $\left(O_1\right)$ tại $M$ và đường phân giác trong của góc $\angle A F C$ cắt $\left(O_2\right)$ tại $N$.

(a) Chứng minh rằng tứ giác $E F M N$ nội tiếp.

(b) Gọi $J$ là giao điểm của $E M$ và $F N$. Chứng minh rằng đường thẳng $I J$ luôn đi qua một điểm cố định.

Ngày thi thứ hai

Bài 5. Cho dãy số $\left(x_n\right)$ bởi $x_0=1, x_1=2014$ và $x_{n+1}=\sqrt[3]{x_n x_{n-1}^2} \forall n \in \mathbb{N}^*$.

(a) Chứng minh rằng dãy số $\left(x_n\right)$ có giới hạn hữu hạn và tìm giới hạn đó.

(a) Với mỗi $n \geq 2$, hãy tìm số nguyên dương $k$ nhỏ nhất sao cho $a=x_n^k$ là một số nguyên. Chứng minh rằng khi đó $a$ không thể viết được dưới dạng tổng các lũy thừa bậc ba của hai số tự nhiên.

Bài 6. Cho $X$ là tập hợp gồm 19 phần tử.

(a) Chứng minh rằng tồn tại ít nhất 2600 tập con 7 phần tử của $X$ sao cho với hai tập con $A, B$ bất kỳ trong số 2600 tập con đó, ta có $|A \cap B| \leq 5$.

(b) Xét một họ $\Omega$ gồm $k$ tập con có 7 phần tử của $X$. Một tập $A \subset X$ được gọi là một cận trên của $\Omega$ nếu như $|A|=8$ và tồn tại một tập con $F$ của họ $\Omega$ sao cho $F \subset A$. Gọi $d$ là số tập con cận trên của họ $\Omega$. Chứng minh rằng $d \geq \frac{3}{2} k$.

Bài 7. Cho tam giác $A B C$ không cân. Gọi $I$ là trung điểm $B C$. Đường tròn $(I)$ tâm $I$ đi qua $A$ cắt $A B, A C$ lần lượt tại $M, N$. Giả sử $M I, N I$ cắt $(I)$ tại $P, Q$. Gọi $K$ là giao điểm của $P Q$ với tiếp tuyến tại $A$ của $(I)$. Chứng minh rằng $K$ thuộc đường thẳng $B C$.

Bài 8. Tìm số nguyên dương $n$ lớn nhất thỏa mãn các điều kiện sau:

$\quad (i)\quad n$ không chia hết cho 3 .

$\quad (ii)$ Bảng vuông $n \times n$ không thể được phủ kín bằng 1 quân tetramino $1 \times 4$ và các quân trimino $1 \times 3$. Trong phép phủ, các quân tetramino và trimino được phép quay dọc nhưng không được phép chườm lên nhau hoặc nằm ra ngoài bảng vuông.

LỜI GIẢI

Bài 1. Cho $a, b, c>0$ thỏa mãn điều kiện $(a+1)(b+1)(c+1)=1+4 a b c$. Chứng minh rằng ta có bất đẳng thức
$\quad\quad\quad\quad\quad\quad\quad\quad\quad\quad a+b+c \leq 1+a b c .$
Lời giải. Điều kiện đã cho viết thành $a b+b c+c a+a+b+c=3 a b c$. Chia hai vế cho $a b c$ rồi đặt $a=\frac{1}{x}, b=\frac{1}{y}, c=\frac{1}{z}$, ta có $x y+y z+z x+x+y+z=3$.
Bất đẳng thức đã cho có thể viết thành
$\quad\quad\quad\quad\quad\quad x y+y z+z x-x y z \leq 1 \text { hay } x+y+z+x y z \geq 2 \text {. }$
Theo bất đẳng thức Schur thì
$\quad\quad\quad\quad\quad\quad (x+y+z)^3+9 x y z \geq 4(x y+y z+z x)(x+y+z) .$
Đặt $m=x+y+z, n=x y+y z+z x$ thì $m+n=3$ và
$\quad\quad\quad\quad\quad\quad\quad\quad\quad\quad x y z \geq \frac{4 m n-m^3}{9} .$
Ta sẽ chứng minh rằng
$\quad\quad\quad\quad\quad\quad m+\frac{4 m n-m^3}{9} \geq 2 \Leftrightarrow m^3+4 m^2-21 m+18 \leq 0$
hay $(m-2)\left(m^2+6 m-9\right) \leq 0$. Chú ý rằng $m^2 \geq 3 n$ nên
$\quad\quad\quad\quad\quad\quad\quad\quad m^2 \geq 3(3-m) \Leftrightarrow m^3+3 m \geq 9$

Do đó $m^2+6 m-9 \geq 0$. Ta xét các trường hợp

  1. Nếu $m>2$ thì $x+y+z>2$ nên hiển nhiên bất đẳng thức cần chứng minh là đúng.
  2. Nếu $m \leq 2$ thì $m-2 \leq 0$ nên ta cũng có $(m-2)\left(m^2+6 m-9\right) \leq 0$.

Vậy trong mọi trường hợp, ta luôn có điều phải chứng minh.

Bài 2. Cho tập hợp $A=[n^3-4 n+15 \mid n \in \mathbb{N}]$. Tìm tất cả các phần tử $a \in A$ thỏa mãn đồng thời hai điều kiện sau đây:

$\quad (i)\quad a$ là số chẵn.

$\quad (ii)$ Nếu $a_1, a_2$ là các ước số của $\frac{a}{2}$ với $a_1, a_2>1$ thì $\operatorname{gcd}\left(a_1, a_2\right)>1$.

Lời giải. Ta thấy rằng $a=n^3-4 n+15$ chẵn nên $n^3+15$ chẵn hay $n$ lẻ. Đặt $n=2 k+1$ với $k \in \mathbb{N}$. Ta có

$\quad\quad\quad\quad\quad\quad a=n^3-4 n+15 =(n+3)\left(n^3-3 n+15\right) $

$\quad\quad\quad\quad\quad\quad\quad\quad\quad\quad\quad\quad\quad\quad =(2 k+4)\left(4 k^2-2 k+3\right)$

nên $\frac{a}{2}=(k+2)\left(4 k^2-2 k+3\right)$. Điều kiện ii) cho thấy rằng $\frac{a}{2}$ phải là lũy thừa của một số nguyên tố, vì nếu nó có hai ước nguyên tố trở lên, đặt là $p, q$ thì chọn $x=p, y=q$, ta có $x, y>1$ nhưng $\operatorname{gcd}(x, y)=1$, không thỏa mãn.

Vì $\left(4 k^2-2 k+3\right)-(k+2)=4 k^2-3 k+1>0$ với mọi $k \in \mathbb{N}$. Do đó, ta phải có $k+2 \mid 4 k^2-2 k+3$. Suy ra

$\quad\quad\quad\quad\quad\quad\quad\quad\quad\quad\frac{4 k^2-2 k+3}{k+2}=4 k-10+\frac{23}{k+2} \in \mathbb{Z} .$

Do đó $k+2 \in{1,23}$ vì $k+2>0$. Ta xét các trường hợp

  1. Nếu $k+2=1$ thì $k=-1$ hay $n=2 k+1=-1<0$, không thỏa mãn.
  2. Nếu $k+2=23$ thì $k=21$ hay $n=43$, tính được $\frac{a}{2}=3 \cdot 5^2 \cdot 23^2$, cũng không thỏa mãn.

Vậy không tồn tại số $a$ nào thỏa mãn.

Bài 3. Tìm tất cả các hàm số $f: \mathbb{N}^* \rightarrow \mathbb{N}^*$ thỏa mãn:

$\quad\quad\quad\quad\quad\quad\quad\quad\quad\quad f\left(\frac{f(n)}{n}\right)=n^2 \forall n \in \mathbb{N}^* .$

Lời giải. Với $n \in \mathbb{N} *$, ta thấy rằng nếu $n=1$ thì $f(f(1))=1$.

Nếu $n>1$ thì gọi $p$ là một ước nguyên tố bất kỳ của $n$.

Vì $\frac{f(n)}{n} \in \mathbb{N} *$ nên $n \mid f(n)$. Đặt $a=v_p(n), b=v_p(f(n))$ thì trước hết, ta có $a \leq b$.

Từ $f\left(\frac{f(n)}{n}\right)=n^2$, ta suy ra rằng $\frac{f(n)}{n} \mid n^2$ hay $f(n) \mid n^3$, tức là $b \leq 3 a$.

Trong biểu thức đã cho, thay $n \rightarrow \frac{f(n)}{n}$ thì

$\quad\quad\quad\quad\quad\quad f\left(\frac{f\left(\frac{f(n)}{n}\right)}{\frac{f(n)}{n}}\right)=\left(\frac{f(n)}{n}\right)^2 \Leftrightarrow f\left(\frac{n^3}{f(n)}\right)=\left(\frac{f(n)}{n}\right)^2$

Do đó, ta phải có

$\quad\quad\quad\quad\quad\quad\quad\quad\quad\left(\frac{f(n)}{n}\right)^2\left|\frac{n^3}{f(n)} \Leftrightarrow f^3(n)\right| n^5 \text { nên } 3 b \leq 5 a \text {. }$

Sau đó lại tiếp tục thay $n$ trong biểu thức đã cho bởi $\frac{n^5}{f^3(n)}$ và cứ như thế, ta xây dựng được hai dãy hệ số của $a, b$ như sau

$\quad\quad\quad\quad\quad\quad\quad\quad\quad\quad u_0=v_0=1, u_1=3, v_1=1 \text { và } $

$\quad\quad\quad\quad\quad\quad\quad\quad u_{k+1}=2 u_{k-1}+v_k, v_{k+1}=2 v_{k-1}+u_k \text { với } k \geq 1 .$

Khi đó

$\quad\quad\quad\quad\quad\quad\quad\quad\quad\quad\quad\quad\frac{v_{2 k}}{u_{2 k}} \leq \frac{b}{a} \leq \frac{u_{2 k+1}}{v_{2 k+1}} .$

Biến đổi công thức của hai dãy, ta có $u_{n+2}=5 u_n-4 u_{n-2}, v_{n+2}=5 v_n-4 v_{n-2}$ và cả hai dãy đều có phương trình đạạc trưng là $t^2-5 t+4=0$. Ngoài ra, dãy chăn và dãy lẻ trong mỗi dãy đều độc lập với nhau.

Ta có $u_0=1, u_2=3, v_0=1, v_2=5$ nên

$\quad\quad\quad\quad\quad\quad\quad\quad u_{2 k}=\frac{13+2 \cdot 16^k}{15}, v_{2 k}=\frac{11+4 \cdot 16^k}{15}, k \geq 1 .$

Từ đó, dễ dàng tính được $\lim \frac{u_{2 k+1}}{v_{2 k+1}}=2$.

Một cách tương tự, ta tính được $\lim \frac{u_{2 k}}{v_{2 k}}=\frac{1}{2}$. Do đó, số $\frac{b}{a}$ bị kẹp ở giữa và là số nguyên nên chỉ có thể là $\frac{b}{a}=2 \Leftrightarrow b=2 a$.

Rõ ràng tập hợp ước nguyên tố của $n$ và $f(n)$ là giống nhau. Hơn nữa, với một ước nguyên tố cụ thể thì số mũ trong $f(n)$ gấp đôi số mũ trong $n$. Suy ra $f(n)=$ $n^2, \forall n>1$.

Tiếp theo, giả sử $f(1)=n>1$ thì ta có $f(f(1))=1$ nên $f(n)=1$, mâu thuẫn. Vì thế nên chỉ có thể $f(1)=1$.

Vậy tất cả các hàm thỏa mãn là $f(n)=n^2, \forall n \in \mathbb{N}^*$.

Bài 4. Cho tam giác $A B C$ nội tiếp $(O)$, có $B, C$ cố định và $A$ thay đổi trên $(O)$. Ký hiệu $(I)$ là đường tròn nội tiếp tam giác $A B C$. Gọi $\left(O_1\right)$ là đường tròn qua $A, B$ và tiếp xúc với đường tròn $(I)$ tại $E$. Gọi $\left(O_2\right)$ là đường tròn qua $A, C$ và tiếp xúc với đường tròn $(I)$ tại $F$. Đường phân giác trong của góc $\angle A E B$ cắt $\left(O_1\right)$ tại $M$ và đường phân giác trong của góc $\angle A F C$ cắt $\left(O_2\right)$ tại $N$.

(a) Chứng minh rằng tứ giác $E F M N$ nội tiếp.

(b) Gọi $J$ là giao điểm của $E M$ và $F N$. Chứng minh rằng đường thẳng $I J$ luôn đi qua một điểm cố định.

Lời giải. (a) Trước hết, ta thấy rằng $O_1, I, E$ thẳng hàng và $O_2, I, F$ thẳng hàng. Vì $M$ là trung điểm cung $A B$ của $\left(O_1\right)$ nên $O_1 M$ là trung trực của $A B$, suy ra $O \in O_1 M$. Tương tự, ta cũng có $O \in O_1 N$.

Gọi $P, Q$ lần lượt là tiếp điểm của $(I)$ với $A B, A C$.

Vì $I P | O_1 M$ (cùng vuông góc với $A B$ ) nên $\angle M O_1 E=\angle P I E$. Hơn nữa, các tam giác $O_1 M E, I P E$ đều cân với đỉnh là $O_1, I$ nên suy ra chúng đồng dạng, tức là $\angle I E P=\angle O_1 E M$ hay $E, P, M$ thẳng hàng. Tương tự thì $F, Q, N$ cũng thẳng hàng.

Vì ta đã có $E, F, P, Q$ cùng thuộc đường tròn $(I)$ nên để có $E, F, M, N$ cùng thuộc một đường tròn thì $\angle E M N=\angle E F N=\angle E P Q$ hay $M N | P Q$.

Mặt khác, $A I \perp P Q$ nên ta cần có $A I \perp M N$.

Thật vậy, sử dụng phương tích với đường tròn $(I)$ ta có

$\quad\quad\quad\quad\quad\quad M A^2-N A^2=M P \cdot M E-N Q \cdot N F=M I^2-N I^2$

nên theo định lý bốn điểm thì $A I \perp M N$, từ đó ta có điều phải chứng minh.

(b) Vì $P Q|M N, O M| I P$ nên dễ dàng có $\angle I P Q=\angle O M N$. Tương tự $\angle I P Q=$ $\angle O N M$.

Do đó, hai tam giác $I P Q, O M N$ đồng dạng với nhau, tức là

$\quad\quad\quad\quad\quad\quad\quad\quad\quad\quad\quad\quad\frac{I P}{O M}=\frac{P Q}{M N} \text {. }$

Ngoài ra,

$\quad\quad\quad\quad\quad\quad\quad\quad\quad\quad\quad\quad\frac{J P}{J M}=\frac{I P}{O M}$

kết hợp với $\angle J P I=\angle J M O$, ta có hai tam giác $J P I, J M O$ đồng dạng, dẫn đến

$\quad\quad\quad\quad\quad\quad\quad\quad\quad\quad\quad\quad\angle P J I=\angle M J O \text {. }$

Từ đây suy ra $I, J, O$ thẳng hàng hay $I J$ luôn đi qua điểm $O$ cố định.

Ngày thi thứ hai

Bài 5. Cho dãy số $\left(x_n\right)$ bởi $x_0=1, x_1=2014$ và $x_{n+1}=\sqrt[3]{x_n x_{n-1}^2} \forall n \in \mathbb{N}^*$.

(a) Chứng minh rằng dãy số $\left(x_n\right)$ có giới hạn hữu hạn và tìm giới hạn đó.

(b) Với mỗi $n \geq 2$, hãy tìm số nguyên dương $k$ nhỏ nhất sao cho $a=x_n^k$ là một số nguyên. Chứng minh rằng khi đó $a$ không thể viết được dưới dạng tổng các lũy thừa bậc ba của hai số tự nhiên.

Lời giải. (a) Đặt $u_n=\log _{2014}\left(x_n\right)$ thì ta thu được dãy $\left(u_n\right)$ như sau

$\quad\quad\quad\quad\quad\quad\quad\quad\quad\quad\left\{\begin{array}{c}u_0=0, u_1=1 \\ u_{n+1}=\frac{1}{3} u_n+\frac{2}{3} u_{n-1}\end{array}\right.$

Từ đó tìm được

$\quad\quad\quad\quad\quad\quad\quad\quad\quad\quad u_n=\frac{3}{5}-\frac{3}{5} \cdot\left(\frac{-2}{3}\right)^n$

Suy ra $\lim _{n \rightarrow+\infty} u_n=\frac{3}{5}$ nên ta có được

$\quad\quad\quad\quad\quad\quad\quad\quad \lim _{n \rightarrow+\infty} x_n=\lim _{n \rightarrow+\infty}\left(2014^{u_n}\right)=2014^{3 / 5}$

(b) Ta thấy rằng để có $\left(x_n\right)^k$ là một số nguyên thì $\frac{3 k\left(3^n-(-2)^n\right)}{5 \cdot 3^n} \in \mathbb{Z}$ nguyên. Ta xét các trường hợp

  1. Nếu $n$ lẻ thì $3^n-(-2)^n=3^n+2^n: 5$. Vì $\operatorname{gcd}\left(\frac{3^n+2^n}{5}, 3^n\right)=1$ nên ta được $3^n \mid 3 k$ nên $k$ nhỏ nhất thỏa mãn điều này là $k=3^{n-1}$.
  2. Nếu $n$ chẵn thì $3^n-2^n \equiv(-2)^n-2^n=0(\bmod 5)$ và tương tự, ta cũng tìm được $k=3^{n-1}$.

Do đó số $k$ nhỏ nhất cần tìm là $k=3^{n-1}$. Tiếp theo, ta sẽ chứng minh rằng phương trình sau không có nghiệm tự nhiên

$\quad\quad\quad\quad\quad\quad\quad a^3+b^3=2014^n \Leftrightarrow(a+b)\left(a^2-a b+b^2\right)=2014^n$

Gọi $n_0$ số nguyên dương nhỏ nhất sao cho tồn tại $a, b \in \mathbb{Z}^{+}$để $a^3+b^3=2014^{n_0}$. Dễ thấy $n_0=1$ không thỏa mãn nên ta chỉ xét $n_0 \geq 2$. Ta xét các trường hợp

  1. Nếu $\operatorname{gcd}\left(a+b, a^2-a b+b^2\right)=1$ thì dễ thấy $(a-b)^2 \geq 1$. Khi đó

$\quad\quad\quad\quad\quad\quad\quad\quad\quad a^2-a b+b^2 \geq a+b>\sqrt{a^2-a b+b^2} .$

Vì $2014=2 \cdot 19 \cdot 53$ nên chỉ có thể xảy ra

$\quad\quad\quad\quad\quad\quad\quad\quad\quad\quad a+b=19^{n_0}, a^2-a b+b^2=106^{n_0} .$

Ngoài ra $(a+b)^2 \leq 4\left(a^2-a b+b^2\right)$ nên ta phải có $361^{n_0} \leq 4 \cdot 106^{n_0}$. Đánh giá này sai khi $n_0 \geq 2$ nên trường hợp này không thỏa mãn.

$2$. Nếu $\operatorname{gcd}\left(a+b, a^2-a b+b^2\right)>1$ thì chẳng hạn $a+b=2^x u, a^2-a b+b^2=2^y v$ với $\quad\quad\quad\quad\quad\quad\quad\quad\operatorname{gcd}(u, 2)=\operatorname{gcd}(v, 2)=1 .$

Các trường hợp còn lại chứng minh tương tự. Ngoài ra

$\quad\quad\quad\quad\quad\quad\quad\quad\quad\quad u v=1007^{n_0}, x+y=n_0 .$

Chú ý rằng $(a+b)^2-\left(a^2-a b+b^2\right)=3 a b$ nên $3 a b$ cũng chẵn, tức là cả hai số $a, b$ đều chẵn (vì nếu không thì $a^3+b^3$ lẻ).

Từ đây dễ dàng chứng minh được $3 v_2(a)=3 v_2(b)=n_0$, ta đưa về $x^{\prime 3}+y^{\prime 3}=$ $1007^{n_0}$. Cứ như thế, ta được 2014 $|a, 2014| b$ nên phương trình sau cũng có nghiệm nguyên dương

$\quad\quad\quad\quad\quad\quad\quad\quad\quad\quad \left(\frac{a}{2014}\right)^3+\left(\frac{b}{2014}\right)^3=2014^{n_0-3} .$

Điều này mâu thuẫn với các chọn $n_0$ nên phương trình trên vô nghiệm. Các trường hợp còn lại tương tự.

Ta có điều phải chứng minh.

Bài 6. Cho $X$ là tập hợp gồm 19 phần tử.

(a) Chứng minh rằng tồn tại ít nhất 2600 tập con 7 phần tử của $X$ sao cho với hai tập con $A, B$ bất kỳ trong số 2600 tập con đó, ta có $|A \cap B| \leq 5$.

(b) Xét một họ $\Omega$ gồm $k$ tập con có 7 phần tử của $X$. Một tập $A \subset X$ được gọi là một cận trên của $\Omega$ nếu như $|A|=8$ và tồn tại một tập con $F$ của họ $\Omega$ sao cho $F \subset A$. Gọi $d$ là số tập con cận trên của họ $\Omega$. Chứng minh rằng $d \geq \frac{3}{2} k$.

Lời giải. (a) Không mất tính tổng quát, ta có thể giả sử $X$ là tập hợp 19 số nguyên dương đầu tiên. Gọi $X(k)$ là tập hợp tất cả các tập con có 7 phần tử của $X$ và tổng các phần tử của nó chia 19 dư $k$.

Khi đó, dễ thấy rằng $|X(0)|+|X(1)|+\cdots+|X(18)|$ chính là số tập con có 7 phần tử tùy ý của $X$ và là $C_{19}^7$.

Ta thấy rằng hai tập hợp $A, B \in X(k)$ tùy ý đều thỏa mãn đề bài.

Thật vậy,

Giả sử $|A \cap B|=6$ (không thể có $|A \cap B|=7$ vì khi đó hai tập hợp trùng nhau).

Đặt $A=[a_1, a_2, a_3, a_4, a_5, a_6, x]$, $B=[a_1, a_2, a_3, a_4, a_5, a_6, y]$ thì

$\quad\quad\quad\quad\quad\quad\quad\quad\quad\quad \sum_{i=1}^6 a_i+x \equiv \sum_{i=1}^6 a_i+y \equiv k \quad(\bmod 19)$

nên $x \equiv y(\bmod 19)$. Suy ra $x=y$, mâu thuẫn. Đến đây, dễ thấy rằng

Ta có điều phải chứng minh.

(b) Xét một tập hợp $F$ thuộc họ $\Omega$. Vì $|X \backslash F|=19-7=12$ nên có tất cả 12 tập hợp $A \subset X$ với $|A|=8$ và $F \subset A$.

Ngược lại, ứng với một tập hợp $A$ là một cận trên của họ $\Omega$, có không quá 8 tập $F$ trong họ $\Omega$ sao cho $F \subset A$. Do đó $d \geq \frac{12}{8} k$ hay $d \geq \frac{3}{2} k$.

Đẳng thức xảy ra khi họ $\Omega$ là tập hợp tất cả các tập con có 7 phần tử của $X$.

Bài 7. Cho tam giác $A B C$ không cân. Gọi $I$ là trung điểm $B C$. Đường tròn (I) tâm $I$ đi qua $A$ cắt $A B, A C$ lần lượt tại $M, N$. Giả sử $M I, N I$ cắt $(I)$ tại $P, Q$. Gọi $K$ là giao điểm của $P Q$ với tiếp tuyến tại $A$ của $(I)$. Chứng minh rằng $K$ thuộc đường thẳng $B C$.

Lời giải. Không mất tính tổng quát, giả sử $A B<A C$.

Kẻ đường kính $A J$ của đường tròn $(I)$. Khi đó, dễ thấy tứ giác $A B J C$ và $A N J Q$ là các hình bình hành nên $J B|A C, J Q| A N$ dẫn đến $J, Q, B$ thẳng hàng. Tương tự $J, P, C$ thẳng hàng.

Gọi $H$ là hình chiếu của $A$ lên $B C$ thì tứ giác $A Q B H$ nội tiếp.

Suy ra

$\quad\quad\quad\quad\quad\quad \angle Q H B=\angle Q A B=\angle Q A M=\angle Q P M=\angle Q P I$

nên tứ giác $P Q H I$ cũng nội tiếp. Gọi $(O)$ là đường tròn ngoại tiếp tam giác $A B C$ thì dễ thấy đường tròn $(A H I)$ tiếp xúc với $(O)$ tại $A$.

Xét ba đường tròn $(O),(A H I),(P Q H I)$ thì

  • Trục đẳng phương của $(O),(A H I)$ là tiếp tuyến của $(O)$ tại $A$.
  • Trục đẳng phương của $(O),(P Q H I)$ là $P Q$.
  • Trục đẳng phương của $(P Q H I),(A H I)$ là $H I$.

Do đó, $K$ chính là tâm đẳng phương của ba đường tròn nên $K \in H I$ hay $K, B, C$ thẳng hàng.

Bài 8. Tìm số nguyên dương $n$ lớn nhất thỏa mãn các điều kiện sau:

$\quad (i)$ n không chia hết cho 3 .

$\quad (ii)$ Bảng vuông $n \times n$ không thể được phủ kín bằng 1 quân tetramino $1 \times 4$ và các quân trimino $1 \times 3$. Trong phép phủ, các quân tetramino và trimino được phép quay dọc nhưng không được phép chườm lên nhau hoặc nằm ra ngoài bảng vuông.

Lời giải. Ta sẽ chứng minh $n=5$ là giá trị lớn nhất cần tìm.

Ta nhận thấy rằng nếu $n=3 k+1, k \geq 1$ thì ta luôn phủ được bảng vuông $n \times n$ bằng cách phủ hàng đầu tiên bằng 1 quân tetramino kích thước $1 \times 4$ (ta sẽ gọi tắt là tetramino) và $k-1$ quân trimino kích thước $1 \times 3$ (ta sẽ gọi tắt là trimino). Các cột còn lại có chiều dài $3 k$ có thể phủ được bằng các quân trimino (xoay dọc lại).

Ta chứng minh rằng nếu $n=3 k+2, k \geq 2$ thì bảng vuông $n \times n$ cũng phủ được. Cách phủ với $n=8$ được minh họa như sau

Dễ dàng thấy rằng với $k \geq 3$ thì ta có thể thu được cách phủ cho bảng vuông $n \times n$ bằng cách phủ phần hình vuông $8 \times 8$ ở góc trên bên trái như trên, phần còn lại gồm 1 hình chữ nhật kích thước $3(k-2) \times(3 k+2)$ và 1 hình chữ nhật kích thước $8 \times 3(k-2)$ phủ được bằng các quân trimino.

Bây giờ ta chứng minh bảng vuông $5 \times 5$ không thể phủ được bằng 1 quân tetramino và 7 quân trimino.

Trước hết ta chứng minh bổ đề: Nếu bảng vuông $5 \times 5$ có thể phủ được bằng một hình vuông $1 \times 1$, ta gọi là unomino và 8 quân trimino thì quân unomino $1 \times 1$ phải phủ ô trung tâm.

Thật vậy,

Ta đánh số các ô của bảng vuông $5 \times 5$ như hình vẽ

Ta thấy rằng một quân trimino luôn phủ đúng một ô mang số 1 , một ô mang số 2 và một ô mang số 3 . Vì số các số 2 bằng 9 , còn số các số 1 và 3 bằng 8 nên nếu phép phủ ở đề bài thực hiện được thì quân unomino phải phủ một ô mang số 2 .

Mặt khác, ta có thể đánh số bảng vuông $5 \times 5$ bằng một cách khác

Các tính chất nói ở trên vẫn đúng cho cách đánh số này, tuy nhiên ở đây số số 1 là 9 , còn số số 2 và 3 là 8 . Do đó, một lần nữa ta kết luận quân unomino phải phủ một ô mang số 1 .

Giao hai điều kiện cần nói trên lại, ta thấy với một cách phủ hợp lệ thì quân unomino phải phủ ô trung tâm.

Quay trở lại với vấn đề phủ bảng vuông $5 \times 5$ bằng 1 quân tetramino và 7 quân trimino. Nếu tồn tại một cách phủ như thế thì cắt quân tetramino thành 1 quân unomino và 1 quân trimino, ta thu được một phép phủ bảng vuông $5 \times 5$ bằng 1 quân unomino và 8 quân trimino.

Theo bổ đề thì quân unomino phải nằm ở ô trung tâm, nghĩa là một đầu của quân tetramino phải nằm ở ô trung tâm, mâu thuẫn (vì khi đó quân tetramino sẽ bị lòi ra ngoài bảng vuông).

Với những lý luận ở trên, ta kết luận $n=5$ là giá trị lớn nhất cần tìm.

 

 

 

 

 

 

 

 

 

 

 

 

 

 

 

 

 

 

 

 

 

 

 

 

 

 

 

 

 

 

 

 

 

 

 

 

 

 

 

 

 

 

 

 

 

 

 

 

 

 

 

 

 

 

 

 

 

 

 

 

 

 

 

 

 

 

 

 

 

 

 

 

 

 

 

 

 

 

 

 

 

 

 

 

 

Đề thi và đáp án kì thi chọn đội tuyển thi Quốc gia trường Phổ thông Năng khiếu năm học 2013 – 2014

ĐỀ THI

Ngày thi thứ nhất

Bài 1. Tìm tất cả các hàm số $f: \mathbb{R} \rightarrow \mathbb{R}$ thoả mãn:

$\quad\quad\quad\quad\quad\quad\quad\quad\quad\quad f\left(x^3+y+f(y)\right)=2 y+x^2 f(x) \forall x, y \in \mathbb{R}$

Bài 2. Cho dãy số $\left(u_n\right)$ thoả mãn $u_1=2013, u_{n+1}=u_n^3-4 u_n^2+5 u_n \forall n \in \mathbb{N}^*$. Tìm tất cả các số nguyên tố $p$ là ước của $u_{2014}+2009$ và $p \equiv 3(\bmod 4)$.

Bài 3. Trong một hội nghị khoa học có 5000 đại biểu tham dự, trong đó có một nhóm gồm 280 người là ban tổ chức. Giả sử rằng mỗi thành viên của ban tổ chức thì quen tất cả thành viên khác của hội nghị. Một uỷ ban gồm một số đại biểu được gọi là uỷ ban làm việc nếu tất cả các thành viên trong đó đều quen nhau, và được gọi là uỷ ban thách thức nếu không có hai thành viên nào của uỷ ban quen nhau. Chứng minh rằng ta có thể phân hoạch các đại biểu (kể cả ban tổ chức) thành các ủy ban mà mỗi ủy ban gồm 5 đại biểu, sao cho các uỷ ban này hoặc là làm việc, hoặc là thách thức.

Bài 4. Cho tam giác $A B C$ có $B, C$ cố định còn $A$ di động sao cho $A B=A C$ và $\angle B A C>60^{\circ}$. Đường thẳng đối xứng với $B C$ qua $A B$ cắt $A C$ tại $P$. Trên đoạn $P C$ lấy điểm $M$ sao cho $P M=P B$. Gọi $N$ là giao điểm của $A B$ và phân giác ngoài của góc $\angle B C A$. Chứng minh rằng $M N$ luôn đi qua một điểm cố định.

Ngày thi thứ hai

Bài 5. Cho 2014 số thực $x_1, x_2, \ldots, x_{2014}$ thoả mãn các điều kiện:

(i) $x_1+x_2+\cdots+x_{2014}=0$,

(ii) $x_1^2+x_2^2+\cdots+x_{2014}^2=2014$.

Tìm giá trị lớn nhất của biểu thức $P=x_1 x_2 \cdots x_{2014}$.

Bài 6. Cho dãy số $\left(u_n\right)$ xác định bởi $u_1=1, u_{n+1}=\frac{u_n}{\sqrt{u_n^2+1}+\sqrt{2}}, \forall n \in \mathbb{N}^*$. Tính $\lim \frac{u_{n+1}}{u_n}$

Bài 7. Cho $n$ là số nguyên dương và $A$ là tập con khác rỗng của $X={1,2, \ldots, n}$.

(a) Tính giá trị của tổng $S(A)=\sum_{E \subset X}(-1)^{|E \cap A|}$ trong đó $E$ lấy trên tất cả các tập hợp con của $X$, kể cả tập hợp rỗng.

(b) Cho $m \in \mathbb{N}^*$. Xét $m$ tập con khác rỗng của $X$ là $A_1, A_2, \ldots, A_m$ và $m$ số nguyên khác 0 là $a_1, a_2, \ldots, a_m$ sao cho $a_1+a_2+\cdots+a_m<0$. Chứng minh rằng tồn tại $E \subset X$ sao cho $\sum_{i=1}^m(-1)^{\left|E \cap A_i\right|} a_i>0$.

Bài 8. Cho tam giác $A B C$ nhọn có $H$ là trực tâm. $P$ là điểm di động bên trong tam giác $A B C$ sao cho $\angle B P C=\angle B H C$. Đường thẳng qua $B$ vuông góc $A B$ cắt $P C$ tại $M$, đường thẳng qua $C$ vuông góc $A C$ cắt $P B$ tại $N$. Gọi $I$ là trung điểm của $M N$. Chứng minh rằng $I$ luôn thuộc một đường thẳng cố định.

 

LỜI GIẢI

Ngày thi thứ nhất

Bài 1 . Tìm tất cả các hàm số $f: \mathbb{R} \rightarrow \mathbb{R}$ thoả mãn:

$\quad\quad\quad\quad\quad\quad f\left(x^3+y+f(y)\right)=2 y+x^2 f(x), \forall x, y \in \mathbb{R} .$

Lời giải. Trong phương trình đã cho, thay $x=y=0$, ta có $f(f(0))=0$.

Lại thay $y=0$ thì

$\quad\quad\quad\quad\quad\quad\quad\quad\quad\quad f\left(f^3+f(0)\right)=x^2 f(x), \forall x .$

Thay $y=f(0)$ thì

$\quad\quad\quad\quad\quad\quad\quad\quad\quad\quad f\left(x^3+f(0)\right)=2 f(0)+x^2 f(x) .$

Từ đây suy ra $f(0)=0$. Thay $y=0$ vào đẳng thức đã cho ta được $f\left(x^3\right)=x^2 f(x)$. Do đó ta có

$\quad\quad\quad f\left(x^3+y+f(y)\right)=2 y+f\left(x^3\right) \text { hay } f(x+y+f(y))=2 y+f(x)\quad\quad\quad(*).$

Thay $y$ bởi $-y$, ta được

$\quad\quad\quad\quad\quad\quad\quad\quad\quad\quad f(x-y+f(-y))=-2 y+f(x) .$

Với $x$ bất kì, ta lấy $2 y=f(x)$ ta được $f(x-y+f(-y))=0$ suy ra $x-y+f(-y)=0$. Do đó, ta được $f(-x)=f(-y+f(-y))=-2 y=-f(x)$. Từ đây suy ra

$\quad\quad\quad\quad\quad\quad\quad\quad\quad\quad f(x+f(y)+f(f(y)))=2 f(y)+f(x)$

$\quad\quad\quad\quad\quad\quad\quad\quad\quad\quad f(x+f(y)+f(f(y)))=2 f(y)+f(x) .$

Trong $(*)$ thay $x=-y$ ta được $f(f(y))=2 y+f(-y)=2 y-f(y)$, kết hợp với đẳng thức trên, ta được

$f(x+2 y)=2 f(y)+f(x) .$

Đến đây cho $x=0$ ta được $f(2 y)=2 f(y)$ nên ta được $f(x+y)=f(x)+f(y)$, tức là $f(x)$ cộng tính. Đến đây ta sẽ tính $f\left((x+1)^3+(x-1)^3\right)$ theo hai cách như sau

  • $f\left((x+1)^3+(x-1)^3\right)=f\left(2 x^3+6 x\right)=2 x^2 f(x)+6 f(x)$.
  • $f\left((x+1)^3+(x-1)^3\right)=(x+1)^2 f(x+1)+(x-1)^2 f(x-1)=(x+1)^2(f(x)+$ $f(1))+(x-1)^2(f(x)-f(1))=2 x^2 f(x)+2 f(x)+4 x f(1)$.

So sánh hai đẳng thức trên, ta được $f(x)=x f(1)=a x$ với mọi $x$. Thử lại ta được $a=1, a=-2$.

Vậy các hàm cần tìm là $f(x)=x, f(x)=-2 x$.

Bài 2. Cho dãy số $\left(u_n\right)$ thoả mãn $u_1=2013, u_{n+1}=u_n^3-4 u_n^2+5 u_n \forall n \in \mathbb{N}^*$.

Tìm tất cả các số nguyên tố $p$ là ước của $u_{2014}+2009$ và $p \equiv 3(\bmod 4)$.

Lời giải. Ta có

$\quad\quad\quad\quad\quad\quad\quad\quad u_{n+1}-2 =\left(u_n-2\right)\left(u_{n-1}-1\right)^2 $

$\quad\quad\quad\quad\quad\quad\quad\quad\quad\quad\quad\quad =\left(u_{n-2}-1\right)^2\left(u_{n-1}-1\right)^2\left(u_{n-2}-2\right) $

$\quad\quad\quad\quad\quad\quad\quad\quad\quad\quad\quad\quad =\left(u_{n-1}-1\right)^2\left(u_{n-2}-1\right)^2 \cdots\left(u_2-1\right)^2\left(u_1-2\right) .$

Do đó

$\quad\quad\quad\quad\quad u_{2014}+2009=2011\left[\left(u_{2013}-1\right)^2\left(u_{2012}-1\right)^2 \cdots\left(u_2-1\right)^2+1\right] .$

Gọi $B$ là biểu thức trong dấu ngoặc vuông thứ hai. Ta có bổ đề quen thuộc là nếu $a^2+b^2$ chia hết cho số nguyên tố $p=4 k+3$ thì $a, b$ cùng chia hết cho $p$. Từ đây suy ra số $B$ có dạng $a^2+1$ nên nó sẽ không có ước nguyên tố dạng $4 k+3$.

Vậy $u_{2014}+9$ chỉ có một ước nguyên tố $p \equiv 3(\bmod 4)$ duy nhất là 2011 .

Bài 3. Trong một hội nghị khoa học có 5000 đại biểu tham dự, trong đó có một nhóm gồm 280 người là ban tổ chức. Giả sử rằng mỗi thành viên của ban tổ chức thì quen tất cả thành viên khác của hội nghị. Một uỷ ban gồm một số đại biểu được gọi là uỷ ban làm việc nếu tất cả các thành viên trong đó đều quen nhau, và được gọi là uỷ ban thách thức nếu không có hai thành viên nào của uỷ ban quen nhau. Chứng minh rằng ta có thể phân hoạch các đại biểu (kể cả ban tổ chức) thành các ủy ban mà mỗi ủy ban gồm 5 đại biểu, sao cho các uỷ ban này hoặc là làm việc, hoặc là thách thức.

Lời giải. Trước hết, ta chứng minh bổ đề sau

ĐỊNH LÝ RAMSEY Với $s, t$ là các số nguyên dương, gọi $R(s, t)$ là số đỉnh ít nhất cần có của một graph để trong đó luôn tồn tại một tập độc lập $s$ đỉnh hoặc một graph con đầy đủ $t$ đỉnh. Khi đó

$\quad\quad\quad\quad\quad\quad\quad\quad\quad\quad\quad\quad R(s, t) \leq C_{s+t-2}^{s-1}\quad\quad\quad(*) .$

Chứng minh. Ta sẽ chứng minh rằng

$\quad\quad\quad\quad\quad\quad\quad\quad R(s, t) \leq R(s-1, t)+R(s, t-1) .$

Để ý rằng với $s=1$ hoặc $t=1$ thì $R(s, t)=1$. Do đó, nếu chứng minh được đánh giá này thì chỉ cần dùng tính chất của tam giác Pascal để có

$\quad\quad\quad\quad\quad\quad\quad\quad R(s, t) \leq C_{s+t-3}^{s-2}+C_{s+t-3}^{s-1}=C_{s+t-2}^{s-1} .$

Đặt $n$ là vế phải của $(*)$ và xét graph $G$ có $n$ đỉnh. Xét $v \in G$ thì

  1. Nếu như có ít nhất $R(s, t-1)$ đỉnh kề với $v$. Khi đó, theo định nghĩa thì trong tập đỉnh đó, sẽ luôn có một tập độc lập $s$ đỉnh hoặc một graph con đầy đủ $t-1$ đỉnh, ghép thêm đỉnh $v$ vào thì thỏa mãn điều kiện của $R(s, t)$.
  2. Nếu như có ít nhất $R(s-1, t)$ đỉnh không kề với $v$. Tương tự trên, trong tập đỉnh đó, cũng sẽ có một một graph con đầy đủ $t$ đỉnh hoặc tập độc lập $s-1$ đỉnh, ghép thêm đỉnh $v$ vào thì thỏa mãn điều kiện của $R(s, t)$.

Từ đó, ta thấy graph $G$ này thỏa mãn điều kiện của $R(s, t)$ nên theo tính nhỏ nhất thì $R(s, t) \leq n$.

Trở lại bài toán,

Xét graph đơn vô hướng $G=(V, E)$ đại diện cho hội nghị khoa học đã nêu, trong đó $V$ là tập hợp các đại biểu và hai đỉnh được nối nhau nếu hai đại biểu tương ứng quen nhau. Ta gọi $T$ là tập hợp đỉnh biểu diễn cho thành viên của ban tổ chức.

Khi đó một ủy ban gồm 5 thành viên là đại diện nếu như đó là một graph đầy đủ, còn đó là thách thức nếu đó là graph không có cạnh. Ta gọi các graph con như thế là graph con “chuẩn”.

Trong các đỉnh $V \backslash T$, ta xóa dần dần các graph con chuẩn đến khi không thực hiện được nữa. Ta gọi tập hợp còn lại là $S$. Ta sẽ chứng minh rằng $S \cup T$ có thể phân hoạch thành các graph con chuẩn như trên.

Theo định lý Ramsey, rõ ràng $|S| \leq C_8^4=70$. Xét một đỉnh $v \in S$ thì giả thiết, $v$ kề với cả 280 đỉnh của $T$ nên ta chọn ra trong đó 4 đỉnh để ghép với $v$ tạo thành một graph con “chuẩn”. Cứ như thế thực hiện cho đến hết các phần tử của $S$, còn lại bao nhiêu phần tử trong $T$ thì chia đều ra thành các graph con “chuẩn” là được.

Bài toán được giải quyết.

Nhận xét. Bài toán gốc của đề thi bị sai và có thể chỉ ra phản ví dụ. Nội dung cụ thể của nó như sau: Trong một hội nghi khoa học có 5000 đại biểu tham dư, mỗi một đại biểu biết ít nhất một thứ tiếng. Một uỷ ban gồm một số đại biểu được gọi là uỷ ban làm việc nếu tất cả các thành viên uỷ ban đều biết chung một thứ tiếng, và được gọi là uỷ ban thách thức nếu không có hai thành viên nào của uỷ ban biết chung một thứ tiếng. Uỷ ban có thể gồm 1 thành viên, khi đó gọi là uỷ ban làm việc hay thách thức cũng được. Chứng minh rằng ta có thể chia các đai biều thành đúng 100 uỷ ban, mỗi đại biểu thuộc đúng 1 uỷ ban, sao cho các uỷ ban này hoặc là uỷ ban làm việc, hoặc là uỷ ban thách thúc.

Ban biên tập đã bổ sung giả thiết như bài toán vừa giải ở trên và cố gắng giữ nguyên các phát biểu còn lại.

Bài 4. Cho tam giác $A B C$ có $B, C$ cố định còn $A$ di động sao cho $A B=A C$ và $\angle B A C>60^{\circ}$. Đường thẳng đối xứng với $B C$ qua $A B$ cắt $A C$ tại $P$. Trên đoạn $P C$ lấy điểm $M$ sao cho $P M=P B$. Gọi $N$ là giao điểm của $A B$ và phân giác ngoài của góc $\angle B C A$. Chứng minh rằng $M N$ luôn đi qua một điểm cố định.

Lời giải. Tam giác $P B M$ cân tại $P$ nên bằng biến đổi góc, ta có

$\quad\quad\quad\quad\quad \angle P B M=\angle P M B \Rightarrow 2 \angle A B C-\angle M B C=\angle A C B+\angle M B C .$

Do đó $\angle A B C=2 \angle M B C$ nên $B M$ là tia phân giác của $\angle A B C$. Theo tính chất đường phân giác thì

$\quad\quad\quad\quad\quad\quad\quad\quad\quad\quad\quad\quad \frac{M C}{M A}=\frac{B C}{B A}=\frac{B C}{A C}$

 

Lại có $C N$ là phân giác ngoài của $\angle A C B$ nên ta cũng có $\frac{N A}{N B}=\frac{C A}{C B}$. Gọi $I$ là trung điểm của $B C$ thì $I$ là điểm cố định.

Xét tam giác $A B C$ với $I$ thuộc $B C, M$ thuộc $A C$ và $N$ thuộc $A B$ với

$\quad\quad\quad\quad\quad\quad\quad\quad\quad\quad\quad\quad \frac{I B}{I C} \cdot \frac{M C}{M A} \cdot \frac{N A}{N B}=1 \cdot \frac{B C}{A C} \cdot \frac{A C}{B C}=1$

thì theo định lý Menelaus đảo, ta có $M, N, I$ thẳng hàng.

Vậy $M N$ luôn đi qua điểm $I$ cố định.

Ngày thi thứ hai

Bài 5. Cho 2014 số thực $x_1, x_2, \ldots, x_{2014}$ thoả mãn các điều kiện

$\quad (i) x_1+x_2+\cdots+x_{2014}=0$,

$\quad (ii) x_1^2+x_2^2+\cdots+x_{2014}^2=2014$.

Tìm giá trị lớn nhất của biểu thức $P=x_1 x_2 \cdots x_{2014}$.

Lời giải. Rõ ràng có thể chọn giá trị các biến thích hợp để $P>0$ nên để tìm giá trị lớn nhất của $P$ thì ta chỉ xét các số $x_1, x_2, \ldots, x_{2014}$ đều khác 0 và số các số âm là chẵn. Không mất tính tổng quát, giả sử $x_1 \geq x_2 \geq \ldots \geq x_{2 m}>0>x_{2 m+1} \geq$ $\ldots \geq x_{2014}$. Đổi dấu các số $y_k=-x_k>0$ với $2 m+1 \leq k \leq 2014$. Khi đó ta viết lại

$\quad\quad\quad\quad\quad\quad \left\{\begin{array}{l}x_1+x_2+\cdots+x_{2 m}=y_1+y_2+\cdots+y_{2 n}=A \\ x_1^2+x_2^2+\cdots+x_{2 m}^2+y_1^2+y_2^2+\cdots+y_{2 n}^2=2014\end{array}\right.$

trong đó $m+n=1007$ (ngoài ra $m, n>0$ vì các số đã cho không thể toàn bộ là dương hoặc toàn bộ là âm). Theo bất đẳng thức Cauchy-Schwarz thì

$\quad\quad\quad\quad\quad\quad\quad\quad\quad\quad\quad\quad 2014 \geq \frac{A^2}{2 m}+\frac{A^2}{2 n} \text { nên } A^2 \leq 4 m n$

Lại theo bất đẳng thức AM-GM thì

$\quad\quad\quad\quad\quad\quad\quad\quad\quad P =\left(x_1 x_2 \ldots x_{2 m}\right)\left(y_1 y_2 \ldots y_{2 n}\right) \leq\left(\frac{A}{2 m}\right)^{2 m}\left(\frac{A}{2 n}\right)^{2 n} $

$\quad\quad\quad\quad\quad\quad\quad\quad\quad\quad =\frac{A^{2 m+2 n}}{2^{2 m+2 n} m^{2 m} n^{2 n}} \leq \frac{(4 m n)^{m+n}}{2^{2 m+2 n} m^{2 m} n^{2 n}}=\left(\frac{m}{n}\right)^{n-m}$

Do $m, n$ khác tính chẵn lẻ nên với vai trò bình đẳng của $m, n$, ta có thể giả sử $m<n$ nên $n-m \geq 1$ và $m \leq 503$. Khi đó, áp dụng bất đẳng thức Bernoulli thì

$\quad\quad\quad \left(\frac{n}{m}\right)^{n-m} \geq 1+\left(\frac{n}{m}-1\right)(n-m)=1+\frac{(n-m)^2}{m} \geq 1+\frac{1}{503}=\frac{504}{503}.$

Suy ra $P \leq\left(\frac{m}{n}\right)^{n-m} \leq \frac{503}{504}$. Đây chính là giá trị lớn nhất cần tìm, dấu bằng xảy ra khi $m=503, n=504$ và

$\quad\quad\quad x_1=x_2=\cdots=x_{1006}=\sqrt{\frac{504}{503}}, x_{1007}=x_{1008}=\cdots=x_{2014}=-\sqrt{\frac{503}{504}} .$

Bài 6. Cho dãy số $\left(u_n\right)$ xác định bởi $u_1=1, u_{n+1}=\frac{u_n}{\sqrt{u_n^2+1}+\sqrt{2}} \forall n \in \mathbb{N}^*$.

$\operatorname{Tính} \lim \frac{u_{n+1}}{u_n}$.

Lời giải. Xét hàm số $f(x)=\frac{x}{\sqrt{x^2+1}+\sqrt{2}}$ với $x \in \mathbb{R}$ thì

$\quad\quad\quad\quad\quad\quad\quad\quad f^{\prime}(x)=\frac{1+\sqrt{2+2 x^2}}{\sqrt{1+x^2}\left(\sqrt{2}+\sqrt{1+x^2}\right)^2}>0$

nên hàm này đồng biến trên $\mathbb{R}$. Dãy số đã cho được viết lại thành

$\quad\quad\quad\quad\quad\quad\quad\quad\quad\quad \left\{\begin{array}{l}u_1=1, \\ u_{n+1}=f\left(u_n\right), n \geq 1\end{array}\right.$

thì $u_1<u_2$ nên dễ dàng chứng minh quy nạp được rằng dãy này giảm.

Do dãy này bị chặn dưới bởi 0 nên nó có giới hạn, đặt giới hạn đó là $L \geq 0$. Trong biểu thức xác định dãy, cho $n \rightarrow+\infty$, ta được

$\quad\quad\quad\quad\quad\quad\quad\quad\quad\quad L=\frac{L}{\sqrt{L^2+1}+\sqrt{2}}$

nên $L=0$. Từ đó suy ra

Bài 7 . Cho $n$ là số nguyên dương và $A$ là tập con khác rỗng của $X={1,2, . ., n}$.

(a) Tính giá trị của tổng $S(A)=\sum_{E \subset X}(-1)^{|E \cap A|}$ trong đó $E$ lấy trên tất cả các tập hợp con của $X$, kể cả tập hợp rỗng.

(b) Cho $m \in \mathbb{N}^*$. Xét $m$ tập con khác rỗng của $X$ là $A_1, A_2, \ldots, A_m$ và $m$ số nguyên khác 0 là $a_1, a_2, \ldots, a_m$ sao cho $a_1+a_2+\cdots+a_m<0$. Chứng minh rằng tồn tại $E \subset X$ sao cho $\sum_{i=1}^m(-1)^{\left|E \cap A_i\right|} a_i>0$.

Lời giải. (a) Nếu $A=X$ thì

$\quad\quad\quad\quad\quad\quad\quad\quad\quad\quad\quad S(A)=\sum_{E \subset X}(-1)^{|E|}=C_n^0-C_n^1+C_n^2-\cdots+(-1)^n C_n^n=0 .$

Còn nếu $A \neq X$, do $S(A)$ chỉ phụ thuộc vào số phần tử của $A$ nên không mất tính tổng quát, ta giả sử rằng $A={1,2, \ldots, k}$ với $k<n$. Khi đó, ta có

$\quad\quad\quad\quad\quad\quad\quad S(A) =\sum_{E \subset X-{k}}(-1)^{|E \cap A|}+\sum_{E \subset X-{k}}(-1)^{|(E \cup{k}) \cap A|} $

$\quad\quad\quad\quad\quad\quad\quad\quad\quad\quad =\sum_{E \subset X-{k}}(-1)^{|E \cap A|}-\sum_{E \subset X-{k}}(-1)^{|E \cap A|}=0 .$

Vậy $S(A)=0, \forall A \subset X$.

(b) Đặt $f(E)=\sum_{i=1}^m(-1)^{\left|E \cap A_i\right|} a_i$. Giả sử $f(E) \leq 0, \forall E$. Mà ta cũng có

$\quad\quad\quad\quad\quad\quad\quad\quad\quad\quad \sum_{E \subset X} f(E)=\sum_{i=1}^m a_i S\left(A_i\right)=0$

Suy ra $f(E)=0, \forall E \subset X$, nhưng điều này là không thể vì $f(\varnothing)<0$. Vậy luôn tồn tại $E$ sao cho $f(E)>0$.

Bài 8. Cho tam giác $A B C$ nhọn có $H$ là trực tâm. $P$ là điểm di động bên trong tam giác $A B C$ sao cho $\angle B P C=\angle B H C$. Đường thẳng qua $B$ vuông góc $A B$ cắt $P C$ tại $M$, đường thẳng qua $C$ vuông góc $A C$ cắt $P B$ tại $N$. Gọi $I$ là trung điểm của $M N$. Chứng minh rằng $I$ luôn thuộc một đường thẳng cố định.

Lời giải. Vẽ đường kính $A A^{\prime}$ của đường tròn $(A B C)$. Vì $A^{\prime} B \perp A B$ nên $B, A^{\prime}, M$ thẳng hàng. Tương tự thì $C, A^{\prime}, N$ thẳng hàng. Giả sử $B A^{\prime}, C A^{\prime}$ cắt lại $(B H C)$ lần lượt tại $E, F$. Mặt khác

$\quad\quad\quad\angle N P M=180^{\circ}-\angle B H C=\angle A=180^{\circ}-\angle B A^{\prime} C=\angle M A^{\prime} N$

nên $P A^{\prime} M N$ là tứ giác nội tiếp.

Ta sẽ chứng minh trung điểm của $A^{\prime} F, A^{\prime} E, M N$ là thẳng hàng. Theo định lý Menelaus đảo thì điều nào tương đương với

$\quad\quad\quad\quad \frac{\overline{A^{\prime} F}}{\overline{A^{\prime} N}}=\frac{\overline{E A^{\prime}}}{\overline{E M}} \Leftrightarrow \frac{\overline{A^{\prime} F}}{\overline{\overline{A^{\prime} E}}}=-\frac{\overline{A^{\prime} N}}{\overline{E M}} \Leftrightarrow \frac{A^{\prime} B}{A^{\prime} C}=\frac{A^{\prime} N}{M E}\quad\quad(*) .$

Vì $\angle B N A^{\prime}=\angle C M E$ và $\angle N B A^{\prime}=\angle M C E$ nên hai tam giác $B N A^{\prime}, C M E$ dồng dạng với nhau. Do đó

$\quad\quad\quad\quad\quad\quad\quad\quad\quad\quad \frac{A^{\prime} N}{M E}=\frac{A^{\prime} B}{C E} .$

Mặt khác, bằng biến đổi góc, ta cũng có tam giác $C A^{\prime} E$ cân tại $C$ nên $C E=C A^{\prime}$. Ta có được

$\quad\quad\quad\quad\quad\quad\quad\quad\quad\quad \frac{A^{\prime} N}{M E}=\frac{A^{\prime} B}{A^{\prime} C} .$

Do đó, khẳng định $(*)$ là đúng. Vậy nên điểm $I$ luôn nằm trên đường trung bình của tam giác $A^{\prime} E F$ là đường cố định.

 

 

 

 

 

 

 

 

 

 

 

 

 

 

 

 

 

 

 

 

 

 

 

 

 

 

 

 

 

 

 

 

 

 

 

 

 

 

 

 

 

 

 

 

 

 

 

 

 

 

 

Đề thi và đáp án kì thi chọn đội tuyển thi Quốc gia trường Phổ thông Năng khiếu năm học 2020 – 2021

ĐỀ THI

Ngày thi thứ nhất

Bài 1. Với mỗi số nguyên dương $n$, tìm số thực $M_n$ lớn nhất sao cho với mọi số thực dương $x_1, x_2, \ldots, x_n$ thì ta đều có

$\quad\quad\quad\quad\quad\quad\quad\quad\sum_{k=1}^n \frac{1}{x_k^2}+\frac{1}{\left(\sum_{k=1}^n x_k\right)^2} \geq M_n\left(\sum_{k=1}^n \frac{1}{x_k}+\frac{1}{\sum_{k=1}^n x_k}\right)^2$

Bài 2. Cho 2021 số nguyên khác 0 . Biết rằng tổng của một số bất kỳ trong chúng với tích của tất cả 2020 số còn lại luôn âm.

(a) Chứng minh rằng với mọi cách chia 2021 số này thành hai nhóm và nhân các số cùng nhóm lại với nhau thì tổng của hai tích cũng luôn âm.

(b) Một bộ số thỏa mãn đề bài thì có thể có nhiều nhất mấy số âm?

Bài 3. Cho hai hàm số $f: \mathbb{R} \rightarrow \mathbb{R}$ và $g: \mathbb{R} \rightarrow \mathbb{R}$ thỏa mãn $g(2020)>0$ và với mọi

$\quad\quad\quad\quad x, y \in \mathbb{R}$ thì $\left\{\begin{array}{l}f(x-g(y))=f(-x+2 g(y))+x g(y)-6 \\ g(y)=g(2 f(x)-y)\end{array}\right.$.

(a) Chứng minh rằng $g$ là hàm hằng.

(b) Chứng minh rằng đồ thị $h(x)=f(x)-x$ nhận $x=1$ là trục đối xứng.

Bài 4. Cho tam giác $A B C$ nhọn, nội tiếp trong đường tròn $(O)$ có trực tâm $H$ và $A H, B H, C H$ cắt cạnh đối diện lần lượt tại $D, E, F$. Gọi $I, M, N$ lần lượt là trung diểm các cạnh $B C, H B, H C$ và $B H, C H$ cắt lại $(O)$ theo thứ tự tại các điểm $L, K$. Giả sử $K L$ cắt $M N$ ở $G$.

(a) Trên $E F$, lấy diểm $T$ sao cho $A T$ vuông góc với $H I$. Chứng minh rằng $G T$ vuông góc với $O H$.

(b) Gọi $P, Q$ lần lượt là giao điểm của $D E, D F$ và $M N$. Gọi $S$ là giao điểm của $B Q, C P$. Chứng minh rằng $H S$ đi qua trung diểm của $E F$.

Ngày thi thứ hai

Bài 5. Cho số nguyên dương $n>1$. Chứng minh rằng với mọi số thực $a \in\left(0 ; \frac{1}{n}\right)$ và mọi đa thức $P(x)$ có bậc $2 n-1$ thỏa mãn điều kiện $P(0)=P(1)=0$, luôn tồn tại các số thực $x_1, x_2$ thuộc $[0 ; 1]$ sao cho $P\left(x_1\right)=P\left(x_2\right)$ và $x_2-x_1=a$.

Bài 6. Giải phương trình sau trên $\mathbb{Z}^{+}:\left(x^2+3\right)^{3^{x+1}}\left[\left(x^2+3\right)^{3^{x+1}}+1\right]+x^2+y=x^2 y$.

Bài 7. Cho các số nguyên $n>k>t>0$ và $X={1,2, \ldots, n}$. Gọi $\mathcal{F}$ là họ các tập con có $k$ phần tử của tập hợp $X$ sao cho với mọi $F, F^{\prime} \in \mathcal{F}$ thì $\left|F \cap F^{\prime}\right| \geq t$. Giả sử không có tập con có $t$ phần tử nào chứa trong tất cả các tập $F \in \mathcal{F}$.

(a) Chứng minh rằng tồn tại một tập hợp $B \subset X$ sao cho $|B|<3 k$ và $|B \cap F| \geq t+1$ với mọi $F \in \mathcal{F}$.

(b) Chứng minh rằng $|\mathcal{F}|<C_{3 k}^{t+1} C_n^{k-t-1}$.

Bài 8. Cho tam giác $A B C$ nội tiếp trong $(O)$ với $B, C$ cố định và $A$ thay đổi trên cung lớn $B C$. Dựng hình bình hành $A B D C$ và $A D$ cắt lại $(B C D)$ ở $K$.

(a) Gọi $R_1, R_2$ lần lượt là bán kính đường tròn ngoại tiếp $(K A B),(K A C)$. Chứng minh rằng tích $R_1 R_2$ không đổi.

(b) Ký hiệu $(T),\left(T^{\prime}\right)$ lần lượt là các đường tròn cùng đi qua $K$, tiếp xúc với $B D$ ở $B$ và tiếp xúc với $C D$ ở $C$. Giả sử $(T),\left(T^{\prime}\right)$ cắt nhau ở $L \neq K$. Chứng minh rằng $A L$ luôn đi qua một điểm cố định.

LỜI GIẢI

Ngày thi thứ nhất

Bài 1. Với mỗi số nguyên dương $n$, tìm số thực $M_n$ lớn nhất sao cho với mọi số thực dương $x_1, x_2, \ldots, x_n$ thì ta đều có

$\quad\quad\quad\quad\quad\quad\quad\quad\sum_{k=1}^n \frac{1}{x_k^2}+\frac{1}{\left(\sum_{k=1}^n x_k\right)^2} \geq M_n\left(\sum_{k=1}^n \frac{1}{x_k}+\frac{1}{\sum_{k=1}^n x_k}\right)^2 .$

Lời giải.

Điều Kiện Cần. Với $x_1=x_2=\cdots=x_n=1$, ta có

$\quad\quad\quad\quad\quad\quad\quad\quad n+\frac{1}{n^2} \geq M_n\left(n+\frac{1}{n}\right)^2 \text { hay } M_n \leq \frac{n^3+1}{\left(n^2+1\right)^2} .$

ĐIỀU KIỆN ĐỦ. Ta sẽ chứng minh hằng số trên thỏa mãn đề bài, tức là

$\quad\quad\quad\quad\quad\quad\quad\quad\sum_{k=1}^n \frac{1}{x_k^2}+\frac{1}{\left(\sum_{k=1}^n x_k\right)^2} \geq \frac{n^3+1}{\left(n^2+1\right)^2}\left(\sum_{k=1}^n \frac{1}{x_k}+\frac{1}{\sum_{k=1}^n x_k}\right)^2\quad\quad\quad\quad ( * )$

Theo bất đẳng thức Cauchy-Schwarz thì

$\quad\quad\quad\quad\quad\quad\quad\quad \sum_{k=1}^n \frac{1}{x_k^2} \geq \frac{1}{n}\left(\sum_{k=1}^n \frac{1}{x_k}\right)^2 .$

Đến đây, đặt $a=\sum_{k=1}^n \frac{1}{x_k}, b=\frac{1}{\sum_{k=1}^n x_k}$ là các số dương, ta đưa về chứng minh

$\quad\quad\quad\quad\quad\quad\quad\quad \frac{a^2}{n}+b^2 \geq \frac{n^3+1}{\left(n^2+1\right)^2}(a+b)^2 .$

Chú ý rằng $a \geq n^2 b$ nên quy đồng và khai triển, ta có

$\quad\quad\quad\quad\quad\quad\quad\quad \left(a-n^2 b\right)\left[\left(2 n^2-n+1\right) a-\left(n^3-n^2+2 n\right) b\right] \geq 0 .$

Chú ý rằng $n^2\left(2 n^2-n+1\right)-\left(n^3-n^2+2 n\right)=2\left(n^2+1\right) n(n-1) \geq 0$ nên

$\quad\quad\quad\quad\quad\quad\quad\quad \left(2 n^2-n+1\right) a \geq n^2\left(2 n^2-n+1\right) b \geq\left(n^3-n^2+2 n\right) b .$

Bất đẳng thức cuối đúng nên $(*)$ đúng. Vậy hằng số tốt nhất cần tìm là $M_{\max }=$ $\frac{n^3+1}{\left(n^2+1\right)^2}$

Nhận xét. Bài này là phiên bản tổng quát của câu bất đẳng thức trong đề Iran 2010 là

$\quad\quad\quad\quad\frac{1}{a^2}+\frac{1}{b^2}+\frac{1}{c^2}+\frac{1}{(a+b+c)^2} \geq \frac{7}{25}\left(\frac{1}{a}+\frac{1}{b}+\frac{1}{c}+\frac{1}{a+b+c}\right)^2 .$

Thực ra cách chứng minh cho trường hợp $n=3$ này cũng phản ánh cách xử lý cho trường hợp tổng quát trong bài toán ban đầu.

Bài 2. Cho 2021 số nguyên khác 0. Biết rằng tổng của một số bất kỳ trong chúng với tích của tất cả 2020 số còn lại luôn âm.

(a) Chứng minh rằng với mọi cách chia 2021 số này thành hai nhóm và nhân các số cùng nhóm lại với nhau thì tổng của hai tích cũng luôn âm.

(b) Một bộ số thỏa mãn đề bài thì có thể có nhiều nhất mấy số âm?

Lời giải: Đặt các số đã cho là $a_1, a_2, \ldots, a_{2021}$ và $S$ là tích của tất cả số này. Theo dề bài thì

$\quad\quad\quad\quad\quad\quad\quad\quad\quad\quad a_k+\frac{S}{a_k}=\frac{a_k^2+S}{a_k}<0$

với mọi $k=1,2, \ldots, 2021$.

Nếu như $S>0$ thì rõ ràng theo trên, ta phải có $a_k<0, \forall k$, nhưng điều này vô lý vì khi đó $S$ là tích của 2021 số âm nên cũng âm. Do đó, $S<0$ và trong các số đã cho, có lẻ số âm.

Nếu như số lượng số âm là lớn hơn 1 , giả sử hai trong các số đó là $a_1, a_2$ và $\left|a_2\right| \geq\left|a_1\right|$. Khi đó, ta có $a_2 a_3 \ldots a_{2021}>0$ (do trong các số này có chẵn số âm) và $\left|a_2 a_3 \ldots a_{2021}\right| \geq\left|a_1\right|$ nên

$\quad\quad\quad\quad\quad\quad\quad\quad\quad\quad a_1+a_2 a_3 \ldots a_{2021} \geq 0 .$

Điều vô lý này cho thấy không thể có nhiều hơn 1 số âm, và vì thế nên có đúng một số âm. Giả sử $a_1$ là số âm duy nhất đó thì ta có $a_1+a_2 a_3 \ldots a_{2021}<0$ nên

$\quad\quad\quad\quad\quad\quad\quad\quad\quad\quad \left|a_1\right|>a_2 a_3 \ldots a_{2021} .$

Với mọi cách chia 2021 số đã cho thành hai nhóm thì sẽ có một nhóm chứa số âm là $a_1$, đồng thời, giá trị tuyệt đối của tích các số trong nhóm đó, vì có chứa $\left|a_1\right|$, nên sẽ lớn hơn tích các số của nhóm còn lại. Suy ra tổng của hai tích sẽ âm.

Do đó, ta có khẳng định ở câu (a) và đáp số cho câu (b) là 1 .

Nhận xét. Nhờ việc khảo sát số lượng số âm, ta chỉ ra trực tiếp được rằng có đúng 1 số âm và các câu (a), (b) trở thành hiển nhiên. Bài toán cũng đúng khi thay 2021 số nguyên bởi $2 n+1$ số thực nào đó có giá trị tuyệt đối không nhỏ hơn 1 .

Bài 3. Cho hai hàm số $f: \mathbb{R} \rightarrow \mathbb{R}$ và $g: \mathbb{R} \rightarrow \mathbb{R}$ thỏa mãn $g(2020)>0$ và

$\quad\quad\quad\quad\quad\quad\quad\quad \left\{\begin{array}{l}f(x-g(y))=f(-x+2 g(y))+x g(y)-6 \\ g(y)=g(2 f(x)-y)\end{array}\right.$

với mọi $x, y \in \mathbb{R}$.

(a) Chứng minh rằng $g$ là hàm hằng.

(b) Chứng minh rằng đồ thị hàm số $h(x)=f(x)-x$ nhận $x=1$ là trục đối xứng.

Lời giải . (a) Trong điều kiện thứ nhất, thay $x$ bởi $\frac{3}{2} g(y)$ thì ta có ngay

$\quad\quad\quad\quad\quad\quad f\left(\frac{g(y)}{2}\right)=f\left(\frac{g(y)}{2}\right)+\frac{3}{2} g^2(y)-6 \text { hay } g^2(y)=4$

Suy ra $g(y)=\pm 2$ với mọi $y$. Do $g(2020)>0$ nên loại trường hợp $g(y)=-2, \forall y$. Ta sẽ chỉ ra rằng không xảy ra trường hợp $\exists a \neq b$ sao cho $g(a)=2, g(b)=-2$. Thay $y=2020$ vào điều kiện đầu, ta có

$\quad\quad\quad\quad\quad\quad\quad\quad\quad\quad f(x-c)-f(-x+2 c)=x c-6$

với $c=g(2020)>0$. Suy ra $f(u)-f(v)$ toàn ánh trên $\mathbb{R}$ với $u, v \in \mathbb{R}$. Tiếp theo trong diều kiện sau, thay $y$ bởi $a, b$, ta có

$\quad\quad\quad\quad\quad\quad g(a)=g(2 f(x)-a)=2 ; g(b)=g(2 f(x)-b)=-2$

với mọi $x \in \mathbb{R}$. Lại do tính toàn ánh, chọn $u, v$ sao cho

$\quad\quad\quad\quad\quad\quad\quad\quad\quad\quad f(u)-f(v)=\frac{a-b}{2}$

thì $2 f(u)-a=2 f(v)-b$. Suy ra

$\quad\quad\quad\quad\quad\quad\quad 2=g(2 f(u)-a)=g(2 f(v)-b)=-2,$

điều này vô lý. Vì thế $g(x)=2, \forall x$ nên $g$ là hàm hằng.

(b) Với $g(x)=2, \forall x$, thay vào điều kiện đầu, ta có

$\quad\quad\quad\quad\quad\quad\quad\quad f(x-2)=f(-x+4)+2 x-6$

hay

$\quad\quad\quad\quad f(x)=f(2-x)+2 x-2 \text { kéo theo } f(x)-x=f(2-x)-(2-x) .$

Điều này cho thấy $h(x)=h(2-x)$ hay $h(x)$ có đồ thị nhận $x=1$ là trục đối xứng.

Bài 4. Cho tam giác $A B C$ nhọn, nội tiếp trong đường tròn $(O)$ có trực tâm $H$ và $\mathrm{AH}, \mathrm{BH}, \mathrm{CH}$ cắt cạnh đối diện lần lượt tại $D, E, F$. Gọi $I, M, N$ lần lượt là trung điểm các cạnh $B C, H B, H C$ và $B H, C H$ cắt lại $(O)$ theo thứ tự tại các điểm $L, K$. Giả sử $K L$ cắt $M N$ ở $G$.

(a) Trên $E F$, lấy điểm $T$ sao cho $A T$ vuông góc với $H I$. Chứng minh rằng $G T$ vuông góc với $O H$.

(b) Gọi $P, Q$ lần lượt là giao điểm của $D E, D F$ và $M N$. Gọi $S$ là giao điểm của $B Q, C P$. Chứng minh rằng $H S$ di qua trung điểm của $E F$.

Lời giải. (a) Giả sử tia $I H$ cắt $(O)$ ở $R$ thì theo kết quả quen thuộc, ta có $\angle A R H=$ $90^{\circ}$. Vì thế nên $T \in A R$. Bằng cách xét trục đẳng phương của các đường tròn đường kính $A H, B C$ và đường tròn $(O)$, ta có $A R, E F, B C$ dồng quy. Từ đó suy ra $T \in B C$.

Gọi $\left(O^{\prime}\right)$ là đường tròn Euler của tam giác $A B C$ thì $D, E, F, M, N \in(O)$. Dễ thấy rằng

$\quad\quad\quad\quad H M \cdot H K=\frac{1}{2} H B \cdot H K=\frac{1}{2} H C \cdot H L=H N \cdot H L$

nên $M, N, K, L$ cùng thuộc đường tròn. Suy ra

$G L \cdot G K=G M \cdot G N$ nên

Ngoài ra, ta cũng có nên $G T$ chính là trục đăng phương của $(O),\left(O^{\prime}\right)$. Điều này cho thấy $G T \perp O O^{\prime}$ hay $G T \perp O H$ (do $O^{\prime}$ là trung diểm của $\left.O H\right)$.

(b) Ta có $D H$ là phân giác của góc $P D Q$, và $P Q \perp H D$ nên dễ thấy tứ giác $H P D Q$ là hình thoi. Ta biến đổi góc như sau

$\quad\quad\quad\quad\quad\quad\quad\quad \angle H P Q=\angle D Q P=\angle Q D B=\angle F H B .$

Suy ra $\angle H P N=\angle M H N$ nên $H N$ tiếp xúc với $(H M P)$ hay $N P \cdot N M=N H^2=$ $N C^2$. Do đó, hai tam giác $N P C$ và $N C M$ dồng dạng với nhau. Suy ra

$\quad\quad\quad\quad\quad\quad\quad\quad\quad\quad \angle N C P=\angle N M C=\angle M C B,$

nên $C P$ là dối trung của tam giác $H B C$. Chứng minh tương tự thì $B Q$ là dối trung trong tam giác $H B C$ nên điểm $S$ chính là điểm Lemoine của tam giác này, kéo theo $H S$ cũng là dối trung của tam giác $H B C$. Lại có $E F$ là dối song ứng với đỉnh $H$ trong tam giác $H B C$ nên suy ra $H S$ chia đôi đoạn thẳng $E F$.

 

Ngày thi thứ hai

Bài 5. Cho số nguyên dương $n>1$. Chứng minh rằng với mọi số thực $a \in$ $\left(0 ; \frac{1}{n}\right)$ và mọi đa thức $P(x)$ có bậc $2 n-1$ thỏa mãn điều kiện $P(0)=P(1)=0$, luôn tồn tại các số thực $x_1, x_2$ thuộc $[0 ; 1]$ sao cho $P\left(x_1\right)=P\left(x_2\right)$ và $x_2-x_1=a$.

Lời giải. Không mất tính tổng quát, giả sử không tồn tại các số $x_1, x_2$ thỏa mãn đề bài. Khi đó, xét đa thức $Q(x)=P(x+a)-P(x)$ sẽ vô nghiệm trên $[0 ; 1-a]$. Suy ra $Q(x)$ sẽ không đổi dấu trên miền đó vì nếu không, dùng tính liên tục thì sẽ mâu thuẫn. Không mất tính tổng quát, giả sử $Q(x)>0, \forall x \in[0 ; 1-a]$. Nhận xét rằng $\frac{1}{a} \notin \mathbb{Z}^{+}$vì nếu không, đặt $\frac{1}{a}=m \in \mathbb{Z}^{+}$thì ta sẽ có $m a=1$ và

$\quad\quad\quad\quad Q(a)+Q(2 a)+\cdots+Q((m-1) a)=P(1)-P(0)=0$

trong khi các số hạng ở vế trái đều dương, vô lý. Tiếp theo, ta có

$\quad\quad\quad\quad\quad\quad Q(a)=P(a)-P(0)>0 \Rightarrow P(a)>0$

Chứng minh tương tự thì $P(k a)>0, \forall k=1,2, \ldots, n$. Mặt khác,

$\quad\quad\quad\quad Q(1-a)=P(1)-P(1-a)=-P(1-a)>0 \text { nên } P(1-a)<0 .$

Tương tự thì

$\quad\quad\quad\quad\quad\quad\quad\quad\quad\quad P(1-l a)<0, \forall l=1,2, \ldots, n .$

Rõ ràng với mỗi $k \in{1,2, \ldots, n}$, ta luôn chọn được số nguyên $l_k \in{1,2, \ldots, n}$ để $(k-1) a<1-l_k a<k a$. Thật vậy, đánh giá này tương đương với

$\quad\quad\quad\quad\quad\quad\quad\quad\quad\quad \frac{1}{a}-k<l_k<\frac{1}{a}+1-k .$

Mà $a \in\left(0 ; \frac{1}{n}\right)$ nên $\frac{1}{a}-k>0$ và $\frac{1}{a}-k \notin \mathbb{Z}$ nên khoảng trên phải chứa một số nguyên và gọi số đó là $l_k$. Theo trên thì

$\quad\quad\quad\quad\quad\quad P((k-1) a)>0, P\left(1-l_k a\right)<0, P(k a)>0$

nên trên khoảng $((k-1) a ; k a)$ thì đa thức $P(x)$ có 2 nghiệm thực phân biệt. Áp dụng điều này cho tất cả các khoảng $(0 ; a),(a ; 2 a), \ldots,((n-1) a ; n a)$ thì ta thấy $P(x)$ sẽ có $2 n$ nghiệm thực phân biệt; trong khi $P(x)$ chỉ có bậc $2 n-1$, vô lý.

Vì thế nên điều giả sử là sai và tồn tại hai số thực $x_1, x_2 \in[0 ; 1]$ thỏa mãn đề bài.

Nhận xét. Ta thấy rằng do $P(x)$ đã có sã̃n hai nghiệm là $x=0, x=1$ nên lập luận trên vẫn đúng khi xét bậc đa thức là $2 n+1$ (vì ta đã chỉ ra được đến $2 n+2$ nghiệm phân biệt). Một phiên bản tương tự khi xét hàm số liên tục $f(x)$ tùy ý là:

Cho số nguyên dương $n$, xét hàm số $f(x)$ liên tục tục trên $[0 ; n]$ sao cho $f(0)=f(n)$. Khi đó, với $k \in{1,2, \ldots, n-1}$, tồn tại $x_1, x_2 \in[0 ; n]$ mà

$\quad\quad\quad\quad\quad\quad\quad\quad\quad\quad x_2-x_1=k \text { và } f\left(x_1\right)=f\left(x_2\right) .$

Bài 6. Giải phương trình sau trên tập số nguyên dương

$\quad\quad\quad\quad\quad\left(x^2+3\right)^{3^{x+1}}\left[\left(x^2+3\right)^{3^{x+1}}+1\right]+x^2+y=x^2 y .$

Lời giải . Dễ thấy $x=1$ không thỏa nên ta xét $x>1$. Thử trực tiếp thấy $x=2$ thỏa mãn. Xét $x>2$, kéo theo $x^2-1>3$. Phương trình đã cho viết lại thành

$\quad\quad\quad\quad\left(x^2+3\right)^{3^{x+1}}\left[\left(x^2+3\right)^{3^{x+1}}+1\right]+1=(y-1)\left(x^2-1\right) $

$\quad\quad\quad\quad \Leftrightarrow x^2-1\mid\left(x^2+3\right)^{3^{x+1}}\left[\left(x^2+3\right)^{3^{x+1}}+1\right]+1 $

$\quad\quad\quad\quad \Leftrightarrow 4^{3^{x+1}}\left(4^{3^{x+1}}+1\right)+1 \equiv 0 \quad\left(\bmod x^2-1\right) .$

Đặt $a=4^{3^{x+1}}$ thì ta có $a^2+a+1 \equiv 0\left(\bmod x^2-1\right)$ nên $a^3 \equiv 1\left(\bmod x^2-1\right)$ hay

$\quad\quad\quad\quad\quad\quad\quad\quad\quad\quad 4^{3^{x+2}} \equiv 1 \quad\left(\bmod x^2-1\right)$

Lại chú ý rằng
$\quad\quad\quad\quad\quad\quad a=4^{3^{x+1}}=4^{3 \cdot 3^x}=\left(4^3\right)^{3^x} \equiv 1 \quad(\bmod 9)$
nên nếu đặt $M=a^2+a+1$ thì $M \equiv 3(\bmod 9)$. Điều này cho thấy $v_3(M)=1$ nên $v_3\left(x^2-1\right) \leq 1$. Mà $x^2-1>3$ và dễ thấy $x^2-1$ lẻ nên $x^2-1$ phải có một ước nguyên tố lẻ $p>3$.

Đặt $h=\operatorname{ord}_p(4)$ thì $h \mid p-1$ và $h \mid 3^{x+2}$. Suy ra $h=3^k$ với $0 \leq k \leq x+2$. Tuy nhiên, nếu $k \leq x+1$ thì $4^{3^k} \equiv 1(\bmod p)$ mà $4^{3^k}-1 \mid a-1$ nên $a \equiv 1(\bmod p)$. Điều này vô lý do sẽ kéo theo
$\quad\quad\quad\quad\quad\quad\quad\quad\quad\quad 0 \equiv a^2+a+1 \equiv 3 \quad(\bmod p) .$
Vì thế nên $k=x+2$ hay $\operatorname{ord}_p(4)=3^{x+2}$ nên $h=3^{x+2} \leq p-1 \leq x-1$. Tuy nhiên, đánh giá này là không thể xảy ra với mọi $x>2$. Vì thế nên nghiệm duy nhất của phương trình dã cho là

$\quad\quad\quad\quad\quad\quad\quad\quad (x ; y)=\left(2 ; \frac{7^{27}\left(7^{27}+1\right)+4}{3}\right)$.

Nhận xét. Các bài toán lũy thừa tầng trong số học thường gợi ý đến việc dùng cấp theo kiểu: với mọi $a, m, k$ nguyên dương lớn hơn 1 và $p$ là số nguyên tố thì $a^{m^k} \equiv 1$ $(\bmod p)$ và $h=\operatorname{ord}_p(a)$ thì $h \mid m^k$ nên có $h=m^t$ với $t \leq k$. Một kết quả có liên quan là: mọi ước nguyên tố $p$ của $2^{2^n}+1$ thì đều thỏa $2^{n+1} \mid p-1$.

Bài 7. Cho các số nguyên $n>k>t>0$ và $X={1,2, \ldots, n}$. Gọi $\mathcal{F}$ là họ các tập con có $k$ phần tử của tập hợp $X$ sao cho với mọi $F, F^{\prime} \in \mathcal{F}$ thì $\left|F \cap F^{\prime}\right| \geq t$. Giả sử không có tập con có $t$ phần tử nào được chứa trong tất cả các tập $F \in \mathcal{F}$.

(a) Chứng minh rằng tồn tại một tập hợp $B \subset X$ sao cho $|B|<3 k$ và $|B \cap F| \geq t+1$ với mọi $F \in \mathcal{F}$.

(b) Chứng minh rằng $|\mathcal{F}|<C_{3 k}^{t+1} C_n^{k-t-1}$.

Lời giải. (a) Theo giả thiết thì rõ ràng $|\mathcal{F}| \geq 3$, vì nếu không thì diều kiện ii) sẽ không được thỏa mãn. Ta xét các trường hợp sau Nếu như mọi tập $F, F^{\prime} \in \mathcal{F}$ đều có tính chất $\left|F \cap F^{\prime}\right| \geq t+1$ thì ta chỉ cần chọn $B$ là một tập bất kỳ trong $\mathcal{F}$ là được, rõ ràng $|B|=k<3 k$, thỏa mãn đề bài.

Ngược lại, tồn tại hai tập $F, F^{\prime} \in \mathcal{F}$ mà $\left|F \cap F^{\prime}\right|=t$ thì theo giả thiết, không có tập con $t$ phần tử nào được chứa trong tất cả tập hợp của họ $\mathcal{F}$; vì thế nên phải có $F^{\prime \prime}$ sao cho $\left(F \cap F^{\prime}\right) \not \subset F^{\prime \prime}$. Mặt khác, vì $\left|F^{\prime \prime}\right|=k>t$ nên phải có phần tử trong $F^{\prime \prime}$ mà không thuộc vào $F \cap F^{\prime}$. Khi đó, xét $B=F \cup F^{\prime} \cup F^{\prime \prime}$ thì rõ ràng

$\quad\quad\quad\quad\quad |B| \leq\left|F \cup F^{\prime}\right|+\left|F^{\prime \prime}\right| \leq 2 k-t+k<3 k .$

Ta sẽ chứng minh rằng tập hợp này thỏa mãn đề bài. Thật vậy, xét tập hợp $G \in \mathcal{F}$

  • Nếu $G$ là một trong ba tập $F, F^{\prime}, F^{\prime \prime}$ thì có ngay

$\quad\quad\quad\quad\quad\quad\quad\quad\quad\quad |B \cap G| \geq k \geq t+1$

  • Nếu $G \neq F, F^{\prime}, F^{\prime \prime}$ thì nếu $|G \cap F| \geq t+1$ hoặc $\left|G \cap F^{\prime}\right| \geq t+1$ là xong; ngược lại $|G \cap F|=t$ và $t$ phần tử chung này không đồng thời thuộc $F^{\prime \prime}$, nên sẽ còn một phần tử chung khác giữa hai tập hợp $G, F^{\prime^{\prime}}$ do ta có $\left|G \cap F^{\prime \prime}\right| \geq t$, kéo theo $|B \cap G| \geq t+1$.

Trong mọi trường hợp, ta luôn chọn được tập hợp $B \subset X$ thỏa mãn đề bài.

(b) Ta thực hiện xây dựng một họ $\mathcal{F}^{\prime}$ các tập con của $X$ như sau

  • Chọn $t+1$ phần tử nào đó trong tập $B$ ở trên, số cách chọn sẽ nhỏ hơn $C_{3 k}^{t+1}$.
  • Chọn $k-t-1$ phần tử nào đó trong tập $X$ thì có $C_n^{k-t-1}$ cách.

Hợp của hai nhóm này lại thì được một tập con của $X$ có $k$ phần tử (hoặc ít hơn) mà giao với $B$ là $t+1$ phần tử. Từ đó dễ thấy rằng $\mathcal{F} \subset \mathcal{F}^{\prime}$ nên ta phải có $|\mathcal{F}| \leq\left|\mathcal{F}^{\prime}\right|<C_{3 k}^{t+1} C_n^{k-t-1}$. Bài toán được giải quyết.

Nhận xét. Cố định $k, t \geq 2$ và cho $n$ đủ lớn thì ta có

$\quad\quad\quad\quad\quad\quad\quad\quad\quad\quad C_{3 k}^{t+1} C_n^{k-t-1}<C_{n-t}^{k-t} .$

Từ đó, ta có chứng minh của định lý Erdós – Ko – Rado (1938) là: $|\mathcal{F}| \leq C_{n-t}^{k-t}$ với $n$ đủ lớn. Trường hợp $t=1$ thì khi $n \geq 2 k$, ta cũng có $|\mathcal{F}| \leq C_{n-1}^{k-1}$.

Các bài toán theo kiểu family set đã được nghiên cứu rất sâu sắc và vì hầu hết là các đánh giá, ước lượng tổ hợp không có dấu bằng nên đó đều là các kết quả khó.

Bài 8. Cho tam giác $A B C$ nội tiếp trong đường tròn $(O)$ với $B, C$ cố định và $A$ thay đổi trên cung lớn $B C$. Dựng hình bình hành $A B D C$ và $A D$ cắt lại $(B C D)$ ở $K$.

(a) Gọi $R_1, R_2$ lần lượt là bán kính đường tròn ngoại tiếp $(K A B),(K A C)$. Chứng minh rằng tích $R_1 R_2$ không đổi.

(b) Ký hiệu $(T),\left(T^{\prime}\right)$ lần lượt là các đường tròn cùng đi qua $K$, tiếp xúc với $B D$ ở $B$ và tiếp xúc với $C D$ ở $C$. Giả sử $(T),\left(T^{\prime}\right)$ cắt nhau ở $L \neq K$. Chứng minh rằng $A L$ luôn đi qua một điểm cố định.

Lời giải. (a) Gọi $R$ là bán kính của $(O)$, ta có

$\quad\quad\quad\quad\quad\quad\quad\quad \angle A K B=180^{\circ}-\angle B K D=\angle B C D=\angle A B C$

nên theo định lý sin thì

$\quad\quad\quad\quad\quad\quad\quad\quad R_1=\frac{A B}{2 \sin A K B}=\frac{2 R \sin C}{2 \sin B}=R \cdot \frac{\sin C}{\sin B} .$

Tương tự thì $R_2=R \cdot \frac{\sin B}{\sin C}$ nên $R_1 R_2=R^2$, không đổi.

(b) Do các tiếp tuyến nên biến đổi góc được

$\quad\quad\quad\quad\quad\quad \angle K L B=180^{\circ}-\angle K B D, \angle K L C=180^{\circ}-\angle K C D .$

Suy ra

$\quad\quad\quad\quad\quad\quad \angle K L B+\angle K L C=360^{\circ}-(\angle K B D+\angle K C D)=180^{\circ}$

nên $B, L, C$ thẳng hàng. Do $B D$ tiếp xúc với $(K B L)$ nên

$\quad\quad\quad\quad\quad\quad \angle B K L=\angle L B D=\angle C K D,$

mà $K D$ là trung tuyến của tam giác $K B C$ nên $K L$ là đối trung của tam giác này. Gọi $I$ là trung điểm $B C$. Ta có

$\quad\quad\quad\quad\quad\quad \angle K B I=\angle K D C=\angle B A K$

nên $B C$ tiếp xúc với $(A B K)$. Suy ra $\frac{B K}{A B}=\frac{I K}{I A}$. Tương tự thì

$\quad\quad\quad\quad\quad\quad \frac{C K}{A C}=\frac{I K}{I A} \text { nên } \frac{K B}{K C}=\frac{A B}{A C} .$

Theo tính chất quen thuộc của đường đối trung thì

$\quad\quad\quad\quad\quad\quad \frac{L B}{L C}=\frac{K B^2}{K C^2}=\frac{A B^2}{A C^2}$

nên $A L$ cũng là đường đối trung của tam giác $A B C$, diều này cho thấy $A L$ đi qua giao điểm hai tiếp tuyến của $(O)$ ở $B, C$, là điểm cố định.

Nhận xét. Điểm $K$ trong đề bài chính là điểm Humpty của tam giác $A B C$, và cách dựng như trên là một cách tuy dễ nhưng tương đối mới.

 

 

 

 

 

 

 

 

 

 

 

 

 

 

 

 

 

 

 

 

 

 

 

 

 

 

 

 

 

 

 

 

 

 

 

 

 

 

 

 

 

 

 

 

 

 

 

 

 

 

 

 

 

 

 

 

 

 

 

 

 

 

 

 

 

 

 

 

 

 

 

 

 

 

 

 

 

 

 

 

 

 

 

 

 

 

 

 

 

 

 

Đề thi và đáp án kì thi chọn đội tuyển thi Quốc gia trường Phổ thông Năng khiếu năm học 2011 – 2012

 

ĐỀ THI

Ngày thi thứ nhất

Bài 1. Cho các số $a, b, c>0$ thoả mãn $a b+b c+c a=1$. Chứng minh rằng:

$\quad\quad\quad\quad\quad\quad\quad\quad\frac{1}{3+2\left(a^2-b c\right)}+\frac{1}{3+2\left(b^2-c a\right)}+\frac{1}{3+2\left(c^2-a b\right)} \geq 1$

Bài 2. Có bao nhiêu bộ số nguyên dương $(x, y, z, t)$ thoả mãn

$\quad\quad\quad\quad\quad\quad\quad\quad\quad\quad 12<x<y<z<t \text { và } x+y+z+t=2011 ?$

Bài 3. Cho tam giác $A B C$ nội tiếp đường tròn $(O, R)$. Gọi $\left(\mathcal{C}_1\right)$ là đường tròn thay đổi luôn qua $B, C$ và lần lượt cắt các cạnh $A B, A C$ tại $M, N$ khác $B, C$.

(a) Chứng minh rằng $(A M N)$ luôn tiếp xúc với một đường cố định.

(b) Cho $B, C$ cố định, $B C=2 R$ và $A$ thay đổi trên $(O)$. Đường thẳng qua $A$ vuông góc $B C$ cắt $(O)$ tại $D$ và cắt $\left(\mathcal{C}_1\right)$ tại $E, F$. Chứng minh rằng nếu $A$ và $\left(\mathcal{C}_1\right)$ thay đổi sao cho $\frac{E F}{A D}=\frac{\sqrt{5}}{2}$ thì $(A M N)$ luôn tiếp xúc với một đường cố định.

Bài 4. Cho $p$ là số nguyên tố lẻ và đa thức $Q(x)=(p-1) x^p-x-1$. Xét dãy số $\left(a_n\right)$ thoả mãn

$\quad\quad\quad\quad\quad\quad\quad\quad a_0=\frac{p-1}{2}, a_n=a_{n-1}+Q\left(a_{n-1}\right) \forall n \in \mathbb{N}^* .$

(a) Chứng minh rằng với mọi số nguyên dương $n$ thì $\left(a_n, p\right)=1$.

(b) Chứng minh rằng với mọi số nguyên dương $n$ thì $Q\left(a_n\right) \equiv 0\left(\bmod p^n\right)$.

Ngày thi thứ hai

Bài 5. Cho dãy số $\left(u_n\right)$ thoả mãn $u_1=\frac{1}{6}$ và $u_{n+1}=u_n^2+\frac{2}{3} u_n \forall n \in \mathbb{N}^*$.

Tìm $\lim \frac{5 u_{n+1}^2-2 u_n^2 u_{n+1}+5 u_n u_{n+1}}{3 u_n^2+u_n u_{n+1}\left(4+u_n^2\right)}$.

Bài 6. Cho hàm số $f: \mathbb{N} \times \mathbb{N} \rightarrow \mathbb{N}$ thoả mãn $f(0,0)=0$ và:

$\quad\quad\quad\quad\quad\quad\quad\quad f(a, b)=\left\{\begin{array}{l}f\left(\left\lfloor\frac{a}{2}\right\rfloor,\left\lfloor\frac{b}{2}\right\rfloor\right) \text { khi } a+b \equiv 0 \quad(\bmod 2) \\ 1+f\left(\left\lfloor\frac{a}{2}\right\rfloor,\left\lfloor\frac{b}{2}\right\rfloor\right) \text { khi } a+b \equiv 1 \quad(\bmod 2)\end{array}\right.$

(a) Có bao nhiêu số tự nhiên $m \leq 2011$ sao cho $f(2011, m)=5$ ?

(b) Cho số lẻ $p$, cho $n \in \mathbb{N}\left(1<p<2^n\right)$ và $A$ là tập hợp gồm $p$ số tự nhiên không vượt quá $2^n-1$. Chứng minh rằng $\sum_{{a, b} \subset A} f(a, b) \leq n \cdot \frac{p^2-1}{4}$.

Bài 7. Cho tam giác $A B C$ nội tiếp đường tròn $(O)$ với $B, C$ cố định và $A$ thay đổi trên $(O)$. Đường trung trực $d$ của $B C$ cắt $A B, A C$ tại $M, N$. Gọi $P, Q$ lần lượt là các điểm đối xứng của $M, N$ qua $O . K$ là giao điểm của $B P$ và $C Q$.

(a) Chứng minh rằng $K$ luôn thuộc một đường tròn cố định.

(b) Kết luận trên còn đúng không khi $d$ là đường thẳng Euler của tam giác $A B C ?$

Bài 8. Với mọi số nguyên dương $n$, đặt $S_n=x^n+y^n+z^n$. Ta đã biết rằng $S_n=$ $P_n(s, t, p)$ với $s=x+y+z, t=x y+y z+z x, p=x y z$. Hãy tính tổng các hệ số của các đơn thức chứa $p$ trong $P_{2011}(s, t, p)$.

 

LỜI GIẢI

Bài 1. Cho các số $a, b, c>0$ thoả mãn $a b+b c+c a=1$. Chứng minh rằng:

$\quad\quad\quad\quad\quad\quad\quad\quad \frac{1}{3+2\left(a^2-b c\right)}+\frac{1}{3+2\left(b^2-c a\right)}+\frac{1}{3+2\left(c^2-a b\right)} \geq 1$

Lời giải. Đặt $a b=x ; b c=y ; c a=z$ thì ta có $x+y+z=1$. Khi đó áp dụng bất đẳng thức Cauchy-Schwarz, ta có

$\quad\quad\quad\quad\quad \sum_{c y c} \frac{1}{3+2\left(\frac{x z}{y}-y\right)} =\sum_{c y c} \frac{y^2}{3 y^2+2 x y z-2 y^3} $

$\quad\quad\quad\quad\quad\quad\quad\quad\quad\quad\quad\quad  \geq \frac{1}{3\left(x^2+y^2+z^2\right)+6 x y z-2\left(x^3+y^3+z^3\right)} .$

Ta đưa về chứng minh

$\quad\quad\quad\quad\quad\quad\quad\quad 3\left(x^2+y^2+z^2\right)+6 x y z-2\left(x^3+y^3+z^3\right) \leq 1 .$

Tuy nhiên đây lại là đẳng thức vì

$\quad\quad\quad\quad 3\left(x^2+y^2+z^2\right)+6 x y z-2\left(x^3+y^3+z^3\right) $

$\quad\quad\quad\quad\quad =3\left(x^2+y^2+z^2\right)-2(x+y+z)\left(x^2+y^2+z^2-x y-y z-z x\right) $

$\quad\quad\quad\quad\quad =3\left(x^2+y^2+z^2\right)-2\left(x^2+y^2+z^2\right)+2(x y+y z+z x) $

$\quad\quad\quad\quad\quad =(x+y+z)^2=1$

Đẳng thức xảy ra khi $x=y=z=1$ hay $a=b=c=\frac{1}{\sqrt{3}}$.

Bài 2. Có bao nhiêu bộ số nguyên dương $(x, y, z, t)$ thoả mãn $12<x<y<z<t$ và $x+y+z+t=2011 ?$

Lời giải. Đặt $x^{\prime}=x-12 ; y^{\prime}=y-12 ; z^{\prime}=z-12 ; t^{\prime}=t-12$. Phương trình đã cho tương đương với:

$\quad\quad\quad\quad\quad x^{\prime}+y^{\prime}+z^{\prime}+t^{\prime}=2011-48=1963 \text { với } 0 \leq x^{\prime}<y^{\prime}<z^{\prime}<t^{\prime}.$

Theo bài toán chia kẹo Euler thì nếu không có điều kiện thứ hai, số nghiệm của phương trình trên sẽ là $C_{1966}^3$. Ta sẽ trừ ra các trường hợp các số bị trùng nhau

  • Số bộ có 3 số giống nhau là $C_4^3 \cdot\left(1+\left\lfloor\frac{1963}{3}\right\rfloor\right)=2620=A$.
  • Số bộ có 2 số giống nhau là $C_4^2\left(\sum_{a=0}^{981}(1964-2 a)\right)=5791836=B$.

Do mỗi bộ nghiệm như trên chỉ tồn tại 1 cách sắp xếp $x, y, z, t$ thỏa mãn nên số bộ thoả mãn đề bài là

$\quad\quad\quad\quad\quad\quad\quad\quad \frac{C_{1966}^3-B+2 A}{4 !}=\frac{C_{1966}^3-5786596}{4 !} .$

Bài 3. Cho tam giác $A B C$ nội tiếp đường tròn $(O, R)$. Gọi $\left(\mathcal{C}_1\right)$ là đường tròn thay đổi luôn qua $B, C$ và lần lượt cắt các cạnh $A B, A C$ tại $M, N$ khác $B, C$.

(a) Chứng minh rằng $(A M N)$ luôn tiếp xúc với một đường cố định.

(b) Cho $B, C$ cố định, $B C=2 R$ và $A$ thay đổi trên $(O)$. Đường thẳng qua $A$ vuông góc $B C$ cắt $(O)$ tại $D$ và cắt $\left(\mathcal{C}_1\right)$ tại $E, F$. Chứng minh rằng nếu $A$ và $\left(\mathcal{C}_1\right)$ thay đổi sao cho $\frac{E F}{A D}=\frac{\sqrt{5}}{2}$ thì $(A M N)$ luôn tiếp xúc với một đường cố định.

Lời giải. (a) Gọi $d$ là đường thẳng qua $A$, song song với $B C$ và cắt đường tròn $(O)$ tại $T$. Bằng biến đổi góc, ta có

$\quad\quad\quad\quad\quad\quad\quad\quad \angle T A C=\angle A C B=\angle A M N .$

Suy ra $A T$ là tiếp tuyến của đường tròn $(A M N)$ nên ( $A M N)$ tiếp xúc với đường thẳng $d$ cố định.

(b) Gọi $H$ là hình chiếu của $A$ lên $B C$. Xét phương tích từ $A$ đến $\mathcal{C}_1$ thì

$\quad\quad\quad\quad\quad\quad\quad\quad A M \cdot A B=A N \cdot A C=A E \cdot A F \text {. }$

Ta có $H E \cdot H F=H B \cdot H C=H A^2$, mà $\frac{E F}{A D}=\frac{\sqrt{5}}{2}$ nên $H E+H F=\sqrt{5} A H$. Giải hệ này, ta có

$\quad\quad\quad\quad\quad\quad\quad\quad H E=\frac{\sqrt{5}-1}{2} A H \text { và } H F=\frac{\sqrt{5}+1}{2} A H .$

Suy ra $A E=A H-H E=\frac{3-\sqrt{5}}{2} A H$ và $A F=A H+H F=\frac{3+\sqrt{5}}{2} A H$. Từ đó ta được

$\quad\quad\quad\quad\quad\quad\quad\quad A E \cdot A F=\frac{3-\sqrt{5}}{2} A H \cdot \frac{3+\sqrt{5}}{2} A H=A H^2 \text {. }$

Vì thế nên $A H^2=A M \cdot A B=A N \cdot A C$, chứng tỏ $H M, H N$ lần lượt vuông góc với $A B, A C$. Suy ra $(A M N)$ có đường kính là $A H$ nên $(A M N)$ tiếp xúc với $B C$ là đường thẳng cố định.

Bài 4. Cho $p$ là số nguyên tố lẻ và đa thức $Q(x)=(p-1) x^p-x-1$. Xét dãy số $\left(a_n\right)$ thoả mãn

$\quad\quad\quad\quad\quad\quad\quad\quad a_0=\frac{p-1}{2}, a_n=a_{n-1}+Q\left(a_{n-1}\right) \forall n \in \mathbb{N}^* .$

(a) Chứng minh rằng với mọi số nguyên dương $n$ thì $\operatorname{gcd}\left(a_n, p\right)=1$.

(b) Chứng minh rằng với mọi số nguyên dương $n$ thì $Q\left(a_n\right) \equiv 0\left(\bmod p^n\right)$.

Lời giải. (a) Ta có

$\quad\quad\quad\quad a_0=\frac{p-1}{2} \text { và } a_1=a_0+(p-1) a_0^p-a_0-1=\frac{(p-1)^{p+1}}{2^p}-1$

không chia hết cho $p$.

Giả sử tồn tại $k$ nhỏ nhất sao cho $p \mid a_k$ thì $k \geq 2$. Ta có

$\quad\quad\quad\quad a_k=(p-1) a_{k-1}^p-1 \text { và } p-1 \equiv-1, a_{k-1}^p \equiv a_k \quad(\bmod p) .$

Suy ra $a_{k-1} \equiv-1(\bmod p)$ từ đó ta được $a_{k-2} \equiv 0(\bmod p)$, mâu thuẫn với tính nhỏ nhất của $k$. Vậy nên ta phải có $\operatorname{gcd}\left(a_n, p\right)=1$ với mọi $n$ nguyên dương.

(b) Ta có $Q(x)=(p-1) x^p-x-1 \equiv(-1) x-x-1=-2 x-1(\bmod p)$ với mọi $x$ nguyên nên

$\quad\quad\quad\quad Q\left(a_1\right) \equiv-2 a_1-1 \equiv-2(p-1) \frac{p-1}{2}+2-1=0 \quad(\bmod p)$

nên khẳng định đúng với $n=1$. Ta sẽ chứng minh bằng quy nạp.

$\quad\quad\quad\quad Q\left(a_{n+1}\right)=(p-1)\left(a_{n+1}^p-a_n^p\right)=(p-1) Q\left(a_n\right)\left(\sum_{i=1}^p a_n^{i-1} a_{n+1}^{p-i}\right).$

Giả sử rằng $p^n \mid Q\left(a_n\right)$ nên suy ra

$\quad\quad\quad\quad a_{n+1} \equiv a_n \quad(\bmod p) \Rightarrow \sum_{i=1}^p a_n^{i-1} a_{n+1}^{p-i} \equiv p a_n^{p-1} \equiv 0 \quad(\bmod p) .$

Như vậy $p^{n+1} \mid Q\left(a_{n+1}\right)$.Theo nguyên lí quy nạp thì ta có điều phải chứng minh.

Bài 5. Cho dãy số $\left(u_n\right)$ thoả mãn $u_1=\frac{1}{6}$ và $u_{n+1}=u_n^2+\frac{2}{3} u_n \forall n \in \mathbb{N}^*$.

Tìm $\lim \frac{5 u_{n+1}^2-2 u_n^2 u_{n+1}+5 u_n u_{n+1}}{3 u_n^2+u_n u_{n+1}\left(4+u_n^2\right)}$.

Lời giải. Trước hết, ta sẽ tìm giới hạn của dãy $\left(u_n\right)$. Bằng quy nạp, ta sẽ chứng minh rằng $0<u_n<\frac{1}{3}, \forall n$. Thật vậy,

  • Với $n=1$ thì khẳng định đúng.
  • Giả sử khẳng định đúng với $n=k>1$ thì $0<u_k<\frac{1}{3}$. Ta có:

$\quad\quad\quad\quad\quad\quad\quad\quad 0<u_{k+1}=u_k^2+\frac{2}{3} u_k<\frac{1}{9}+\frac{2}{3} \cdot \frac{1}{3}=\frac{1}{3}$

nên khẳng định cũng đúng với $n=k+1$.

Theo nguyên lí quy nạp, khẳng định được chứng minh. Xét hàm số $f(x)=x^2+$ $\frac{2}{3} x, x \in\left(0 ; \frac{2}{3}\right)$ thì $f^{\prime}(x)=2 x+\frac{2}{3}>0$ nên đây là hàm đồng biến. Dãy số đã cho chính là $u_1=\frac{1}{6}, u_{n+1}=f\left(u_n\right), n=1,2,3, \ldots$

Hơn nữa $u_2=\frac{1}{6^2}+\frac{2}{3} \cdot \frac{1}{6}=\frac{5}{36}<\frac{1}{6}$ nên đây là dãy giảm và bị chặn dưới nên có giới hạn. Gọi $l$ là giới hạn của dãy thì

$\quad\quad\quad\quad\quad\quad\quad\quad l=l^2+\frac{2}{3} l \Leftrightarrow l=0 \text { hay } l=\frac{1}{3} \text {. }$

Nhưng do dãy này giảm và theo chứng minh trên thì $0<u_n<\frac{1}{3}, \forall n$ nên giới hạn của dãy là 0 .

Theo công thức xác định dãy, ta có $\frac{u_{n+1}}{u_n}=u_n+\frac{2}{3}$. Do dãy $\lim u_n=0$ nên dãy tương ứng $\left(\frac{u_{n+1}}{u_n}\right)$ có giới hạn là $\frac{2}{3}$. Từ đó, ta tính được

$\quad\quad\quad\quad\quad\quad\quad\quad\quad\quad\quad\quad\quad\quad =\frac{5\left(\frac{2}{3}\right)^2-2 \cdot 0+5 \cdot \frac{2}{3}}{3+\frac{2}{3}\left(4+0^2\right)}=\frac{50}{51} .$

Bài 6 . Cho hàm số $f: \mathbb{N} \times \mathbb{N} \rightarrow \mathbb{N}$ thoả mãn $f(0,0)=0$ và:

$\quad\quad\quad\quad\quad\quad\quad\quad f(a, b)=\left\{\begin{array}{l}f\left(\left\lfloor\frac{a}{2}\right\rfloor,\left\lfloor\frac{b}{2}\right\rfloor\right) \text { khi } a+b \equiv 0 \quad(\bmod 2) \\ 1+f\left(\left\lfloor\frac{a}{2}\right\rfloor,\left\lfloor\frac{b}{2}\right\rfloor\right) \text { khi } a+b \equiv 1 \quad(\bmod 2)\end{array}\right.$

(a) Có bao nhiêu số tự nhiên $m \leq 2011$ sao cho $f(2011, m)=5$ ?

(b) Cho số lẻ $p$, cho $n \in \mathbb{N}\left(1<p<2^n\right)$ và $A$ là tập hợp gồm $p$ số tự nhiên không vượt quá $2^n-1$. Chứng minh rằng $\sum_{{a, b} \subset A} f(a, b) \leq n \cdot \frac{p^2-1}{4}$.

Lời giải. (a) Đổi số 2011 sang hệ nhị phân, ta có $2011=\overline{11111011011}(2)$. Khi đổi số $m$ sang hệ nhị phân, ta cũng có tương ứng $m=\overline{a_1 a_2 \ldots a_{11}(2)}$ (do $m \leq 11$ nên ta chỉ xét 11 chữ số).

Do công thức xác định của hàm, ta thấy $f(2011, m)$ chính bằng số vị trí trong dãy chữ số trên mà hai chữ số tại cùng vị trí là khác tính chẵn lẻ.

Trong 11 chữ số của $m$, ta chọn 5 vị trí để cho chúng khác tính chẵn lẻ với các chữ số của 2011 thì có $C_{11}^5=462$ cách. Mỗi cách chọn tính chẵn lẻ đó tương ứng với đúng một số $m$.

Tuy nhiên, ta phải trừ đi trường hợp đổi tính chẵn lẻ tại vị trí thứ 6 (và giữ nguyên từ $a_1 \rightarrow a_5$ ), tức là

$\quad\quad\quad\quad\quad\quad\quad\quad a_1=a_2=\cdots=a_6=1$

khi đó thì $m>2011$, không thỏa mãn. Ta sẽ đếm số cách chọn $m$ như thế. Trong 5 vị trí từ $a_7 \rightarrow a_{11}$, chọn ra 4 vị trí để đổi tính chẵn lẻ, có $C_5^4=5$ cách. Chú ý rằng số 0 ở vị trí thứ 9 không ảnh hưởng vì sau nó chỉ còn 2 vị trí, không đủ để thực hiện chọn ra 4 vị trí để đổi tính chẵn lẻ như trên.

Vậy nên số các số $m$ thỏa mãn là $462-5=457$.

(b) Đổi tất cả $p$ số của tập $A$ sang hệ nhị phân thì mỗi số sẽ có không quá $n$ chữ số và xếp vào bảng ô vuông kích thước $p \times n$. Mỗi dòng tương ứng với một số, và số nào không có đủ $n$ chữ số trong hệ nhị phân thì ta thêm 0 vào trước nó. Khi đó, tổng $\sum_{{a, b} \subset A} f(a, b)$ chính bằng tổng các cặp vị trí khác nhau trên mỗi cột.

$\quad\quad\quad\quad\quad\quad\quad\quad\quad\quad\quad\quad \begin{array}{|l|l|l|l|}\hline a_{1,1} & a_{1,2} & \cdots & a_{1, n} \\ \hline a_{2,1} & a_{2,2} & \cdots & a_{2, n} \\ \hline \cdots & \cdots & \cdots & \cdots \\ \hline a_{p, 1} & a_{p, 2} & \cdots & a_{p, n} \\ \hline\end{array}$

Xét cột thứ 1 , giả sử trên đó có $x$ số 0 và $y$ số 1 với $x+y=p$. Khi đó, số cặp chữ số khác nhau trên cột này sẽ là

$\quad\quad\quad\quad x y=\frac{1}{4}\left[(x+y)^2-(x-y)^2\right]=\frac{1}{4}\left[p^2-(x-y)^2\right] \leq \frac{p^2-1}{4}$

(do $x, y$ khác tính chẵn lẻ nên $|x-y| \geq 1$ ). Tương tự với các cột khác, số cặp chữ số khác nhau cũng không vượt quá $\frac{p^2-1}{4}$. Và do tính độc lập giữa các cột, ta có

$\quad\quad\quad\quad\quad\quad\quad\quad \sum_{{a, b} \subset A} f(a, b) \leq n \cdot \frac{p^2-1}{4} .$

Bài 7. Cho tam giác $A B C$ nội tiếp đường tròn $(O)$ với $B, C$ cố định và $A$ thay đổi trên $(O)$. Đường trung trực $d$ của $B C$ cắt $A B, A C$ tại $M, N$. Gọi $P, Q$ lần lượt là các điểm đối xứng của $M, N$ qua $O$. $K$ là giao điểm của $B P$ và $C Q$.

(a) Chứng minh rằng $K$ luôn thuộc một đường tròn cố định.

(b) Kết luận trên còn đúng không khi $d$ là đường thẳng Euler của tam giác $A B C ?$

Lời giải. Ta sẽ chứng minh bài toán tổng quát khi thay trung trực $B C$ lẫn đường thẳng Euler bởi đường thẳng $d$ bất kỳ đi qua $O$.

Kẻ đường kính $B B^{\prime}, C C^{\prime}$ của $(O)$ và giả sử $B^{\prime} N, C^{\prime} M$ cắt nhau ở $T$. Khi đó, vì $M, N, O$ thẳng hàng nên theo định lý Pascal đảo thì lục giác tạo bởi các đỉnh $A, B, C, B^{\prime}, C^{\prime}, T$ nội tiếp. Do đó, $T \in(O)$.

Ngoài ra, vì $\angle B T N=\angle B T B^{\prime}=90^{\circ}$ nên $T B \perp T N$, tương tự thì $T C \perp T M$.

Kẻ đường kính $T K^{\prime}$ của $(O)$ thì do $O$ là trung diểm chung của $T K^{\prime}, M P$ nên tứ giác $T M K^{\prime} P$ là hình bình hành. Suy ra $T M | K^{\prime} P$ nên $K^{\prime} P \perp T C$.

Mà tứ giác $C M C^{\prime} P$ cũng là hình bình hành nên $C P | C^{\prime} M$, mà $C^{\prime} M \perp T C$ nên $C P \perp T C$. Từ các điều này, ta suy ra $K^{\prime}, P, C$ thẳng hàng. Tương tự thì $K^{\prime}, Q, B$ thẳng hàng. Vì thế nên $K^{\prime} \equiv K$, hay $K$ luôn thuộc đường tròn $(O)$ cố định.

Nhận xét. Trong bài toán trên, $T$ chính là giao điểm của hai đường tròn đường kính $B N, C M$. Nếu gọi $S$ là giao điểm còn lại thì ta chứng minh được bằng phép nghịch đảo trực tâm $H$ rằng $S$ nằm trên đường tròn Euler của tam giác $A B C$.

Bài 8. Với mọi số nguyên dương $n$, đặt $S_n=x^n+y^n+z^n$. Ta đã biết rằng $S_n=P_n(s, t, p)$ với $s=x+y+z, t=x y+y z+z x, p=x y z$. Hãy tính tổng các hệ số của các đơn thức chứa $p$ trong $P_{2011}(s, t, p)$.

Lời giải. Theo định lý Viete thì $x, y, z$ là nghiệm của phương trình

$\quad\quad\quad\quad\quad\quad\quad\quad\quad a^3-s a^2+t a-p=0 .$

Để tính tổng hệ số của tất cả các đơn thức trong $P_{2011}$, ta xét $P_{2011}(1,1,1)$. Tương tự, tổng các hệ số của các đơn thức không chứa $p$ trong $P_{2011}$ là $P_{2011}(1,1,0)$. Do đó, ta cần tính

$\quad\quad\quad\quad\quad\quad\quad\quad M=P_{2011}(1,1,1)-P_{2011}(1,1,0) .$

Xét phương trình $a^3-a^2+a-1=0$ có ba nghiệm là $a=1, a=i$ và $a=-i$. Vì $P_{2011}(s, t, p)=x^n+y^n+z^n$ nên ta có

$\quad\quad\quad\quad\quad\quad\quad P(1,1,1)=1^{2011}+i^{2011}+(-i)^{2011}=1 .$

Tiếp tục xét $a^3-a^2+a=0$ có ba nghiệm là $a=0, a=\frac{1 \pm i \sqrt{3}}{2}$. Áp dụng công thức Moivre của lũy thừa số phức, ta tính được

$\quad\quad\quad\quad P(1,1,0) =0^{2011}+\left(\frac{1+i \sqrt{3}}{2}\right)^{2011}+\left(\frac{1-i \sqrt{3}}{2}\right)^{2011} $

$\quad\quad\quad\quad\quad\quad\quad\quad =\left(\cos \frac{\pi}{3}+i \sin \frac{\pi}{3}\right)^{2011}-\left(\cos \frac{2 \pi}{3}+i \sin \frac{2 \pi}{3}\right)^{2011} $

$\quad\quad\quad\quad\quad\quad\quad\quad =\left(\cos \frac{2011 \pi}{3}+i \sin \frac{2011 \pi}{3}\right)-\left(\cos \frac{4022 \pi}{3}+i \sin \frac{4022 \pi}{3}\right)=1$

Vì thế nên $M=0$.